Superpage
Stomach

Authors: Diana Agostini-Vulaj, D.O., Omar Aljuboori, M.B.B.S., Naziheh Assarzadegan, M.D., Dorukhan Bahceci, M.D., Adrian C. Bateman, M.B.B.S., M.D. , Phoenix D. Bell, M.D., M.S., Kyra Berg, M.D., Satyapal Chahar, M.D., Irene Y. Chen, M.D., Runjan Chetty, M.B.B.Ch., Ph.D., Tony El Jabbour, M.D., David J. Escobar, M.D., Ph.D., Matteo Fassan, M.D., Ph.D., Jennifer Findeis-Hosey, M.D., Yujun Gan, M.D., Ph.D., Monica T. Garcia-Buitrago, M.D., Giby V. George, M.B.B.S., Raul S. Gonzalez, M.D., Nathalie Guedj, M.D., Ph.D., Saroona Haroon, M.B.B.S., Aaron R. Huber, D.O., Gagandeep Kaur, M.D., Xiuli Liu, M.D., Ph.D., Natalia Liu, M.D., Teri A. Longacre, M.D., Yuxin Lu, M.D., Carolina Martinez Ciarpaglini, M.D., Ph.D., Kelsey E. McHugh, M.D., Hunter Monroe, B.S., Matthew Morrow, M.D., Emma J. Norton, M.B.B.S., Klaudia M. Nowak, M.D., Byoung Uk Park, M.D., Nat Pernick, M.D., Thomas P. Plesec, M.D., Dana Razzano, M.D., Jen Rytych, M.D., Namrata Setia, M.D., Divya Sharma, M.D., Tiffany Sheganoski, D.O., Natalya Shlyakhova, M.D., Supriya Srivastava, M.D., Ph.D., Tanner Storozuk, M.D., Iresha Vithanage, M.B.B.S., M.D., Monika Vyas, M.D., Hanlin L. Wang, M.D., Ph.D. , Cindy Wang, M.D., Elliot Weisenberg, M.D., Kwun Wah Wen, M.D., Ph.D., Mohamed Yakoub, M.D., Feng Yin, M.D., Ph.D., Tao Zhang, M.D., Lizhi Zhang, M.D.
Editorial Board Members: Diana Agostini-Vulaj, D.O., Naziheh Assarzadegan, M.D., Wei Chen, M.D., Ph.D., Raul S. Gonzalez, M.D., Catherine E. Hagen, M.D., Aaron R. Huber, D.O., Xiaoyan Liao, M.D., Ph.D., Claudio Luchini, M.D., Ph.D., Maryam Kherad Pezhouh, M.D., M.Sc.
Deputy Editors-in-Chief: Raul S. Gonzalez, M.D., Catherine E. Hagen, M.D., Aaron R. Huber, D.O.
Editor-in-Chief: Debra L. Zynger, M.D.

Copyright: 2003-2024, PathologyOutlines.com, Inc.

GI related: Jobs, Fellowships, Conferences, Cases, CME, Board Review

Related chapters: Esophagus, Gallbladder & extrahepatic bile ducts, Liver & intrahepatic bile ducts, Pancreas

Editorial Board oversight: Diana Agostini-Vulaj, D.O. (last reviewed December 2023), Aaron R. Huber, D.O. (last reviewed May 2023)
Page views in 2024 to date: 47

Acute gastric ulcer
Definition / general
  • Also called stress ulcer
  • Severe form of acute gastritis in which erosion reaches muscularis mucosa
  • Some authors consider large erosions to be stress ulcers
  • If underlying condition resolves, complete healing occurs
  • Common autopsy finding
  • Causes: NSAIDs, steroids, severe physiologic stress (shock, extensive burns, sepsis, severe trauma, increased intracranial pressure, post-intracranial surgery, intensive care unit), systemic amyloidosis, post-chemotherapy, radiation therapy
  • Complications: bleeding (may be massive; 1-4% of patients require transfusion), perforation, obstruction from edema / scarring

  • Specific forms:
    • Curling ulcer: severe burns and duodenal ulcer
    • Cushing ulcer: increased intracranial pressure and gastric, esophageal, or duodenal ulcers
Gross images

Images hosted on other servers:

Gastric ulcer

Microscopic (histologic) description
  • Usually < 1 cm, often multiple, small brown circular ulcer base (digested blood) anywhere in stomach
  • No induration of margins of ulcer - abrupt lesions with normal adjacent mucosa; no scarring or blood vessel thickening
Microscopic (histologic) images

Images hosted on other servers:

Various images


Acute gastritis
Definition / general
  • Acute gastritis is a sudden inflammation or swelling in the lining of the stomach characterized histologically by inflammatory cells in the lamina propria of the gastric mucosa (predominantly polymorphonuclear leukocytes); the leukocytes may also penetrate the glandular lumen or submucosa based on the severity of the lesion
  • According to the Kyoto Global Consensus Report, gastritis is best classified based on etiology or atrophic versus nonatrophic gastritis and not based on duration (Curr Treat Options Gastroenterol 2020;18:488)
Essential features
  • Sudden inflammation occurring in short interval of time
  • Usually associated with polymorphonuclear leukocyte infiltration
  • Commonly associated with nonsteroidal anti-inflammatory drugs (NSAIDs), steroid and alcohol usage
  • Acute gastritis commonly includes erosive or acute hemorrhagic gastritis, nonerosive gastritis and rarely, acute Helicobacter gastritis and acute suppurative / necrotizing or acute phlegmonous gastritis
Terminology
  • Active gastritis
ICD coding
  • ICD-10:
    • K29.0 - acute gastritis
      • K29.00 - acute gastritis without bleeding
      • K29.01 - acute gastritis with bleeding
    • K29.2 - alcoholic gastritis
      • K29.20 - alcoholic gastritis without bleeding
      • K29.21 - alcoholic gastritis with bleeding
Epidemiology
  • No gender predilection
  • Strongly associated with alcohol intake, smoking, usage of NSAIDs and steroids
Sites
  • Entire stomach with a predilection in antrum
Pathophysiology
  • Acute gastritis occurs when the mucosal defense mechanism is disturbed due to inflicting agents like alcohol, NSAIDs, steroids, chemotherapeutic agents, etc.; this weakened mucosal defense allows the acid to further damage the mucosal lining
  • In acute phlegmonous gastritis, transmural infection is caused by Staphylococcus, Streptococcus, Clostridium or Escherichia coli
  • Acute gastritis could also be caused by infection from H. pylori wherein the bacilli may cause transient hypochlorhydria and epigastric pain; however, it is self limiting and later may present as chronic gastritis and H. pylori infection (World J Gastroenterol 2014;20:5461)
Etiology
  • Drugs (NSAIDs like aspirin, ibuprofen, naproxen), Helicobacter species (H. pylori and H. heilmanii), stress, uremia, ischemia, shock, corrosive agents, sepsis, trauma, alcohol, cocaine, severe burns, major surgery (Dig Liver Dis 2000;32:34)
  • Rarely by chemotherapeutic agents like nivolumab, pembrolizumab, cytomegalovirus and Epstein-Barr virus, Staphylococcus, Streptococcus and Clostridium species (Dig Dis Sci 2021;66:2461, Clin J Gastroenterol 2019;12:274)
Clinical features
  • Most patients are asymptomatic
  • Nonspecific epigastric discomfort, abdominal pain, nausea, vomiting and loss of appetite could be presenting features in some
  • May present as upper gastrointestinal bleeding (hematemesis) due to gastric erosions and ulcerations or rarely as melena (black tarry stools)
  • Phlegmonous gastritis presents as a medical emergency with severe abdominal pain, nausea, vomiting (Medicine (Baltimore) 2019;98:e17777)
Diagnosis
  • By patient history; confirmation by endoscopy and biopsy
Laboratory
  • Complete blood count (CBC): elevated white blood cell counts with neutrophilia, with or without low hemoglobin
    • Increased lactate dehydrogenase (LDH)
    • Stool test for bleeding
    • Urease level could be helpful in predicting acute gastritis in patients with dyspepsia (Acta Med Indones 2022;54:42)
Radiology description
Radiology images

Images hosted on other servers:

CT scan of the stomach

Prognostic factors
  • Usually resolves by itself within a few days or weeks once the inflammation settles and the causative agents are removed
  • Can progress to chronic gastritis if not treated; in case of Helicobacter pylori infection, it usually leads to chronic gastritis, which further increases the risk of atrophic gastritis and sometimes gastric cancer
Case reports
Treatment
  • Self limiting and usually resolves upon removal of the causative factors
  • Proton pump inhibitor therapy and H2 antagonists
  • Eradication therapy in cases of acute H. pylori gastritis (lansoprazole, clarithromycin, amoxicillin and rebapemide) (J Gastroenterol Hepatol 2000;15:1377)
  • Broad spectrum antibiotics in cases of phlegmonous or suppurative gastritis
Clinical images

Images hosted on other servers:

Acute phlegmonous gastritis

Gross description
  • Superficial erosive lesions
  • Swollen edematous stomach wall, with or without pus accumulation in acute phlegmonous gastritis
Gross images

Images hosted on other servers:

Acute phlegmonous gastritis

Diffusely hyperemic gastric mucosa

Areas of gastric hemorrhage depicting gastric erosions

Microscopic (histologic) description
  • For alcoholic, stress related or NSAID induced gastritis, initially all the segments of the stomach are involved, followed by predominance in antrum
  • Diffuse edema in lamina propria, capillary congestion and interstitial hemorrhage in varying degrees
  • Erosions may or may not be present, with regenerative epithelium as the only evidence
  • A few neutrophils are present (Am J Surg Pathol 1996;20:1161)
  • Acute phase of Helicobacter (though rarely encountered) shows marked degenerative changes in mucus epithelium including mucus depletion, cellular exfoliation and syncytial regenerative changes
  • Neutrophilic infiltration on the surface and foveolar epithelium along with pit abscesses can be seen (Am J Surg Pathol 1996;20:1161)
  • In acute phlegmonous gastritis, there is diffuse necrosis in the stomach wall extending from submucosa to muscularis layer to the serosa; there is an accumulation of pus and extensive infiltration of neutrophils (Medicine (Baltimore) 2018;97:e0629)
Microscopic (histologic) images

Contributed by Supriya Srivastava, M.D., Ph.D. and Xiaoyan Liao, M.D., Ph.D.
Acute gastric erosion Acute gastric erosion Acute gastric erosion

Acute gastric erosion

Acute gastritis

Acute gastritis

Acute phlegmonous gastritis

Acute phlegmonous gastritis

Positive stains
Sample pathology report
  • Stomach, antral biopsy:
    • Stomach biopsy shows acute erosive gastritis (see comment)
    • Comment: No evidence of intestinal metaplasia, H. pylori, dysplasia or malignancy.
Differential diagnosis
  • Peptic ulcer disease:
    • Peptic ulcer disease presents as round or oval ulcer / crater in the gastric mucosa, while acute gastritis usually presents with redness, superficial erosions or shallow ulcers
    • Duration of disease is another important differentiating feature between peptic ulcer disease and acute gastritis
  • Chronic gastritis:
    • Longstanding duration
    • Presence of chronic inflammatory infiltrate like plasma cells and lymphocytes
  • Chronic H. pylori gastritis:
    • Chronic inflammatory infiltrate with plasma cells and lymphocytes and neutrophils
Board review style question #1
A 40 year old working professional woman visited her general practitioner with complaints of abdominal pain, nausea and loss of appetite. On eliciting her clinical history, the general practitioner found that she had some deadlines to meet in her office and was working overtime; in addition, she was taking ibuprofen for her body pain during the past few days. The general practitioner referred her to a gastroenterologist for an endoscopy. What could be the possible endoscopic finding?

  1. 2 cm friable lesion in the fundus of the stomach
  2. 5 cm ulcer at lesser curvature with irregular margins
  3. Superficial mucosal erosion with erythema
  4. Thickened and tortuous mucosal folds
Board review style answer #1
C. Superficial mucosal erosion with erythema. In this context, this endoscopy most likely shows features of acute gastritis induced by the frequent usage of ibuprofen along with overproduction of gastric acid due to anxiety and stress. Answer D is incorrect because thickened and tortuous folds are usually seen in chronic gastritis, Menetrier disease or malignancy. Answers A and B are incorrect because they are characteristic features of malignant ulcers.

Comment Here

Reference: Acute gastritis
Board review style question #2

A 52 year old woman with 4 weeks of acute bacterial sinusitis was treated with a course of doxycycline combined with aggressive nasal lavage. She then developed dyspepsia. Upper endoscopy was performed and found patches of mild erythema in the antrum and body; additionally, there were areas of pale yellow mucosa in the cardia and fundus. Biopsy was performed to rule out H. pylori, intestinal metaplasia, dysplasia or malignancy. What would be the most likely diagnosis of this patient?

  1. Acute gastritis
  2. Chronic active gastritis
  3. Chronic gastritis
  4. Signet ring cell carcinoma
Board review style answer #2
A. Acute gastritis. This is a typical case of doxycycline induced acute gastritis. In the representative image, notice neutrophil infiltrates and superficial mucosal capillaries with characteristic eosinophilic degenerative changes. Answer C is incorrect because chronic gastritis features lymphoplasmacytic infiltrates more than neutrophils. Answer B is incorrect because chronic active gastritis is usually seen in patients with Helicobacter pylori infection. The cells are degenerative looking rather than dysplasia. Answer D is incorrect because there are no atypical cells or signet ring cell carcinoma in this biopsy.

Comment Here

Reference: Acute gastritis

Adenomas
Definition / general
  • Intestinal type adenoma: polypoid lesion with dysplastic intestinalized epithelium
  • Foveolar type adenoma: polypoid lesion with dysplastic foveolar epithelium
Essential features
  • Endoscopically visible polypoid lesion
  • Classified into low grade or high grade dysplasia
  • No lamina propria invasion
  • Foveolar type adenoma maintains apical mucin cap
Terminology
  • Foveolar type adenoma; gastric adenoma, foveolar type
  • Intestinal type adenoma; gastric adenoma, intestinal type
ICD coding
  • ICD-O: 8210/0 - adenomatous polyp, NOS
  • ICD-10: D13.1 - benign neoplasm of stomach
  • ICD-11: 2E92.1 & XH3DV3 - benign neoplasm of stomach & adenoma, NOS
Epidemiology
  • Prevalence of gastric adenomas (Microorganisms 2021;9:108)
    • Intestinal type adenoma is the most frequent adenoma (89.1%)
    • Foveolar type adenoma (4.3%)
    • Pyloric gland adenoma (3.4%)
    • Adenomas associated with hereditary tumor syndromes (2.8%)
    • Oxyntic gland adenoma (0.4%)
Sites
  • Intestinal type adenoma: commonly occurs in stomach compartment where intestinal metaplasia commonly occurs (i.e., antrum and angularis incisura)
  • Foveolar type adenoma: commonly occur in oxyntic mucosa (i.e., body and fundus)
Pathophysiology
  • Genetic or epigenetic factors that promote intestinal metaplasia
Etiology
  • Sporadic and syndromic
  • Intestinal type adenoma
    • Associated with intestinal metaplasia
      • H. pylori infection
      • Autoimmune gastritis
      • Syndromes with APC gene mutations (i.e., familial adenomatous polyposis [FAP])
  • Foveolar type adenoma
    • Commonly associated with polyposis syndromes
      • Familial adenomatous polyposis
      • Gastric adenocarcinoma and proximal polyposis of the stomach (GAPPS)
    • Sporadic is rare
Clinical features
  • Predominantly asymptomatic
  • Large adenomas may cause recurrent bleeding and anemia
  • Associated with metachronous gastric adenocarcinoma
Diagnosis
  • Diagnosed by upper endoscopy with biopsy / polypectomy
Prognostic factors
  • Intestinal type gastric adenoma is more likely (but still with low probability) to progress to gastric cancer as compared to foveolar type gastric adenoma
  • Foveolar type gastric adenoma with hybrid phenotype (both intestinal and gastric differentiation) carries higher cancer risk than only intestinal type adenoma
  • Adenomas with low grade dysplasia have an annual malignant transformation rate of 0.6% and those with high grade dysplasia have annual malignant transformation rate of 6% (Histopathology 2021;78:106)
Case reports
Treatment
  • Endoscopic mucosal resection or endoscopic mucosal dissection
Gross description
  • Intestinal type adenomas are flatly elevated whitish polyps
  • Foveolar type adenomas are elevated lesions with a smooth surface and contractions (Digestion 2021;102:878)
  • Usually small singular polyps, < 2 cm in largest dimension
  • In FAP patients, usually coexist with multiple fundic gland polyps
Microscopic (histologic) description
  • Intestinal type adenoma
    • Polypoid dysplastic columnar epithelium and tubules with varying degrees of Paneth and goblet cells (intestinal differentiation)
    • Hyperchromatic elongated nuclei, similar to those seen in a colon tubular adenoma
    • Background mucosa may exhibit chronic gastritis with intestinal metaplasia
    • Lacks apical mucin caps
  • Foveolar type adenoma
    • Dysplastic columnar epithelium (foveolar type) with apical mucin caps
    • Background gastric mucosa usually otherwise without abnormality (no inflammation, atrophy or metaplasia)
    • May contain both intestinal and gastric differentiation in the same adenoma (hybrid phenotype)
  • Dysplasia graded as low grade or high grade
    • If high grade dysplasia, staged as pTis
Microscopic (histologic) images

Contributed by Cindy Wang, M.D.
Foveolar type adenoma

Foveolar type adenoma

Intestinal type adenoma

Intestinal type adenoma

Hybrid phenotype

Hybrid phenotype

Positive stains
Negative stains
  • Foveolar type adenoma: MUC2
  • Intestinal type adenoma: MUC5AC
Sample pathology report
  • Stomach, fundus polyp, biopsy:
    • Fragments of gastric adenoma, foveolar type (see comment)
    • Comment: There is no evidence of high grade dysplasia. Clinical and endoscopic correlation recommended to ensure adequacy of excision.

  • Stomach, antral polyp, endoscopic mucosal resection:
    • Gastric adenoma, intestinal type, with focal high grade dysplasia, 1.3 cm, excised (see comment)
    • Comment: There is no evidence of lamina propria or muscularis mucosae invasion. There is no adenomatous epithelium at resection margins. The background gastric mucosa shows intestinal metaplasia. No Helicobacter organisms are present.
Differential diagnosis
  • Reactive atypia:
    • Uniform nuclear enlargement with visible nucleoli without crowding
    • Cellular maturation at mucosal surface
    • Surrounding inflammation, intestinal metaplasia or foveolar hyperplasia
  • Other types of adenomas:
    • Oxyntic adenoma:
      • Irregular glands lined by columnar cells with a predominant chief cell component, mimicking oxyntic glands
    • Pyloric gland adenoma:
      • Closely packed glands lined by cuboidal to low columnar epithelium with ground glass cytoplasm, bland nuclei and without apical mucin
  • Gastric polyps with dysplasia:
    • Fundic gland polyp with dysplasia:
      • Background gastric mucosa shows dilated parietal cell lined glands with low or high grade epithelial dysplasia
    • Hyperplastic polyp with dysplasia:
      • Background gastric mucosa with cystically dilated, branching crypts and edematous lamia propria with mixed inflammatory infiltrate and low or high grade epithelial dysplasia
  • Intramucosal adenocarcinoma:
    • Complex architecture and significant atypia with invasion into lamina propria
    • Variable desmoplastic stromal change
Board review style question #1
A polyp is identified in the gastric antrum. The background nonlesional gastric mucosa shows extensive Helicobacter organisms and moderate amounts of intestinal metaplasia. What is the most likely diagnosis of this lesion?

  1. Foveolar type adenoma
  2. Fundic gland polyp
  3. Intestinal type adenoma
  4. Pyloric gland adenoma
Board review style answer #1
C. Intestinal type adenoma is commonly found in the antrum with association to background intestinal metaplasia. Longstanding H. pylori infection is a common cause of intestinal metaplasia. Intestinal type adenomas are far more common than foveolar type adenoma and pyloric gland adenomas. Fundic gland polyps are found in oxyntic mucosa (i.e., gastric body or fundus).

Comment Here

Reference: Adenomas
Board review style question #2

What is the diagnosis for the image above?

  1. Intestinal type adenoma
  2. Intestinal type adenoma with high grade dysplasia
  3. Intramucosal adenocarcinoma
  4. Reactive atypia
Board review style answer #2
C. Intramucosal adenocarcinoma. Section shows complex glandular architecture with back to back and cribriforming glands. The nuclei show significant atypia. Focally, there is glandular budding. The nuclei show significant atypia with nuclear rounding and loss of polarity. All features are consistent with intramucosal adenocarcinoma.

Comment Here

Reference: Adenomas

Amyloid
Definition / general
  • Deposition of insoluble extracellular protein, usually AL (light chains associated with myeloma) or AA (acute phase proteins secondary to chronic inflammation or familial Mediterranean fever) types
  • Also associated with chronic dialysis (β2-microglobulin)
  • Rarely idiopathic
  • 70% of cases of AL amyloidosis and 55% of AA amyloidosis involve GI tract, often stomach
  • Primary amyloidosis (AL) patients often have monoclonal proteins in serum / urine

  • Symptoms:
    • Frequently none
    • Bloating, pain, obstruction, hematemesis, hemorrhage
Gross description
  • Usually normal (multiple biopsies recommended in patients with systemic amyloidosis)
  • May have diffuse involvement and form a mass
Microscopic (histologic) description
  • Appears as amorphous, waxy, salmon pink material, often with shatter artifact
  • Often infiltration around blood vessels deep to mucosa
Positive stains
  • Congo red (red / green birefringence under polarized light), Thioflavin immunofluorescence
Electron microscopy description
  • 7.5 to 10 nanometer fibrils in twisted beta-pleated sheets

Anatomy
Definition / general
  • Normal volume is 1.5 liters, capacity is 3 liters, potentially more
  • Fine mosaic-like pattern of mucosa because of punctuations by gastric pits or foveolae
  • Longitudinal infoldings of mucosa and submucosa known as rugae are coarser proximally and when stomach is empty

  • Cardia:
    • Narrow conical portion distal to gastroesophageal junction
    • Many authors claim that cardiac mucosa is reflux-associated epithelia and not normally present
    • Of note in the AJCC Cancer Staging Manual, 7th Edition (2010), carcinomas of the esophagus and esophagogastric junction (the proximal 5 cm of the stomach) are staged identically

  • Fundus:
    • Dome shaped proximal stomach

  • Body / corpus:
    • Remainder of stomach to incisura angularis

  • Incisura angularis:
    • Where stomach narrows before it joins duodenum

  • Antrum:
    • Incisura angularis to pyloric sphincter (3-4 cm)

  • Pylorus:
    • Muscular ring that controls flow of food content into proximal duodenum

  • Lesser curvature:
    • Medial curvature of stomach on the right

  • Greater curvature:
    • Lateral curvature of stomach
Gross images

Contributed by Leon Metlay, M.D.

22 week fetus with congenital diaphragmatic hernia

Liver and stomach in chest, heart shifted to right, both lungs small

Hypoplastic left lung and left heart

Hypoplastic right lung

Groove in liver, where diaphragm edge was



Images hosted on other servers:

Normal appearance of the stomach, opened along the greater curvature

Normal appearance of the gastric antrum extending to the pylorus at the right of center


Anatomy & histology
Definition / general
  • A review of the normal constituents of the gastric wall, with a focus on the gastric mucosa, its compartments, its cell types and their cellular products
Essential features
  • Anatomic regions: cardia, fundus, body, antrum / pylorus
  • Layers: mucosa, submucosa, muscularis propria, subserosa, serosa
  • Cell types: mucous cells, parietal cells, chief cells, endocrine cells
  • Cardia and antrum: mucinous glands
  • Body and fundus: oxyntic glands
Anatomy
  • Normal volume is 1.5 liters, capacity is 3 liters, potentially more
  • Fine mosaic-like pattern of mucosa because of punctuations by gastric pits or foveolae
  • Longitudinal infoldings of mucosa and submucosa known as rugae are coarser proximally and when stomach is empty
  • Cardia
    • Narrow conical portion distal to gastroesophageal junction
    • Many authors claim that cardiac mucosa is reflux associated epithelia and not normally present
    • Of note in the AJCC Cancer Staging Manual, 8th Edition (2017), carcinomas of the esophagus and the esophagogastric junction (tumor with epicenters ≤ 2 cm into cardia) are staged identically
  • Fundus
    • Dome shaped proximal stomach
  • Body / corpus
    • Remainder of stomach to incisura angularis
  • Incisura angularis
    • Where stomach narrows before it joins duodenum
  • Antrum
    • Incisura angularis to pyloric sphincter (3 - 4 cm)
  • Pylorus
    • Muscular ring that controls flow of food content into proximal duodenum
  • Lesser curvature
    • Medial curvature of stomach on the right
  • Greater curvature
    • Lateral curvature of stomach
Physiology
  • General functions: digestion, motility, microbial defense (Statpearls: Physiology, Stomach [Accessed 28 May 2020])
  • Digestive system organ that receives contents from the esophagus via the gastroesophageal sphincter and empties its contents into the duodenum via the pyloric sphincter
    • Partially digests food boluses received from the esophagus
      • Mechanical digestion: back and forth churning by inner oblique layer of muscularis propria
      • Chemical digestion: acidic milieu breaks down proteins and kills food derived microbes
        • Parietal cells secrete hydrochloric acid (HCl), which maintains acidic pH of stomach and denatures proteins
        • Chief cells secrete pepsinogen, which breaks down proteins when activated to pepsin by the acidic environment
        • Microenvironment of the stomach is largely regulated by enteroendocrine cell hormone secretion and vagus nerve innervation
    • Empties chyme (partially digested food) into the duodenum
      • Gastric emptying largely facilitated by the inner circular and outer longitudinal layers of muscularis propria
        • Smooth muscle contractions (peristalsis) are controlled by the interstitial cells of Cajal (J Physiol 2006;571:1)
      • Majority of nutrient absorption will occur in small bowel
Diagrams / tables

Contributed by Kelsey E. McHugh, M.D.
Missing Image

Anatomic regions

Missing Image

Layers of gastric wall

Missing Image

Gastric mucosa

Clinical images

Contributed by Kelsey E. McHugh, M.D.
Missing Image

Gastric body with
unremarkable
rugal folds

Gross description
  • A malleable gastric wall with pink, smooth, glistening serosa and pink-tan velvety mucosa arranged in coarse rugal folds which are more prominent in the body and fundus, whereas antral mucosa is more flat (Am J Surg Pathol 1986;10:48)
Gross images

Contributed by Leon Metlay, M.D. and Kelsey E. McHugh, M.D.

Anatomy:
Missing Image

Formalin fixed gastric body

Missing Image

Formalin fixed gastric rugal folds



Images hosted on other servers:

Anatomy:

Normal appearance of the stomach

Normal appearance of the gastric antrum

Missing Image Missing Image

Total gastrectomy anatomic regions

Microscopic (histologic) description
  • Layers:
    • Mucosa (with muscularis mucosae)
    • Submucosa (with Meissner plexus)
    • Muscularis propria (outer longitudinal layer, Auerbach / myenteric plexus, inner circular layer, innermost oblique layer)
    • Subserosa
    • Serosa (Am J Surg Pathol 1986;10:48)
  • Anatomic regions:
    • Cardia: variable length extending from 1 - 15 mm (average 5 mm) (Am J Surg Pathol 2002;26:1207)
    • Body (corpus) and fundus: known as oxyntic mucosa
      • Tightly packed glands acid and enzyme secreting glands
      • Ratio of pit to gland volume, 25:75
      • Parietal cells and chief cells are the glandular constituents
      • Can have interspersed mucin cells, especially in glandular isthmus / neck
      • Corpus antrum boundary moves proximally with age due to reduction of oxyntic mucosal volume
    • Antrum / pylorus: distal 3 - 4 cm
      • Loosely packed mucous secreting glands
      • Ratio of pit to gland volume, 50:50
      • Usually no cystic dilatation of glands
      • G cells are an endocrine cell unique to this anatomic region
      • Proximal extent along the lesser curvature exceeds that along the greater curvature (Gastroenterology 1972;63:584)
  • Components of gastric mucosa:
    • Gastric pits: surface invaginations that function as conduits of secretions; entirely lined by surface mucous (foveolar) cells regardless of anatomic region (Am J Surg Pathol 1986;10:48)
    • Gastric glands: synthesize acids, enzymes and mucins; constituents and their products vary depending on anatomic region of the stomach
  • Gastric cell types
    • Mucous cells
      • Secrete bicarbonate rich mucus that coats and lubricates the gastric surface
        • Serves protective function against autodigestion
      • Surface (foveolar) epithelium contains cytoplasmic neutral mucins
        • Lightly eosinophilic apical mucin cap
      • Mucous glands contain cytoplasmic neutral and acidic mucin (Arch Pathol 1968;85:580)
        • Lightly eosinophilic to clear bubbly / vacuolated cytoplasm
        • Lack an apical mucin cap
      • All mucous cells have round, basally oriented nuclei
    • Parietal cells
      • Produce hydrochloric acid via H+ / K+ -ATPase pump (Varela: Histology, Parietal Cells, 2019)
        • Hydrochloric acid maintains gastric acidity (pH 1.5 to 3.5), which:
          • Activates pepsinogen to active pepsin enzyme
          • Kills food derived bacteria
          • Facilitates food digestion
          • Promotes absorption of minerals (e.g. phosphate, calcium, iron)
      • Produce intrinsic factor, which is critical for vitamin B12 absorption in small bowel (Varela: Histology, Parietal Cells, 2019)
      • Secretion highly regulated by extrinsic and intrinsic neuroendocrine system (Physiol Rev 2020;100:573)
        • Stimulators of gastric acid secretion include:
          • Vagus nerve (via acetylcholine neurotransmitter)
          • Gastrin hormone
            • Hypergastrinemia can be induced by glucocorticoids
          • Histamine
          • Grehlin
        • Inhibitors of gastric acid secretion include:
          • Somatostatin
          • Pharmacologic agents (e.g. proton pump inhibitors)
      • Relatively large, triangular cells with eosinophilic cytoplasm due to abundant mitochondria
      • Centrally placed nuclei with evenly distributed chromatin
      • Predominantly occupy the superficial half of body / fundic glands
    • Chief cells
      • Produce pepsinogen I and II propeptides (Gastroenterology 1992;102:699)
        • Activated to pepsin enzyme via high acidity (low pH) environment
      • Cuboidal cells with basophilic to amphophilic cytoplasm due to abundant rough endoplasmic reticulum
      • Predominantly occupy the basal half of corpus glands
    • Endocrine cells
      • Generally inconspicuous, round cells with clear cytoplasm
      • Typically fewer than 20 endocrine cells per gland
      • Cell types include: (Regul Pept 2000;93:31)
        • G cells: produce gastrin
          • Limited to the gastric antrum
        • D cells: produce somatostatin
          • Distributed throughout whole stomach
        • Enterochromaffin (EC) cells: produce serotonin
          • Distributed throughout whole stomach
        • Enterochromaffin-like (ECL) cells: produce histamine (Int J Mol Sci 2019;20:E2444)
          • Limited to the gastric body and fundus
          • Secrete histamine in response to gastrin produced by G cells
          • Represent 30% of all endocrine cells
          • Long term gastrin stimulation causes enterochromaffin-like cell hyperplasia (e.g. in atrophic gastritis)
        • X cells: produce grehlin (Int J Pept 2010;2010:945056)
          • Limited to the gastric body and fundus
      • Concentrated within mucous neck region in antrum / pylorus
        • 50% G cells, 30% enterochromaffin cells, 15% D cells, 5% other
      • Scattered throughout oxyntic glands in body and fundus
        • Majority enterochromaffin-like cells; minority enterochromaffin cells, X cells, D cells
  • Gastric mucosa - general:
    • There are no visibly (grossly or microscopically) distinct boundaries between mucosal zones / anatomic regions (Am J Surg Pathol 1986;10:48)
    • Gastric mucosa is very metabolically active
    • Gastric surface epithelium is normally replaced every 4 - 8 days (Gastroenterology 1977;72:962)
    • Gastric parietal and chief cells turn over more slowly: every 1 - 3 years (J Natl Cancer Inst 1969;42:9)
    • Undifferentiated stem cells are concentrated in the isthmus / neck of gastric glands throughout the entire stomach
    • Lamina propria provides structural support (Am J Surg Pathol 1986;10:48)
      • Structural components: reticulin, collagen and elastin fibers; capillaries and arterioles; nerve fibers; few smooth muscle fibers
      • Cell types: fibroblasts, histiocytes, plasma cells, lymphocytes; occasional mast cells and eosinophils
    • Lymphoid tissue is sparse in lamina propria (Hum Pathol 1993;24:577)
      • Superficial lamina propria with small numbers of lymphocytes (B cells > T cells) and plasma cells
      • Can see small lymphoid aggregates (no germinal centers) in gastric antrum immediately superficial to muscularis mucosae
      • Intraepithelial lymphocytes absent or sparse
    • Lymphatic channels are present within gastric mucosa, generally immediately superficial to muscularis mucosae
    • Gastric mucosa protects itself against autodigestion / high acidity (Physiol Rev 1993;73:823)
      • Mucus secretion: mucus is relatively impermeable to H+; also fluid with acid or pepsin exits gastric glands as jets and penetrates surface mucus layer without contacting surface epithelial cells (Am J Physiol Gastrointest Liver Physiol 2017;313:G599)
      • Bicarbonate secretion creates pH neutral microenvironment adjacent to cell surface
      • Intercellular tight junctions prevent back diffusion of H+ and disruptions are quickly repaired
      • Rich blood flow supplies bicarbonate and nutrients and removes acid
      • Muscularis mucosae limits injury; if intact, mucosal repair occurs in hours / days versus weeks if not intact
Microscopic (histologic) images

Contributed by Kelsey E. McHugh, M.D.
Missing Image

Gastric wall layers

Missing Image

Oxyntic mucosa

Missing Image

Antral mucosa

Missing Image

Gastrin immunostain

Missing Image

Cardia mucosa

Missing Image

Parietal and chief cells


Missing Image

Gastric pits

Missing Image

Pseudo-signet ring cell artifact

Missing Image

Distended foveolar cells

Missing Image

PAS / AB in distended foveolar cells

Missing Image

Neuroendocrine cells in gastric antrum

Virtual slides

Images hosted on other servers:
Missing Image

Unremarkable gastric body

Cytology description
  • Mucous cells: tightly cohesive columnar cells with basal round to ovoid nuclei in orderly, honeycombed sheets
    • Usually the predominant cell type
  • Chief cells: small cuboidal cells with round, smooth nuclei, fine nuclear chromatin and basophilic cytoplasmic granules in honeycombed sheets
    • Comparable in appearance to pancreatic or salivary acinar cells
  • Parietal cells: pyramidal to flask shaped cells with round nuclei, coarse nuclear chromatin and abundant intensely eosinophilic cytoplasm
    • Larger than chief cells
Cytology images

Contributed by Kelsey E. McHugh, M.D.
Missing Image

Surface foveolar epithelium

Missing Image

Parietal cells

Missing Image

Chief cells

Positive stains
Sample pathology report
  • Stomach, cardia, biopsy:
    • Gastric cardia mucosa with no significant diagnostic alteration
    • No evidence of Helicobacter pylori microorganisms
  • Stomach, body / fundus, biopsy:
    • Gastric oxyntic mucosa with no significant diagnostic alteration
    • No evidence of Helicobacter pylori microorganisms
  • Stomach, antrum, biopsy:
    • Gastric antral mucosa with no significant diagnostic alteration
    • No evidence of Helicobacter pylori microorganisms
Board review style question #1

    Which gastric cell type, pictured above, produces intrinsic factor, an integral glycoprotein in the luminal absorption of vitamin B12?

  1. Chief cells
  2. Endocrine cells
  3. Enterochromaffin-like cells
  4. Mucous cells
  5. Parietal cells
Board review style answer #1
E. Parietal cells

Comment Here

Reference: Anatomy & histology
Board review style question #2
    The Meissner plexus is located within which layer of the stomach?

  1. Mucosa
  2. Muscularis propria
  3. Serosa
  4. Submucosa
  5. Subserosa
Board review style answer #2
D. Submucosa

Comment Here

Reference: Anatomy & histology
Board review style question #3
    Which of the following ratios most accurately described the gastric pit to gastric gland ratio within the corpus (body / fundus) of the stomach?

  1. 10:90
  2. 25:75
  3. 50:50
  4. 75:25
  5. 90:10
Board review style answer #3
B. 25:75

Comment Here

Reference: Anatomy & histology

Arteriovenous malformation
Definition / general
  • Arteriovenous malformation is rare
  • Dieulafoy lesion
Terminology
  • Dieulafoy lesion is also called caliber persistent artery, Dieulafoy disease
Case reports
Microscopic (histologic) description
  • Dieulafoy lesion
Microscopic (histologic) images

Contributed by Raul Gonzalez, M.D.
Gastric arteriovenous malformation

Gastric arteriovenous malformation


Atrophic gastritis
Definition / general
  • Type of chronic gastritis characterized by prominent loss of glands secondary to chronic environmental (Helicobacter pylori infection) or autoimmune etiologies
Essential features
  • Destruction of glands occurs in gastric body / fundus in autoimmune type and in the antrum in environmental type
  • Associated laboratory findings include increased gastrin, decreased pepsinogen I, decreased pepsinogen I/pepsinogen II ratio, iron deficiency and B12 deficiency
  • Metaplastic epithelial changes may become dysplastic and progress to gastric adenocarcinoma
  • Decreased acid production can lead to enterochromaffin-like (ECL) cell hyperplasia and development of well differentiated neuroendocrine tumors
Terminology
  • Environmental (metaplastic) atrophic gastritis (EMAG), antrum restricted atrophic gastritis, multifocal atrophic gastritis, type B gastritis
  • Autoimmune (metaplastic) atrophic gastritis (AMAG), atrophic autoimmune gastritis, type A gastritis
ICD coding
  • ICD-10
    • K29.40 - chronic atrophic gastritis without bleeding
    • K29.41 - chronic atrophic gastritis with bleeding
  • ICD-11: DA42.73 - chronic atrophic gastritis of unknown etiology
Epidemiology
Sites
  • Environmental atrophic gastritis typically initiates in the antrum of the stomach and progressively involves the body and fundus (multifocal)
  • Autoimmune atrophic gastritis involves the body and fundus with sparing of the antrum (Curr Gastroenterol Rep 2020;22:38)
Pathophysiology
  • Environmental atrophic gastritis is caused by direct damage to gastric mucosa by environmental factors: predominantly H. pylori infection and possibly dietary factors, such as high salt intake (Cancer Epidemiol Biomarkers Prev 2017;26:1133)
  • Autoimmune atrophic gastritis is caused by the immune mediated destruction of parietal cells (Nat Rev Dis Primers 2020;6:56)
    • CD4+ T cells that are reactive against the H+ / K+ ATPase on parietal cells are the major driver
    • These helper T cells stimulate B cells to produce antiparietal cell and anti-intrinsic factor antibodies and activate cytotoxic T cells to damage parietal cells
    • Loss of parietal cells results in reduced HCl production and compensatory G cell hyperplasia in the antrum resulting in overproduction of gastrin
    • Increased gastrin stimulates hyperplasia of enterochromaffin-like cells (ECL cells), increasing the risk for neuroendocrine tumors
    • Damaged gastric epithelium undergoes metaplastic changes, increasing the risk of gastric adenocarcinoma
Etiology
  • Environmental atrophic gastritis is directly caused by damage from environmental factors: predominantly H. pylori infection and possibly dietary factors (Gastroenterology 2017;153:420, Cancer Epidemiol Biomarkers Prev 2017;26:1133)
  • Autoimmune atrophic gastritis thought to arise by 1 of 2 potential mechanisms (Autoimmun Rev 2019;18:215)
    • Primary autoimmune disease involving antibodies against parietal cells and intrinsic factor
    • Secondary autoimmune process via molecular mimicry of the beta subunit of H. pylori urease and the beta subunit of ATPase on parietal cells
Clinical features
  • May be asymptomatic
  • ~57% of patients report one or more gastrointestinal symptoms, including heartburn, regurgitation, dyspepsia and early satiety (Acta Biomed 2018;89:88)
  • Iron deficiency is an early finding secondary to decreased HCl production by parietal cells
  • Pernicious anemia (B12 deficiency) secondary to loss of intrinsic factor production by parietal cells
    • Prevalence of 37 - 69% of patients with autoimmune atrophic gastritis
    • > 50% of patients with B12 deficiency were positive for H. pylori infection (World J Gastroenterol 2017;23:563)
Diagnosis
  • Requires histopathologic confirmation; American Gastroenterological Association Institute guidelines recommend the updated Sydney protocol (5 gastric biopsies placed in separately labeled jars)
    • 2 from antrum (lesser and greater curvature) within 2 - 3 cm from the pylorus
    • 2 from body (lesser and greater curvature)
    • 1 from the incisura angularis
  • Correlation with clinical, serologic and histopathologic findings is recommended (Gastroenterology 2021;161:1325)
Laboratory
  • Iron deficiency: microcytic anemia
  • B12 deficiency: macrocytic anemia, elevated methylmalonic acid (Gastroenterology 2021;161:1325)
  • Elevated serum gastrin secondary to achlorhydria due to loss of parietal cells (Gastrointest Endosc 2013;77:284)
  • Decreased serum pepsinogen I (produced by chief cells) and decreased pepsinogen I/pepsinogen II ratio (Gastroenterology 2021;161:1325)
    • Pepsinogen II is produced by pyloric glands and Brunner glands
  • Antiparietal cell and anti-intrinsic factor antibodies are commonly seen in autoimmune atrophic gastritis (combined sensitivity ~73% in one study) (Autoimmun Rev 2014;13:459)
Radiology description
  • Best evaluated using barium Xray fluoroscopy
  • Patchy and mild involvement is not typically recognizable
  • Severe atrophy shows a diffusely narrowed stomach that has a smooth contour with loss of rugal folds (Radiology 2008;246:33)
Radiology images

Images hosted on other servers:
Atrophic gastritis barium Xray

Barium Xray

Prognostic factors
  • Progression of atrophic gastritis to malignancy depends on severity and extent of atrophy and intestinal metaplasia
  • Risk of progression to gastric adenocarcinoma ranges from 0.1% to 0.3% per year
  • Risk of neuroendocrine tumor development ranges from 0.4% to 0.7% per year (Gastroenterology 2021;161:1325)
Case reports
Treatment
  • All patients with atrophic gastritis should be tested for H. pylori and treated, if positive (Gastroenterology 2021;161:1325)
  • No established treatment for autoimmune type atrophic gastritis; immunosuppression for symptomatic relief has shown some success in a case report (ACG Case Rep J 2020;7:e00496)
  • Patients should be evaluated for anemia and B12 / iron deficiencies and supplemented if present (Gastroenterology 2021;161:1325)
  • No consensus on the interval for endoscopic surveillance, American Gastroenterological Association Institute guidelines recommends every 3 years in advanced cases (Gastroenterology 2021;161:1325)
Clinical images

Contributed by Kirbylee Nelson, M.D.
Prominent submucosal vessels

Prominent submucosal vessels

Loss of rugal folds

Loss of rugal folds

Gross description
  • Early atrophic gastritis may appear normal endoscopically
  • Typical endoscopic appearance of late atrophic gastritis shows loss of gastric rugal folds and prominent submucosal vessels secondary to the thinning of the overlying mucosa
  • Intestinal metaplasia appears nodular and may display the light blue crest sign: blue-white lines seen on the crests of the epithelial surface (Gastroenterology 2021;161:1325)
Microscopic (histologic) description
  • Atrophy (loss of native glands) of gastric mucosa with lamina propria fibrosis, with or without metaplastic epithelial changes (Dig Liver Dis 2021;53:1237)
  • Environmental atrophic gastritis (Arch Pathol Lab Med 2019;143:1327)
    • Early chronic H. pylori infections typically contain neutrophils and show a diffuse, superficial infiltrate of lymphocytes and plasma cells in the lamina propria of antral mucosa
    • Longstanding H. pylori infections show atrophy and pseudopyloric (mucinous) and intestinal metaplasia
    • Oxyntic gland destruction is typically less prominent than autoimmune atrophic gastritis
    • Enterochromaffin-like (ECL) cell hyperplasia is less commonly seen than autoimmune type atrophic gastritis
  • Autoimmune atrophic gastritis features differ between early, florid and end phases (Arch Pathol Lab Med 2019;143:1327)
    • Early phase
      • Mixed chronic inflammatory infiltrate in the basal lamina propria of oxyntic mucosa (bottom heavy)
      • Inflammatory cells can include lymphocytes, plasma cells, eosinophils and mast cells
      • Lymphocytes may infiltrate glands
      • Occasional apoptotic bodies may be seen
      • Pseudopyloric (mucinous), pancreatic acinar or (less often) intestinal metaplasia may be seen
      • Hypertrophic parietal cells may form polypoid nodules (oxyntic gland pseudopolyps)
    • Florid phase
      • Marked atrophy of oxyntic mucosa
      • Diffuse lymphoplasmacytic inflammation
      • Prominent intestinal metaplasia
      • G cell hyperplasia present in the antrum
    • End phase
      • Near complete loss of oxyntic glands and fibrosis of the lamina propria
      • Decreased inflammation
      • ECL cell hyperplasia (micronodular / nodular or linear): 5 or more cells forming a nodule surrounded by a basement membrane and measuring < 0.5 mm (micronodular / nodular) / 5 or more cells lining the base of pits (linear)
Microscopic (histologic) images

Contributed by Jen Rytych, M.D. and Joshua A. Hanson, M.D.
Atrophic mucosa

Atrophic mucosa

Intestinal metaplasia

Intestinal metaplasia

ECL cell hyperplasia ECL cell hyperplasia

ECL cell hyperplasia

Atrophic mucosa and inflammation

Atrophic mucosa and inflammation


Pseudopyloric metaplasia

Pseudopyloric metaplasia

Intestinal metaplasia

Intestinal metaplasia

Positive H. pylori stain

Positive H. pylori stain

Positive gastrin stain

Positive gastrin stain

Atrophic gastric body


Gastrin stain, body

Chromogranin stain, body

Antrum in autoimmune gastritis

Gastrin stain, antrum

Positive stains
Negative stains
  • Negative gastrin IHC in antralized (loss of oxyntic glands) gastric body (no G cells present confirms biopsy is from body or fundus)
  • Negative H. pylori IHC excludes atrophic H. pylori gastritis and increases the likelihood of autoimmune atrophic gastritis (Arch Pathol Lab Med 2019;143:1327)
Videos

Histology of atrophic gastritis

Histology of autoimmune atrophic gastritis

Sample pathology report
  • Gastric polyps, biopsies:
    • Chronic atrophic gastritis, enterochromaffin-like (ECL) cell hyperplasia and intestinal metaplasia (see comment)
    • Helicobacter pylori immunostain is negative
    • Comment: Sample of gastric body type mucosa is confirmed by gastrin negative immunostaining. Synaptophysin and chromogranin A highlight linear and micronodular ECL hyperplasia. Combined morphologic and immunohistochemical findings suggest atrophic gastritis, autoimmune type and confirmatory testing (such as antiparietal or anti-intrinsic factor serologies and serum gastrin and B12 levels) may be considered.
    • No evidence of dysplasia or infiltrative process identified in the submitted specimen.
Differential diagnosis
Board review style question #1

A 60 year old man with abdominal pain undergoes an upper endoscopy and biopsies from the body of the stomach are shown. What cell type is lost that leads to a decrease in acid production and what micronutrient is most commonly deficient as a direct result of this in these patients?

  1. Chief cells; calcium
  2. Chief cells; iron
  3. G cells; calcium
  4. Parietal cells; calcium
  5. Parietal cells; iron
Board review style answer #1
E. Parietal cells; iron. Parietal cells in the stomach not only secrete intrinsic factor, which is important for the absorption of vitamin B12, but also produce hydrochloric acid, which is necessary for the conversion of ferric (Fe3+) iron to absorbable ferrous (Fe2+) iron. Iron deficiency is a common early finding in atrophic gastritis. Answers A, C and D are incorrect because by comparison, calcium absorption requires vitamin D and is inhibited by high levels of phytic acids, sodium and caffeine. Celiac disease is associated with vitamin D deficiency and therefore calcium deficiency. Answer C is incorrect because G cells secrete gastrin into systemic circulation, which stimulates the proliferation of parietal cells and ECL cells and is not directly involved in micronutrient absorption. Answers A and B are incorrect because chief cells secrete digestive enzymes, such as pepsinogen and lipase, that break down macronutrients, such as proteins and lipid; they are not directly involved in the absorption of micronutrients.

Comment Here

Reference: Atrophic gastritis
Board review style question #2

A 55 year old woman presents with vague upper abdominal discomfort, bloating and occasional nausea. Upper endoscopy reveals a thinning of the gastric mucosa with areas of erythema and multiple patchy white plaques scattered throughout the mucosal surface. Multiple biopsies are taken. Which of the following histologic findings would be most consistent with a diagnosis of atrophic gastritis?

  1. Antral predominant acute or chronic inflammation in a top heavy, band-like distribution
  2. Gastric mucosa with dilated capillaries, intravascular fibrin thrombi and reactive foveolar changes
  3. Increased gastric intraepithelial lymphocytes and expansion of the lamina propria by lymphocytes and plasma cells
  4. Loss of parietal cells and replacement of gastric glands by intestinal type epithelium
  5. Tortuosity and hyperplasia of foveolar epithelium with loss of mucin
Board review style answer #2
D. Loss of parietal cells and replacement of gastric glands by intestinal type epithelium. These are the defining histologic features of atrophic gastritis. Answer C is incorrect because it is the common presentation for lymphocytic colitis. Answer B is incorrect because it lists the typical histologic features for gastric antral vascular ectasia. Answer E is incorrect because it describes the typical histologic features of reactive gastropathy. Answer A is incorrect because it is the common presentation for a typical H. pylori gastritis.

Comment Here

Reference: Atrophic gastritis

Bezoars
Table of Contents
Definition / general
Definition / general
  • Foreign bodies composed of hair or vegetable matter
  • Motility abnormalities and obstructions predisopose to bezoar formation, may complicate diabetes mellitus, myotonic muscular dystrophy, chronic drug therapy (opiates, neuromuscular blockers)

  • Phytobezoars:
    • Plant material concretions
    • Usually caused by ingestion of unripened persimmons
    • Stomach acid causes polymerization of tannin monomers in fruit, causing a tannin-cellulose-hemicellulose-protein complex
    • Dissolves with cellulase (Arch Pathol Lab Med 1980;104:159)

  • Trichobezoars:
    • Also known as hairballs
    • Ingested hair coated with decaying food
    • Associated with partial gastrectomy, partial outlet obstruction, lack of teeth, vagotomy, trichophagia

Carcinoma-general
Definition / general
  • More than 90% of gastric carcinomas are adenocarcinomas (Ann Oncol 2016;27:v38)
  • Gastric cancer is a heterogeneous disease with different phenotypes, genotypes and clinical progress, including sensitivity to treatments and prognoses
Essential features
ICD coding
  • ICD-O: 8140/3 - Adenocarcinoma, NOS
  • ICD-O: 8145/3 - Adenocarcinoma, diffuse type
Epidemiology
Sites
  • Most frequent location of gastric adenocarcinoma is the incisura angularis
  • Due to its epidemiological, etiological and clinical differences, cardia and noncardia (distal) gastric cancer are distinguished (Prz Gastroenterol 2019;14:26)
    • Cardia gastric cancer arises in the region adjoining the gastroesophageal junction and shares epidemiological characteristics with esophageal adenocarcinoma
    • Noncardia gastric cancer is more common and arises in the lower portion of the stomach
    • More than 90% of cases of the noncardia subtype are associated with H. pylori infection
Etiology
  • Gastric cancer demonstrates familial aggregation in around 10% of cases (Ann Oncol 2016;27:v38)
  • 1 - 3% of all gastric cancers are associated with inherited genetic predisposition disorders, such as familial adenomatous polyposis (APC mutation), hereditary diffuse gastric cancer (CDH1 mutations / loss of heterozygosity) and gastric adenocarcinoma and proximal polyposis of the stomach (GAPPS) (Prz Gastroenterol 2019;14:26)
  • Most sporadic gastric cancers are now considered to be inflammation driven and their etiology is characteristically environmental, usually related to H. pylori infection (Histopathology 2020;76:182)
  • Chronic H. pylori infection is recognized as the main cause of noncardia (distal) gastric cancer (odds ratio 5.9 within 10 years of infection) (Int J Cancer 2015;136:487)
  • Patients with gastric intestinal metaplasia (GIM) may have a greater than tenfold increased risk of gastric cancer than the general population
  • Gastric dysplasia is linked to an increased risk of adenocarcinoma (6% of patients with high grade dysplasia progress to cancer within 5 years) and should alert to the possibility of synchronous gastric cancer (Gut 2019;68:1545)
  • 5 - 10% of gastric carcinomas are associated with Epstein-Barr virus (EBV) infection, especially in tumors located in the cardia and gastric pouch (ESMO Open 2019;4:e000470, Appl Immunohistochem Mol Morphol 2017;25:12)
  • Smoking is associated with an increased risk of both gastric cardia (HR, 2.86; 95% CI, 1.73e4.70) and noncardia (HR, 2.04; 95% CI, 1.32e3.16) cancers (Best Pract Res Clin Gastroenterol 2017;31:509)
  • Heavy alcohol drinking has been pointed to as a risk factor for gastric cancer; however, the available information on the association between alcohol consumption and risk of gastric cancer is contradictory (Int J Cancer 2017;141:1950, Best Pract Res Clin Gastroenterol 2017;31:509)
  • Other proposed risk factors are diet (salt, preserved meats, coffee), low socioeconomic status, obesity (body mass index over 25 kg/m is associated with 1.13 odds ratio of developing cancer), pernicious anemia (risk of gastric cancer 6.9%), previous gastric surgery, race / ethnicity (higher risk in Asians and African Americans) and blood group A, among others (Prz Gastroenterol 2019;14:26)
Clinical features
  • Symptoms of gastric cancer are often nonspecific, frequently leading to diagnosis at an advanced stage (> 70%) (Tumour Biol 2017;39:1010428317714626)
  • 35% of gastric cancer patients in the U.S. have metastasis at diagnosis (clinical stage 4) (Clin Gastroenterol Hepatol 2020;18:534)
  • It is not uncommon for patients to have undergone several months of therapy for peptic ulcer disease before gastric cancer diagnosis
  • Most common symptoms are abdominal pain (50 - 65%) and weight loss (40%) (Kufe: Holland-Frei Cancer Medicine, 6th edition, 2003)
  • Although anemia is a frequent finding among patients with gastric cancer, upper gastrointestinal bleeding is less common and occurs in 16 - 17% of patients (Kufe: Holland-Frei Cancer Medicine, 6th edition, 2003)
  • Endoscopically, according to the Paris system, early gastric cancer is classified as follows (Gastrointest Endosc 2003;58:S3)
    • 0-I: polypoid - pedunculated (0-Ip) or sessile (0-Is)
    • 0-II: nonpolypoid - superficial elevated (0-IIa), flat (0-IIb) or superficial shallow, depressed (0-IIc)
    • 0-III: nonpolypoid - excavated (ulcerated)
  • Paris type 0-I and 0-III lesions have often submucosal infiltration and are not an ideal indication for endoscopic treatment (Endoscopy 2015;47:829)
Diagnosis
  • Diagnosis is routinely made on endoscopic biopsy
  • In the case of a malignant gastric ulcer, at least 7 biopsies of the heaped up edges of the ulcer and base should be performed (Gastrointest Endosc 2015;82:1)
  • Obtaining representative biopsies from cases with linitis plastica growth may be difficult because this condition is associated with infiltration of the submucosa or muscularis propria of the stomach, reducing the yield of mucosal biopsies (Gastrointest Endosc 2015;82:1)
  • In the absence of metastatic disease, endoscopic ultrasound (EUS) with or without FNA is indicated for local regional staging: tumor depth (pT1 - 2, pT3 - 4) and regional lymph node assessment (pN) (Gastrointest Endosc 2015;82:1)
  • Laparoscopy with or without peritoneal washings for malignant cells is recommended in all clinical stage IB - III gastric cancers which are considered potentially resectable, to exclude radiologically occult metastatic disease (Ann Oncol 2016;27:v38)
Radiology images

Images hosted on other servers:
Gastric tumor infiltrating the diaphragm

Gastric tumor infiltrating the diaphragm

Prognostic factors
Case reports
Treatment
  • Early gastric cancer can be treated through endoscopic mucosal resection (EMR) or endoscopic submucosal dissection (ESD) if they are confined to the mucosa, well differentiated, ≤ 2 cm and nonulcerated (Gastrointest Endosc 2015;82:1, Ann Oncol 2016;27:v38)
  • Early gastric cancer Paris 0-IIa, 0-IIb and 0-IIc show a low probability of advanced histology; therefore, EMR is acceptable for treatment in these cases (Endoscopy 2015;47:829)
  • For stage pT2 tumors and beyond, with no evidence of distant metastasis (stage IB - III), radical gastrectomy is indicated (Ann Oncol 2016;27:v38)
  • Perioperative (pre and postoperative) chemotherapy with a platinum / fluoropyrimidine combination is recommended for patients with resectable gastric cancer (Ann Oncol 2016;27:v38)
  • Doublet or triplet platinum / fluoropyrimidine combinations are recommended for fit patients with advanced gastric cancer (Ann Oncol 2016;27:v38)
  • Anti-HER2 drugs are currently added to the first line platinum based chemotherapy for the treatment of advanced gastric cancer cases showing HER2 amplification (Ann Oncol 2019;30:1254, Ann Oncol 2016;27:v38)
  • Immune checkpoint blockade inhibitors are recently approved for gastric cancer treatment
Clinical images

Contributed by Carolina Martinez Ciarpaglini, M.D., Ph.D.
Hemorrhagic gastric mass

Hemorrhagic gastric mass



Images hosted on other servers:
Ulcerated gastric mass Ulcerated gastric mass

Ulcerated gastric mass

Early gastric cancer

Early gastric cancer

Gross description
  • Typically, gastric cancer presents as a mass lesion; in other cases, a nonhealing gastric ulcer may be the presenting form (Gastrointest Endosc 2015;82:1)
  • Diffuse gastric cancer is characterized by an infiltrative growth pattern knows as linitis plastica
    • In these cases, neoplastic cells permeate deeply through the gastric wall, evoking an intense desmoplastic reaction causing diffuse rugal flattening and a rigid thickened wall
  • The pattern of linitis plastica is highly correlated with the presence of signet ring cells (J Gastrointest Surg 2020;24:1018)
  • The term linitis plastica should only be used for the description of the macroscopic characteristics of the tumor (Gastric Cancer 2019;22:1)
Gross images

Contributed by Clara Alfaro, M.D., Ph.D. and Carolina Martinez Ciarpaglini, M.D., Ph.D.
Mass in the gastric body

Mass in the gastric body

Linitis plastica

Linitis plastica

Serosal retraction

Serosal retraction

Giant distal gastric mass

Giant distal gastric mass

Microscopic (histologic) description
  • The most common and worldwide histological classifications for gastric cancer are the Lauren and the WHO classifications
  • Lauren classification (Acta Pathol Microbiol Scand 1965;64:31)
    • Diffuse type:
      • Lack of or poor cohesion between the neoplastic cells
      • Composed of scattered, small clusters or rows of cells with little or no gland formation
      • Neoplastic cells usually show an atypical morphology with irregular nuclear contours and variable amounts of eosinophilic cytoplasm
      • In some cases, there is a variable component of cells showing the so called signet ring morphology
      • Signet ring cells show ample cytoplasmic mucin, which appears optically clear on H&E staining and an eccentrically placed nucleus (Gastric Cancer 2019;22:1)
    • Intestinal type:
      • Composed of tubular or glandular structures similar to intestinal adenocarcinoma
      • Neoplastic cells usually contain apical mucin vacuoles
      • Unlike diffuse gastric cancer, intestinal tumors grow along broad cohesive fronts to form an exophytic mass
    • Mixed
    • Undifferentiated
  • WHO classification of gastric carcinomas comprises a more detailed classification based on morphological and in some cases immune expression profile of tumors, as listed here
  • Main similarities and differences between the 2 classifications are
    • WHO classification separates tubular gastric carcinoma into 3 categories: well, moderately and poorly differentiated, based on tubular formation
    • Lauren intestinal subtype corresponds with the papillary and the well and moderately differentiated tubular categories of the WHO classification
    • Lauren diffuse subtype corresponds with the poorly cohesive category of the WHO classification (Gastric Cancer 2019;22:1)
    • WHO classification separates poorly cohesive carcinoma into 2 categories (Gastric Cancer 2019;22:1):
      • Poorly cohesive, signet ring cell phenotype: composed only or predominately (more than 90%) of signet ring cells
      • Poorly cohesive, not otherwise specified (NOS): all other cases that do not display signet ring cell morphology (Gastric Cancer 2019;22:1)
    • Both classifications recognize a mixed subtype, usually composed of cases with intestinal / tubular or papillary and diffuse / poorly cohesive components
      • There is currently no cutoff concerning the percentage of each component for a tumor to be classified as mixed adenocarcinoma (Gastric Cancer 2019;22:1)
    • Most of the undifferentiated cases in the Lauren classification comprise cases classified as poorly differentiated tubular adenocarcinomas according to the WHO classification
    • WHO classification includes a series of less frequent variants, such as papillary, hepatoid, lymphoid stroma rich, among others, which are not defined in the Lauren classification (Gastric Cancer 2019;22:1)
  • Modified Lauren classification established 3 scenarios based not only on the histological appearance of the lesion but also on tumor location and association with pre-existing atrophy in the surrounding mucosa (Clin Cancer Res 2011;17:2693)
    • Proximal nondiffuse gastric cancer:
      • Comprises tumors located in the gastric cardia or gastroesophageal junction
      • Includes intestinal tumors with any grade of differentiation or mixed tumors with a minor component of diffuse carcinoma
      • Surrounding mucosa usually shows evidence of glandular dysplasia in the setting of chronic inflammation without atrophy
    • Diffuse gastric cancer:
      • Includes tumors composed entirely of diffuse morphology located anywhere in the stomach
      • In these cases, the remaining nontumoral mucosa shows no apparent gastritis or atrophy
    • Distal nondiffuse gastric cancer:
      • In these cases, the bulk of the tumor is usually located in the distal stomach and may extend to the mid body or the pylorus
      • This category includes intestinal tumors with any grade of differentiation or mixed tumors with a minor component of diffuse carcinoma
Microscopic (histologic) images

Contributed by Carolina Martinez Ciarpaglini, M.D., Ph.D.
Gastric adenocarcinoma intestinal subtype

Gastric adenocarcinoma intestinal subtype

Diffuse adenocarcinoma

Diffuse adenocarcinoma

Poorly cohesive adenocarcinoma with signet ring cell phenotype Poorly cohesive adenocarcinoma with signet ring cell phenotype

Poorly cohesive adenocarcinoma with signet ring cell phenotype

Virtual slides

Images hosted on other servers:
Diffuse gastric carcinoma

Diffuse gastric carcinoma

Negative stains
Molecular / cytogenetics description
Sample pathology report
  • Gastric mass, antrum, endoscopic biopsy:
    • Diffuse gastric carcinoma (poorly cohesive, NOS, per WHO classification)
  • Gastric mass, antrum, endoscopic biopsy:
    • Diffuse gastric carcinoma (poorly cohesive, signet ring cell type, per WHO classification)
  • Gastric mass, body, endoscopic biopsy:
    • Intestinal type gastric carcinoma (tubular low grade, per WHO classification)
Differential diagnosis
Board review style question #1

Select the correct histopathological diagnosis for this type of gastric tumor

  1. Intestinal gastric carcinoma (Lauren classification)
  2. Poorly cohesive carcinoma, NOS (WHO classification)
  3. Poorly cohesive carcinoma with signet ring cell phenotype (WHO classification)
  4. Tubular carcinoma (WHO classification)
Board review style answer #1
B. Poorly cohesive carcinoma, NOS (WHO classification)

Comment Here

Reference: Carcinoma-general
Board review style question #2
Which of the following is true about signet ring cells in gastric cancer?

  1. Definition of signet ring cells relies purely on morphological criteria
  2. E-cadherin staining is useful to recognize them
  3. If more than 30% are present, the tumor is called poorly cohesive signet ring cell type according to the WHO definition
  4. Pattern of linitis plastica is not correlated with the presence of signet ring cells
Board review style answer #2
A. Definition of signet ring cells relies purely on morphological criteria: ample cytoplasmic mucin which appears optically clear on H&E staining and an eccentrically placed nucleus.

Comment Here

Reference: Carcinoma-general

Chronic gastritis
Definition / general
  • Chronic mucosal inflammatory changes leading to mucosal atrophy and epithelial metaplasia, usually without erosions
  • Most cases are type B or non-autoimmune gastritis
  • Associated with chronic Helicobacter pylori infection (Am J Surg Pathol 2006;30:242), toxins (alcohol, tobacco), reflux of bilious duodenal secretions (post-antrectomy or other), obstruction (bezoars, atony), radiation
  • Incidence increases with age; in Europe / Japan, affects 50% at age 60+
  • Histology does not correlate well with symptoms

  • Superficial chronic gastritis:
    • Inflammation confined largely to mucosa occupied by gastric pits
Microscopic (histologic) description
  • Plasma cells, lymphocytes, occasional lymphoid follicles
  • May have eosinophils and neutrophils also
  • May have reduced cytoplasmic mucin, reactive epithelial changes (nuclear and nucleolar enlargement)
  • May have subnuclear vacuolation in antral glands or pits (PAS negative), probably represents degenerative response to cell injury

  • Intestinal metaplasia: affects antral and body / fundic mucosa, with partial replacement by metaplastic goblet cells of intestinal morphology, absorptive cells and Paneth cells; extensive if involves 25% of biopsy tissue
  • Immunophenotypically distinct from intestinal metaplasia of GE junction or Barrett's esophagus (Am J Surg Pathol 2001;25:87)
  • Complete intestinal metaplasia: mucosal pattern resembles small bowel epithelium with goblet and absorptive cells, villi and crypts; sialomucins predominate
  • Incomplete intestinal metaplasia: no absorptive cells, columnar cells resemble gastric foveolar cells; neutral mucins and sulfomucins are present

CMV
Definition / general
  • Generally associated with bone marrow transplants, AIDS, other immunosuppression
  • In pediatric population may be associated with hypertrophic gastropathy
  • Gastric involvement usually part of systemic disease
  • May cause perforation or fistulas
Microscopic (histologic) description
  • Eosinophilic intranuclear inclusions, variable granular purple cytoplasmic inclusions
  • Severe cases have ulceration, hemorrhage and perforation
Microscopic (histologic) images

Contributed by Andrey Bychkov, M.D., Ph.D. and Jijgee Munkhdelger, M.D., Ph.D.

CMV gastritis ulcer

CMV gastritis IHC

CMV infected cells

Classic cytomegalic cell



Images hosted on other servers:

Duodenal biopsy


Collagenous gastritis
Definition / general
  • Poorly understood disease manifesting as thickened subepithelial collagen in the stomach
Essential features
  • An extremely rare disease characterized by the presence of a band of increased subepithelial collagen in the stomach, with associated chronic inflammation in the lamina propria
  • Often subtyped into pediatric and adult versions
  • No standard therapy
ICD coding
  • ICD-10: K52.89 - Other specified noninfective gastroenteritis and colitis
Epidemiology
Sites
  • Pediatric patients possibly more likely to have body / fundus predominant disease versus antrum in adults (Mod Pathol 2015;28:533)
Pathophysiology
  • Poorly understood etiology / pathophysiology
    Clinical features
    • Associated with celiac disease, collagenous colitis, collagenous sprue and other autoimmune disease (Arch Pathol Lab Med 2001;125:1579)
    • Most common clinical symptoms include abdominal pain, anemia, diarrhea, nausea / vomiting, GI bleeding and weight loss
    • Through limited case reports, the disease has been phenotyped into adult and pediatric types but many patients may not fit either category (Am J Surg Pathol 2001;25:1174)
      • Pediatric type classically presents with upper GI symptoms including abdominal pain and anemia, which are a result of the disease process
      • Adult type is characterized by accompanying collagenous colitis, related to underlying autoimmune processes or celiac disease
      Diagnosis
      • Established by gastric biopsy
        Case reports
        Treatment
        • No standard therapy has been defined
        Clinical images

        Contributed by Matthew Morrow, M.D.
        Missing Image

        Gastric nodularity

        Missing Image

        Gastric mucosa with mild nodularity



        Images hosted on other servers:
        Missing Image

        Nodular gastric mucosa

        Gross description
        • Nodularity of the gastric corpus is the characteristic endoscopic finding but is not seen in all cases (World J Gastrointest Endosc 2015;7:265)
          • Depressed mucosa between the nodules shows atrophy and deposition of subepithelial collagen, whereas the nodular lesions themselves show unaffected mucosa (Dig Dis Sci 2007;52:995)
        • Other mucosal findings include erythema, erosions and exudate
        • Occasionally there are no findings (akin to collagenous colitis)
        Microscopic (histologic) description
        • Patchy increased subepithelial collagen, having a thickness ≥ 10 µm
          • Maximum thickness on average ranges from 15 to 115 μm, mean 55.1 μm (Mod Pathol 2015;28:533)
          • Often associated with the entrapment of red blood cells, inflammatory cells and superficial capillaries (Am J Surg Pathol 2001;25:1174)
          • Associated with the denudation or separation of the surface gastric epithelium
        • Increased chronic inflammatory infiltrate of the lamina propria including increased eosinophils and many times neutrophils
        • Increased intraepithelial lymphocytes, but fewer than in lymphocytic gastritis
        • Histologic features may persist for years (Mod Pathol 2015;28:533)
        Microscopic (histologic) images

        Contributed by Matthew Morrow, M.D.
        Missing Image Missing Image

        Increased subepithelial collagen with lamina propria chronic inflammation

        Missing Image

        Denudation of surface gastric epithelium

        Missing Image

        Thickened subepithelial collagen

        Missing Image

        Collagenous gastritis 100x

        Missing Image

        Abundant eosinophils 40x



        Images hosted on other servers:
        Missing Image Missing Image

        Thickened subepithelial collagen

        Missing Image

        Lamina propria inflammation

        Positive stains
        • Masson trichrome stain highlights the thickened subepithelial layer of collagen
        Negative stains
        Sample pathology report
        • Stomach, antrum, biopsy:
          • Collagenous gastritis (see comment)
          • Comment: A trichrome special stain highlights a thickened basement membrane.
        Differential diagnosis
        Additional references
        Board review style question #1

          A 10 year old boy presents with chronic abdominal pain and anemia. The following is a biopsy from the stomach. Which of the following statements is true?

        1. Immunosuppression is standard therapy
        2. The absence of both HLA-DQ2 and HLA-DQ8 haplotypes essentially excludes the diagnosis
        3. The histologic features may persist for years
        4. The pediatric type of the disease is classically characterized by accompanying collagenous colitis
        Board review style answer #1
        C. The histologic features may persist for years. In one study, of the patients who had follow up biopsies, 75% had persistent histologic features, sometimes lasting up to 10 years (Mod Pathol 2015;28:533).

        Comment Here

        Reference: Collagenous gastritis
        Board review style question #2
          Which of the following statements about collagenous gastritis is true?

        1. An increase in collagen is seen in the deep lamina propria and bowel wall
        2. If nodular gastric mucosa is seen, for diagnosis of this entity it is best to biopsy at the tips of the nodules
        3. It can be associated with celiac disease
        4. It is mainly a disease of the elderly
        Board review style answer #2
        C. Collagenous gastritis can be associated with celiac disease

        Comment Here

        Reference: Collagenous gastritis

        Cowden
        Definition / general
        • Also known as multiple hamartoma syndrome
        • Due to rare autosomal dominant germline mutation in PTEN on 10q23
        • Tricholemmomas on face, acral keratoses, oral papillomas, GI polyps that may occur in stomach, small intestine or colon (35 - 40%), increased incidence of breast and thyroid carcinomas, also endometrial and renal cell carcinomas
        Gross description
        • Small (1 - 2 mm), sessile polyps
        Microscopic (histologic) description
        • Excess lamina propria splayed and dissected into lobules by disorganized fascicles of muscularis mucosa running upward from base of mucosa
        Microscopic (histologic) images

        Image hosted on other server:

        Gastric and colonic polyps


        Cronkhite-Canada
        Definition / general
        • Very rare juvenile polyposis disorder with diffuse GI polyposis, alopecia, hyperpigmentation, dystrophic changes in fingernails and toenails
        • Nonhereditary, unknown etiology
        • Onset usually in middle age adults
        • Patients may have diarrhea, weight loss, abdominal pain, anorexia, weakness, hematochezia
        • Occasionally gastric adenocarcinoma, adenocarcinoma anywhere in GI tract (Am J Surg Pathol 1985;9:65)
        Case reports
        Microscopic (histologic) description
        Differential diagnosis

        Diffuse large B cell lymphoma
        Definition / general
        • High grade B-cell tumor (Am J Surg Pathol 2003;27:790)
        • May arise from transformed extranodal marginal zone (MALT) lymphoma
        • Usually > age 50 years
        • 5 year survival is 65% (related to stage)
        • Coexistence of MALT lymphoma and diffuse large B-cell lymphoma: do not report as "high grade MALT" (MALT is usually indolent); report instead as diffuse large B-cell lymphoma with coexisting MALT lymphoma
        Prognostic factors
        Case reports
        Gross description
        • Large polypoid or lobulated mass with superficial or deep ulceration; often in distal stomach, but sparing pylorus
        Microscopic (histologic) description
        • Infiltrative pattern of high grade, centroblast like cells; multinucleated forms may resemble Reed-Sternberg cells
        Positive stains
        Differential diagnosis
        • Undifferentiated carcinoma: continuity between tumor cells and epithelium, acinar pattern, muscularis mucosa destruction, positive for keratin and mucin

        Diffuse type
        Definition / general
        • Diffuse type gastric adenocarcinoma - carcinoma composed of poorly differentiated / signet ring cells, present as scattered individual cells or clusters
        • This type of carcinoma is associated with aggressive behavior (rapid invasion, chemoresistance, peritoneal metastasis) and poor prognosis
        Essential features
        • Diffuse type gastric carcinoma is an aggressive form of gastric cancer that varies from the intestinal type in many aspects
        • It can be seen in sporadic and familial settings
        • Mutations in the CDH1 gene are most commonly implicated and can be germline in familial settings
        • Diffuse type carcinoma is composed of poorly cohesive cells that can be either signet ring cell type or non signet ring cell (poorly differentiated) type
        • This an aggressive neoplasm characterized by early spread via lymphatic dissemination and by local extension into neighboring organs or as peritoneal carcinomatosis
        Terminology
        • Official WHO term is poorly cohesive carcinoma
        • Other terms: Lauren diffuse type, linitis plastica, signet ring cell gastric adenocarcinoma
        ICD coding
        • ICD-10: C16.9 - malignant neoplasm of stomach, unspecified
        Epidemiology
        • Incidence of diffuse type gastric adenocarcinoma is increasing worldwide but is slightly less than intestinal type (Eur J Cancer 2020;130:23)
        • Diffuse type is more uniformly distributed geographically but with an increasing incidence in the U.S. and in Europe (Oncol Lett 2019;18:674)
        • More common in younger patients: mean age 48 years
        • No sex predilection seen; M = F (Oncol Lett 2019;18:674)
        • While diffuse type gastric carcinomas can be seen in sporadic settings, a significant proportion of cases are seen in the context of hereditary diffuse gastric cancer (HDGC) syndrome
        • Patients with hereditary diffuse gastric cancer are also predisposed to lobular breast cancer
        Pathophysiology
        • Majority of gastric carcinomas of diffuse type may develop from the ECL cells secondary to long term hyperstimulation with gastrin (Int J Mol Sci 2018;19:4109)
        • Underlying genetic alterations maybe germline or biallelic somatic mutations
        • Mutations in CDH1 gene are most commonly implicated; pathogenic germline mutations in this gene are responsible for HDGC
        • E-cadherin is a calcium dependent adhesion molecule that interacts with beta catenin to maintain the structure of epithelium
        • When a patient with germline E-cadherin gene (CDH1) mutation acquires a second hit, whether by loss of heterozygosity or promoter methylation or somatic mutation, it results in development of gastric carcinomas of diffuse type at an early age (Int J Mol Sci 2018;19:4109)
        • CDH1 mutations seem to be the most frequent abnormality; other adhesion molecules could be involved in fewer cases, such as somatic mutations of beta catenin / APC genes or dysregulation of the Wnt / beta catenin pathway (World J Gastroenterol 2015;21:11428)
        • Other genes responsible for HDGC include CTNNA1, PALB2, BRCA2, etc., each described in a handful of cases
        Etiology
        • Diffuse type gastric carcinoma originates from the gastric mucosa and in sporadic cases is usually associated with gastritis (Oncol Lett 2016;11:2959)
        • HDGC may have minimal or no background gastritis and may have early precursor components (described below)
        Clinical features
        • Potential symptoms include indigestion, nausea or vomiting, dysphagia, postprandial fullness, loss of appetite, melena, hematemesis, weight loss (Front Surg 2017;4:42)
        • May cause pathologic peritoneal and pleural effusions, obstruction of the gastric outlet
        Diagnosis
        • Endoscopy shows flat or depressed lesions with an ill defined border (Gut Liver 2017;11:807)
        • In some cases, thickened folds may be a clue to the underlying abnormality
        • Very early lesions may only show some pale discoloration or even grossly normal mucosa
        • Chromoendoscopy increases the yield of detection of early lesions
        • Biopsy
        • Chest radiograph to evaluate for metastatic lesions
        • CT scan or MRI of the chest, abdomen and pelvis
        Laboratory
        Radiology description
        • Thickening of the gastric wall
        • Gastric outlet obstruction
        Radiology images

        Images hosted on other servers:

        Gastric wall thickening, incomplete distention

        Prognostic factors
        • Advanced stage diffuse type adenocarcinoma has poor prognosis
        • Diffuse type gastric carcinoma spreads early via lymphatic dissemination and by local extension into neighboring organs or as peritoneal carcinomatosis
        • Lymph node metastasis is an important prognostic factor
        • High CA19-9 is associated with poor survival (Eur J Med Res 2011;16:29)
        Case reports
        Treatment
        • In early gastric cancer, minimally invasive procedures such as wedge resection or advanced endoscopic techniques may constitute an equally radical treatment method (Prz Gastroenterol 2018;13:200)
        • Gastrectomy and lymphadenectomy for more advanced lesions
        • Chemotherapy including 5-fluorouracil (5-FU), platinum, taxanes, irinotecan and anthracycline; these drugs may be administered separately, using double or triple drug regimens in combination with epirubicin or docetaxel (Prz Gastroenterol 2018;13:200)
        • For patients with a germline mutation, a prophylactic gastrectomy is offered as the modality of choice
        Gross description
        • Very early stage lesions, typically in context of prophylactic gastrectomy, may not produce any visible abnormalities / lesions (Front Pharmacol 2018;9:1421)
        • More advanced lesions may present as thickening, ulceration or just rigid, leather bottle-like stomach (linitis plastica) due to extensive infiltration by the malignancy; may cause pyloric obstruction (Int J Mol Sci 2018;19:4109)
        • Gastric carcinoma of diffuse type typically invades the submucosa at an early phase and the tumor cells often spread in the upper layers of the stomach wall instead of growing as a tumor protruding into the lumen
        • Therefore, diffuse gastric carcinomas may be easily overseen at early phases even at endoscopy
        Gross images

        Contributed by Gagandeep Kaur, M.D. and Monika Vyas, M.D.

        Grossly no residual tumor

        Negative for gross lesion



        Images hosted on other servers:

        Marked thickening of gastric wall with loss of rugae

        Thickening of gastric wall

        Frozen section description
        • Intraoperative consultation is frequently sought for evaluation of margin status at the time of gastrectomy
        • Carcinoma can be present at the margin in the form of small foci and lack significant tissue reaction or inflammation, making frozen section diagnosis extremely challenging
        • Additionally, detached signet ring cells can be seen as an artifact in the gastric mucosa, making interpretation even more difficult (J Med Genet 2015;52:361)
        • Due to their diffuse morphology, the tumor cells may be indistinct and difficult to distinguish from inflammatory or stromal cells (Virchows Arch 2020 Mar 25 [Epub ahead of print])
        • Helpful features are the presence of cells with a single distinct cytoplasmic mucin vacuole, focal gland formation and perineural invasion
        • Other helpful features are irregular nuclear contours, large nuclear size with many nuclei > 4x the size of a small lymphocyte, disruption / obliteration of normal structures and the presence of mitotic figures
        Microscopic (histologic) description
        • Neoplastic cells that are isolated or arranged in small aggregates without well formed glands
        • Poorly cohesive carcinomas can be either signet ring cell type or non signet ring cell (poorly differentiated) type
        • Signet ring cell carcinoma (SRCC) is composed predominantly or exclusively of signet ring cells, which are characterized by a central, optically clear, globoid droplet of cytoplasmic mucin with an eccentrically placed nucleus
        • Signet ring cells may form a lace-like glandular or delicate microtrabecular pattern
        • Other cellular subtypes include tumors composed of neoplastic cells resembling histiocytes or lymphocytes
        • Others have deeply eosinophilic cytoplasm, bizarre pleomorphic nuclei
        • 2 types of precursor lesions are described in the context of HDGC (but not sporadic cases) (Front Pharmacol 2018;9:1421):
          • In situ SRCC: signet ring cells are confined within the epithelium of glands / foveolae without breaking the basement membrane
          • Pagetoid SRCC: signet ring cells spread below the preserved epithelium of glands / foveolae without breaking the basement membrane
        • Early lesions arise in the region of mucous neck cells of the gastric glands and are limited to the lamina propria; they may lack any tissue or inflammatory response
        • 3 cell types have been described in the carcinomas seen in patients with HDGC - small SRCC, large SRCC and poorly differentiated cells
        • Advanced SRCC in patients with HDGC resemble sporadic SRCC microscopically
        • There is marked desmoplasia, in particular when infiltrating into the submucosa or beyond
        Microscopic (histologic) images

        Contributed by Gagandeep Kaur, M.D. and Monika Vyas, M.D.

        Diffuse gastric carcinoma associated with chronic gastritis

        Signet ring cells

        Peritoneal involvement

        CK stain highlighting perineural invasion


        Early diffuse gastric carcinoma

        In situ carcinoma

        Pagetoid spread of signet ring cell carcinoma

        Cytology description
        • Single atypical cells or clusters of cells forming a vague gland (Front Pharmacol 2018;9:1421):
        • Malignant cells with eccentric nuclei
        • Intracytoplasmic mucin can be identified
        • Atypical naked nuclei
        Positive stains
        Negative stains
        • E-cadherin: complete absence of membranous staining
        • Aberrant staining patterns: dotted staining (due to staining of nonfunctional residual domains in proteins), cytoplasmic staining and reduced membranous staining (Int J Clin Exp Pathol 2017;10:8980)
        • TTF1, p53
        Molecular / cytogenetics description
        • Mutations in CDH1 gene leading to reduced E-cadherin expression (J Gastrointest Oncol 2012;3:251)
        • Activated ErbB2 / ErbB3 complex in signet ring cell carcinoma binds PI3K leading to phosphorylation of tyrosine residues and activation of downstream pathways including p38 MAP kinase (World J Gastroenterol 2015;21:11428)
        • Activation of p38 MAP kinase lead to loss of cell to cell contact by disruption of adherent junctions
        • MUC4 has been reported to increase activation of the ErbB2 / ErbB3 complex
        • MUC4 belongs to the family of mucins that are normally expressed in gastric mucosae (MUC1, MUC5AC, MUC6) or expressed de novo in gastric cancer (MUC2, MUC4)
        • Among the molecular subtypes of gastric carcinoma (based on the TGCA data), the genomically stable category is enriched in diffuse type phenotype associated with CDH1 mutations (Nat Rev Dis Primers 2017;3:17036)
        Sample pathology report
        • Stomach, total gastrectomy:
          • Poorly differentiated adenocarcinoma, diffuse type (tumor size), with invasion into the submucosa
          • Surgical margins are negative for adenocarcinoma
          • Five (0/5) benign reactive lymph nodes
          • See synoptic report and comment
        Differential diagnosis
        • Lymphoma with artifactual signet ring cells due to cytoplasmic shrinkage (Arch Pathol Lab Med 1997;121:623):
        • Metastases from breast (lobular carcinoma) (Arch Pathol Lab Med 2001;125:567):
          • Both have signet ring cell morphology and negative E-cadherin stain
          • GATA3 / ER positive in breast primary and negative in gastric carcinoma
        • Reactive epithelial atypia / pseudo signet ring cells associated with tissue injury or otherwise:
          • Histology of the background tissue may offer clues; detached cells floating on surface or in a space with inflammatory cells should be interpreted with caution, as they often represent pseudo signet ring cells
          • E-cadherin stain will show membranous positivity in reactive epithelial atypia and may exhibit aberrant expression in true signet ring cells
        • Xanthoma:
          • Foamy histiocytes that are CD68 positive and PAS negative
        Board review style question #1

        A 56 year old woman presents with nausea, vomiting and 4 months of weight loss. Endoscopy is performed and shows an erythematous area with ill defined borders. Biopsy is performed and shows poorly differentiated adenocarcinoma with signet ring type morphology. What is the most likely gene mutated in this malignancy?

        1. CDH1
        2. FAP
        3. HER2
        4. PTEN
        Board review style answer #1
        Board review style question #2
        A 30 year old woman is found to have a germline CDH1 mutation and undergoes a prophylactic gastrectomy. Which other malignancy is she at a higher risk of developing?

        1. Clear cell renal cell carcinoma
        2. Lobular carcinoma of breast
        3. Melanoma
        4. Osteosarcoma
        Board review style answer #2
        B. Lobular carcinoma of breast

        Comment Here

        Reference: Diffuse (poorly cohesive) type adenocarcinoma

        Doxycycline
        Definition / general
        • Doxycycline, an oral tetracycline antibiotic, has been associated with gastric mucosal injury with characteristic endoscopic and histopathologic features
        Essential features
        • Patients present with dysphagia, epigastric abdominal pain and retrosternal chest pain
        • Endoscopic finding: gastric mucosal erosions, nonbleeding ulcers, gastric mucosa with white plaque-like lesions
        • Histopathologic findings: superficial mucosal capillaries with eosinophilic degenerative changes and fibrin microthrombi (Am J Surg Pathol 2013;37:259)
        Terminology
        • Pill induced gastritis
        • Acute erosive gastritis with ulceration
        ICD coding
        • No official coding established
        Epidemiology
        • Patients with bacterial infections such as Lyme disease
        • Patients in need of prevention of malaria
        • Patients with severe acne or rosacea (Am J Surg Pathol 2017;41:374)
        Sites
        • Stomach (gastric body and fundus > gastric antrum)
        • Can also occur in esophagus
        Clinical features
        • Dysphagia, epigastric abdominal pain and retrosternal chest pain
        • The time frame for drug use ranges from 5 days to 4 years
        Diagnosis
        • Clinical presentation, medication history, endoscopic findings, histopathologic findings
        Prognostic factors
        Case reports
        Treatment
        • Cessation of the medication
        Gross description
        • Gastric mucosal erosions and nonbleeding ulcers
        • Gastric mucosa with white plaque-like lesions
        Microscopic (histologic) description
        • Superficial mucosal capillaries with eosinophilic degenerative changes (Am J Surg Pathol 2013; 37:259)
        • Background gastric mucosa with reactive changes in foveolar epithelium, superficial mucosal necrosis and neutrophilic infiltration within lamina propria
        Microscopic (histologic) images

        Contributed by Feng Yin, M.D., Ph.D.

        Doxycycline associated mucosal injury

        Eosinophilic capillary degenerative changes

        Positive stains
        Sample pathology report
        • Stomach, biopsy:
          • Gastric oxyntic mucosa with active inflammation, superficial mucosal necrosis and eosinophilic degeneration of superficial capillaries, features most compatible with doxycycline induced gastric mucosal injury
          • Negative for intestinal metaplasia or dysplasia
          • No morphologic evidence of Helicobacter pylori organisms
        Differential diagnosis
        Board review style question #1

        A 75 year old female with history of morbid obesity and rheumatoid arthritis presented with abdominal pain and hematemesis. She had periprosthetic infection and was on antibiotics. EGD shows a circumferential ulcer covered with white exudates below the gastroesophageal junction. A representative photomicrograph of the biopsy is shown. Which is the most likely diagnosis?

        1. Autoimmune atrophic gastritis
        2. Cytomegalovirus gastritis
        3. Doxycycline related gastric mucosal injury
        4. Gastric antral vascular ectasia
        5. Portal hypertensive gastropathy
        Board review style answer #1
        C. The biopsy reveals gastric mucosa with superficial capillary vessel wall eosinophilic degeneration, prominent lamina propria neutrophilic infiltration and mild reactive foveolar epithelial changes. Endoscopic findings include a circumferential ulcer covered with white exudates. These findings are most compatible with doxycycline associated gastric mucosal injury.

        Comment Here

        Reference: Doxycycline
        Board review style question #2
        Which of the following is true about doxycycline related gastric mucosal injury?

        1. Associated with granuloma formation
        2. Characteristic findings include perivascular edema and endotheliitis
        3. Gastric antrum is the most common site of injury
        4. Snakeskin mucosal pattern on endoscopy
        5. Superficial mucosal vascular degenerative changes
        Board review style answer #2
        E. Doxycycline induced gastritis usually causes damage to the gastric body or fundus, and the characteristic histopathologic findings include superficial mucosal necrosis, degenerative changes involving capillary vessel wall and fibrin microthrombi.

        Comment Here

        Reference: Doxycycline

        Dysplasia
        Definition / general
        • Gastric dysplasia is a precursor lesion to gastric adenocarcinoma; it can be flat, polypoid (adenoma) or arise on the surface of an existing nondysplastic polyp
        Essential features
        • Gastric dysplasia is a precursor lesion to gastric adenocarcinoma
        • Classified into low and high grade
        • Histologic subtypes include intestinal, foveolar, crypt, tubule neck and serrated type dysplasia
        • Isolated lesions may be treated with endoscopic submucosal resection or dissection
        Terminology
        • High grade dysplasia may be diagnosed as noninvasive carcinoma in Japan (Am J Surg Pathol 1999;23:511)
        • Alternate / historic name for intestinal dysplasia is adenomatous dysplasia
        • Alternate / historic names for foveolar dysplasia are type II dysplasia and hyperplastic dysplasia
        ICD coding
        • ICD-10: D00 - carcinoma in situ of oral cavity, esophagus and stomach
        Epidemiology
        • Male predominant
        • Higher incidence in Asia, Eastern Europe and South America
        • Risk factors include Helicobacter infection
        Sites
        • Stomach
        Pathophysiology
        • Most common cause of gastric dysplasia is Helicobacter pylori infection, which results in chronic active gastritis, atrophy and intestinal metaplasia
        Etiology
        • Helicobacter infection is the most common cause worldwide (Gastroenterology 1997;113:S56)
        • Smoking, radiation exposure and occupation in rubber manufacturing
        Clinical features
        • Predominantly asymptomatic, although larger lesions may cause bleeding or anemia
        • Can present as a flat, ulcerated or polypoid lesion on endoscopy
        Diagnosis
        • Diagnosed by upper endoscopy with biopsy
        Prognostic factors
        • High grade dysplasia is associated with a much higher rate of progression to adenocarcinoma (Gut 2003;52:1111)
        Case reports
        Treatment
        • Treated with endoscopic submucosal resection or endoscopic submucosal dissection
        Microscopic (histologic) description
        • Gastric dysplasia is divided into several subtypes
        • Intestinal and foveolar type are the most common
        • Crypt dysplasia and serrated dysplasia are more recently described but are recognized subtypes in the World Health Organization gastrointestinal book
        • Tubule neck or globoid dysplasia is the rare precursor lesion to poorly cohesive gastric carcinoma
        • Intestinal type dysplasia:
          • Similar morphology to colorectal adenomas
          • Tubular architecture
          • Cells have elongated nuclei with clumped chromatin, hyperchromasia and pseudostratification
          • Frequently admixed goblet cells and Paneth cells
          • Often arising in a background of intestinal metaplasia
          • High grade intestinal dysplasia shows loss of polarity, nuclear rounding, anisonucleosis, vesicular chromatin and increased N/C ratios
        • Foveolar type dysplasia:
          • Surface involvement with irregular branching
          • Cuboidal to columnar cells with hyperchromatic oval nuclei and pale cytoplasm
          • High grade foveolar dysplasia shows loss of polarity, increased N/C ratios, vesicular chromatin and prominent nucleoli
        • Tubule neck dysplasia or globoid dysplasia:
          • Rare precursor lesion to poorly cohesive (signet ring) gastric carcinoma
          • Increased cellularity in the gastric neck region
          • Enlarged clear cells with vacuolated cytoplasm and similar appearance to signet ring cells
        • Crypt dysplasia or pit dysplasia:
          • Marked cytologic atypia characterized by nuclear pleomorphism, increased N/C ratio, irregular nuclear contours and hyperchromasia that is limited to the gastric crypts / pits and does not reach the surface (Histopathology 2016;68:843)
        • Serrated dysplasia:
          • Rare polyp similar to the colorectal traditional serrated adenoma (Histol Histopathol 2013;28:453)
          • Characterized by stratified pencillate nuclei and abundant eosinophilic cytoplasm with slit-like serrations
        Microscopic (histologic) images

        Contributed by Kyra Berg, M.D.
        Elongated and hyperchromatic nuclei

        Elongated and hyperchromatic nuclei

        Elongated nuclei with visible nucleoli

        Elongated nuclei with visible nucleoli

        Loss of polarity

        Loss of polarity

        Increased cellularity

        Increased cellularity

        Positive stains
        Molecular / cytogenetics description
        Sample pathology report
        • Stomach, biopsy:
          • Low grade dysplasia, intestinal type (see comment)
          • Comment: Background of intestinal metaplasia. Negative for Helicobacter organisms by immunohistochemistry.
        Differential diagnosis
        Additional references
        Board review style question #1
        Which of the following is the most common risk factor for gastric dysplasia?

        1. Alcohol use
        2. Helicobacter infection
        3. Radiation exposure
        4. Smoking
        Board review style answer #1
        B. Helicobacter infection

        Comment Here

        Reference: Gastric dysplasia
        Board review style question #2

        What is the best diagnosis for the stomach biopsy shown above?

        1. High grade dysplasia
        2. Intramucosal adenocarcinoma
        3. Low grade dysplasia
        4. Radiation atypia
        Board review style answer #2
        A. High grade dysplasia

        Comment Here

        Reference: Gastric dysplasia

        Early gastric carcinoma
        Definition / general
        • Early gastric cancer (EGC) is defined as a cancer confined to the mucosa or submucosa, regardless of lymph node metastasis
        Essential features
        • Defined as a cancer confined to the mucosa or submucosa, regardless of lymph node metastasis
        • Tubular adenocarcinoma is the most common histologic type
        • EGC is preceded by a cascade of precancerous lesions
        • Diagnosis is made by endoscopy confirmed by biopsy
        • Lymph node metastasis is the most important prognostic factor
        Terminology
        • Mucosal or submucosal gastric carcinoma
        ICD coding
        • ICD-10: C16 - malignant neoplasm of stomach
        Epidemiology
        • In a large Chinese series of 4,380 consecutive gastric carcinomas (J Dig Dis 2018;19:586):
          • Frequency: 17%
          • Average age of patients: 62.2 years
          • M:F = 2.29:1
        • EGC is more commonly observed in China, Japan and Korea, due in large part to screening
        Sites
        • In the stomach (J Dig Dis 2018;19:586):
          • Cardia: 18.8%
          • Corpus / fundus: 13.4%
          • Antrum / angularis / pylorus: 67.8%
        Pathophysiology
        • In response to gastric epithelial injury, mainly by environmental factors such as Helicobacter pylori and Epstein-Barr virus (EBV) induced chronic inflammation, gastric stem cells in the affected region become dysfunctional, leading to dysregulated differentiation and gastric carcinoma (Gastrointest Tumors 2017;3:115)
        • EGC is preceded by a cascade of precancerous lesions: chronic gastritis (atrophic or not) → intestinal metaplasia (complete or not) → low grade dysplasia → high grade dysplasia (in situ carcinoma) → invasive carcinoma (J Dig Dis 2012;13:2)
        Etiology
        • Multifactorial disease
        • Sporadic in 90% of cases
        • Familial / hereditary setting in 10%: CDH1 mutation in hereditary diffuse gastric carcinoma, APC mutation and proximal polyposis
        • Environmental factors include smoking, diet (high intake of salt, preserved or smoked foods, meat), Helicobacter pylori or EBV infection
        • Reference: Gastroenterol Clin North Am 2013;42:219
        Diagrams / tables

        Contributed by Guedj Nathalie, M.D., Ph.D.
        Endoscopic subclassification

        Endoscopic subclassification

        Clinical features
        • Frequently asymptomatic
        • If symptomatic, clinical features are nonspecific (epigastralgia, dyspeptic syndrome)
        Diagnosis
        • Endoscopy, with or without the dye spraying method, is the method of choice
        • Diagnosis is confirmed with biopsies
        Laboratory
        • Anemia
        Radiology description
        • Endoscopic ultrasound (EUS) has become a reliable method for predicting the invasion depth of EGC
        Radiology images

        Images hosted on other servers:
        EUS, mucosal gastric cancer

        EUS, mucosal gastric cancer

        Prognostic factors
        • 5 year cancer specific survival for gastric cancer confined to the mucosa and submucosa is 99.3% and 96.7%, respectively (Dig Endosc 2020;32:180)
        • Lymph node metastasis is the most important prognostic factor in gastric carcinoma
        Case reports
        Treatment
        • Gastrectomy with lymphadenectomy is the gold standard treatment (Dig Endosc 2020;32:180)
        • However, when the prognosis of patients with endoscopic resection is similar to that of patients with gastrectomy in some categories of EGC, endoscopic resection is acceptable
        • Endoscopic resection should be en bloc because precise pathological diagnosis is essential for risk assessment of lymph node metastasis and there is potential risk of local recurrence after piecemeal resection
        • Endoscopic resection is sufficient if the risk of lymph node metastasis is low:
          • Tumor size < 30 mm
          • Absence of lymphatic invasion
          • Submucosal invasion < 500 μm (sm1)
          • Negative vertical margin
          • Absence of vascular invasion
        Clinical images

        Images hosted on other servers:
        Endoscopy, type 0 lesions

        Endoscopy, type 0 lesions

        Gross description
        • Macroscopically subclassified into 3 main types: 0-I, 0-II and 0-III (see Diagram above)
          • Type 0-I: protruding
          • Type 0-IIa: superficially elevated
          • Type 0-IIb: flat
          • Type 0-IIc: superficially depressed
          • Type 0-III: excavated
        • Reference: Rev Gastroenterol Mex (Engl Ed) 2020;85:69
        Gross images

        Contributed by Guedj Nathalie, M.D., Ph.D.
        Type 0-IIc

        Type 0-IIc

        Microscopic (histologic) description
        • Tubular adenocarcinoma is the most common histologic type (Gastrointest Tumors 2017;3:115)
          • Nuclear abnormalities
            • Hyperchromasia, nuclear enlargement, high nuclear to cytoplasmic ratio, marked pleomorphism, increased mitotic figures with atypical forms, prominent nucleoli
          • Glandular structures with subtle architectural changes, mimicking the letters W, H , Y or X at low power view
        • Papillary adenocarcinoma
          • Finger-like projections on the mucosal surface
          • Lined with neoplastic columnar cells
          • Micropapillary and tubular components
          • Worse prognosis compared with tubular adenocarcinoma: more lymphovascular invasion and nodal metastasis
        • Poorly cohesive carcinoma, including signet ring cell carcinoma
          • Neoplastic cells that are isolated or arranged in small aggregates without well formed glands
          • Signet ring cells are characterized by a central, optically clear, globoid droplet of cytoplasmic mucin with an eccentrically placed nucleus
          • There is low risk of nodal metastasis for intramucosal signet ring cell carcinoma
        • Mucinous adenocarcinoma
          • Malignant epithelium and extracellular mucin pools
        • Rare subtypes: gastric carcinoma with lymphoid stroma, hepatoid adenocarcinoma, micropapillary adenocarcinoma, gastric adenocarcinoma of fundic gland type
        Microscopic (histologic) images

        Contributed by Guedj Nathalie, M.D., Ph.D.
        Gastric adenocarcinoma

        Gastric adenocarcinoma

        Tubular adenocarcinoma

        Tubular adenocarcinoma

        Cytologic and nuclear atypia

        Cytologic and nuclear atypia

        Angioinvasion

        Angioinvasion

        Positive stains
        Negative stains
        Molecular / cytogenetics description
        Molecular / cytogenetics images
        Videos

        Early gastric carcinoma

        Sample pathology report
        • Stomach, distal gastrectomy:
          • Submucosal tubular gastric adenocarcinoma; an elevated lesion, type 0-IIa of 4 cm (see comment)
          • Comment: Histological study of whole lesion shows predominantly intramucosal adenocarcinoma, with focal extension beyond the muscularis mucosa. The infiltration depth does not exceed 500 μm. Carcinoma architecture is tubular. Lymphovascular invasion is present.
        Differential diagnosis
        Board review style question #1

        What is the most common histologic type of early gastric carcinoma?

        1. Hepatoid carcinoma
        2. Mucinous adenocarcinoma
        3. Papillary adenocarcinoma
        4. Signet ring cell carcinoma
        5. Tubular adenocarcinoma
        Board review style answer #1
        E. Tubular adenocarcinoma

        Comment Here

        Reference: Early gastric carcinoma
        Board review style question #2

        What is the most important prognostic factor of early gastric carcinoma?

        1. Angioinvasion
        2. Incomplete mucosal resection
        3. Lymph node metastasis
        4. Perineural invasion
        5. Tumor size
        Board review style answer #2
        C. Lymph node metastasis

        Comment Here

        Reference: Early gastric carcinoma

        Eosinophilic gastroenterocolitis

        Erosive gastritis
        Definition / general
        • Erosive gastritis is loss of superficial epithelium of the gastric mucosa caused by impairment of mucosal defenses
        Essential features
        • Loss of superficial epithelium of the gastric mucosa, erosion (wearing away away) of the stomach lining, along with inflammation
        • Common in the body of the stomach
        • Usually an acute inflammatory process
        • Diagnosis is mostly clinical and endoscopic
        • References: Arq Gastroenterol 2022;59:275, Eur J Radiol 1982;2:205
        Terminology
        • Acute erosive gastritis
        • Hemorrhagic gastritis
        • Acute hemorrhagic gastritis
        ICD coding
        • ICD-10: К29.60 - other gastritis without bleeding
        Epidemiology
        • Both genders, at any age, can be affected when exposed to the causative agents of erosive gastritis
        Sites
        • Typically occurs in the gastric body but the antrum may also be involved
        Pathophysiology
        • Mucosal damage due to increased acid secretion, decreased bicarbonate buffer and reduced blood flow
        • Increased acid production could be secondary to factors such as stressed state (due to various reasons such as, physiologic stress related to severe trauma, severe burns, ventilator dependency or intracranial trauma), elevated levels of ACH and histamine, resulting in increased acid production
        • Stress results in the release of angiotensin II, which decreases blood flow to the mucosa, causing reactive oxygen species formation, resulting in oxidative stress on the mucosa
        • Prolonged exposure of gastric mucosa to acids also increases risk for development of erosive gastritis
        • References: Histopathology 2007;50:15, J Gastroenterol 2010;45:131, Korean J Fam Med 2015;36:300
        Etiology
        • Common causes include nonsteroidal anti-inflammatory drugs (NSAIDs), alcohol and stress (burn, surgery, trauma)
        • Other causes are cancer chemotherapy, uremia, ischemia and shock, acid / alkali ingestion, mechanical trauma (nasogastric tube), distal gastrectomy, Helicobacter infection, hemodialysis and peritoneal dialysis (Front Immunol 2022;13:840916)
        Clinical features
        • Usually due to acute inflammatory processes but may be subacute or chronic
        • When acute, can present with GI bleeding (Front Immunol 2022;13:840916)
        • When subacute or chronic, mild nonspecific symptoms (pain, nausea and vomiting) or no symptoms
        • Lesions develop as soon as 12 hours after the insult
        • Can complicate to ulceration or perforation in severe untreated cases
        Diagnosis
        Case reports
        Treatment
        • Treatment is supportive
        • Includes acid suppressive therapy and removal of the inciting cause
        Clinical images

        Images hosted on other servers:

        Endoscopy

        Gross description
        Microscopic (histologic) description
        • Loss of superficial epithelium of the gastric mucosa
        • Mild to moderate edema, congested vessels and scattered neutrophils in the lamina propria
        • Ulceration, granulation tissue and hemorrhage in more severe disease
        • Moderate to severe acute inflammatory infiltrate, fibrinopurulent exudate, mucosal sloughing, regenerative changes in the adjacent epithelium in late disease
        • References: Arq Gastroenterol 2022;59:275, Korean J Gastroenterol 2017;69:278
        Microscopic (histologic) images

        Contributed by Iresha Vithanage, M.B.B.S., M.D. and Xiaoyan Liao, M.D., Ph.D.
        Gastric mucosa with erosive gastritis

        Gastric mucosa with erosive gastritis

        Surface erosion and inflammation Surface erosion and inflammation Surface erosion and inflammation

        Surface erosion and inflammation

        Acute erosive gastritis Acute erosive gastritis

        Acute erosive gastritis

        Positive stains
        • If Helicobacter pylori is the suspected cause, then immunohistochemistry and special stains can be used (see Helicobacter pylori)
        Sample pathology report
        • Stomach, body, biopsy:
          • Erosive gastritis
          • No Helicobacter-like organisms identified
          • No intestinal metaplasia is seen
        Differential diagnosis
        Board review style question #1
        A 58 year old man was admitted through the emergency department after suffering from polytrauma in a traffic accident. He becomes vitally stable after resuscitation, however, develops persistent nausea associated with epigastric pain. Gastroendoscopy shows small superficial mucosal erosions or erythematous lesions in the gastric body and fundus. Which of the following is true regarding the pathophysiology of development of this condition in this patient?

        1. Angiotensin II is released, which increases blood flow to the gastric mucosa
        2. Elevated levels of ACH and histamine in stress result in decreased acid production
        3. Increased acid secretion leads to the development of gastric erosions
        4. Naturally produced nitric oxide is an enhancing factor for gastric erosion
        5. There is no significant dysregulation of gastric pH during the process
        Board review style answer #1
        C. Increased acid secretion leads to the development of gastric erosions. One of the most common causes of erosive gastritis is stress. In the stressed state (may be due to physiologic stress related to severe trauma, severe burns, ventilator dependency or intracranial trauma), elevated levels of ACH and histamine result in increased acid production, thus inducing erosive gastritis, which can later on lead to hemorrhage if left undiagnosed / untreated. Naturally produced nitric oxide is believed to be protective against stress gastritis, as it leads to vasodilation. Angiotensin II is released as a result of stress, which decreases blood flow to the gastric mucosa, resulting in formation of reactive oxygen species (StatPearls: Stress Induced Gastritis [Accessed 6 February 2023]).

        Comment Here

        Reference: Erosive gastritis
        Board review style question #2

        A 42 year old woman with rheumatoid arthritis presents with dyspepsia and bloating. Gastroendoscopy shows small erythematous lesions in the gastric body. On histopathology, there are surface erosions with ectatic superficial capillaries and mild edema in the lamina propria (see image above). Intestinal metaplasia is not seen. What is the diagnosis?

        1. Erosive gastritis
        2. Gastric antral vascular ectasia (GAVE)
        3. H. pylori gastritis
        4. Low grade dysplasia
        5. Reactive gastropathy
        Board review style answer #2
        A. Erosive gastritis. Longstanding NSAID use (in this case, for rheumatoid arthritis) predisposes a patient to erosive gastritis. In this patient, there were small erythematous lesions in the gastric body and histology is compatible erosive gastritis. GAVE usually shows watermelon stomach; endoscopically and histologically there is increase in blood vessel number and diameter with fibrin thrombi and fibromuscular hyperplasia. H. pylori gastritis will show active chronic gastritis with organisms which can be seen on H&E, special stain or immunostain. Low grade dysplasia will show hyperchromatic and pleomorphic nuclei within tall columnar epithelium. There will be abrupt transition from benign to dysplastic epithelium. Reactive gastropathy is characterized by foveolar hyperplasia, mucin depletion in surface epithelial cells and lamina propria expansion by fibromuscular and capillary hyperplasia (Cureus 2020;12:e10873).

        Comment Here

        Reference: Erosive gastritis

        Extranodal marginal zone lymphoma of mucosa associated lymphoid tissue (MALT lymphoma)
        Definition / general
        Essential features
        • Stomach is the most common site involved by MALT lymphoma
        • Most gastric MALT lymphomas associated with gastritis caused by Helicobacter pylori infection
        • Microscopic features: centrocyte-like cells, monocytoid cells, plasma cells, occasional large transformed lymphocytes, lymphoepithelial lesions, adjacent nonneoplastic germinal centers
        • Atypical features of lymphoid infiltrate help distinguish MALT lymphoma from reactive conditions, such as H. pylori gastritis (J Clin Pathol 2007;60:361)
        Terminology
        • Also called MALToma
        ICD coding
        • ICD-10: C88.4 - Extranodal marginal zone B cell lymphoma of mucosa associated lymphoid tissue (MALT lymphoma)
        Epidemiology
        • 7 - 8% of all B cell non-Hodgkin lymphomas (Leuk Lymphoma 2013;54:1891)
        • Median age: seventh decade
        • M:F = ~1:1
        • Up to 50% of primary gastric lymphomas
        • 85 - 90% of all GI MALT lymphomas
        Sites
        • Stomach (35%) is most common site of extranodal marginal zone lymphoma (Blood 2016;127:2082)
        • Ocular adnexa (13%)
        • Lung (9%)
        • Skin (9%)
        • Salivary glands (8%)
        Etiology
        • Related to chronic inflammatory disorders driven by infectious agents or autoimmune stimuli (Blood 2016;127:2082)
        • Stomach: H. pylori associated gastritis
          • H. pylori infection attracts B cells, T cells and neutrophils to the gastric mucosa; B cell proliferation is driven by reactive T cells, as well as by cytokines
          • Chronic proliferative state of these B cells, as well as neutrophil mediated release of reactive oxygen species in areas of chronic inflammation, induces additional oncogenic events that eventually make lymphoproliferation independent of antigenic stimulation
          • Additional genetic alterations can ultimately result in transformation to clinically aggressive diffuse large B cell lymphoma (Nat Rev Gastroenterol Hepatol 2010;7:336)
        Clinical features
        • Can be asymptomatic or cause epigastric pain, anorexia, weight loss, anemia, early satiety, occasionally fever and night sweats
        • Most are low stage (stage I and II) at presentation
        • 23 - 40% of patients may present with disseminated disease, involving other mucosal sites, lymph nodes or bone marrow (Blood 2000;95:802)
        • > 75% of cases will regress with successful treatment for H. pylori
        • Increased risk has been reported in patients with chronic hepatitis C virus (HCV) infection (Blood 2016;127:2082)
        Diagnosis
        • Diagnostic workup includes endoscopic findings, histology with immunohistochemistry and molecular analysis
        Prognostic factors
        Case reports
        Treatment
        • H. pylori eradication - first line therapy (Gut 2012;61:507)
          • Curative in majority of gastric MALT lymphomas
          • Resolution can take up to 24 months
          • Some primary gastric diffuse large B cell lymphomas may also respond to antibiotic eradication therapy alone
        • If H. pylori eradication therapy fails → second line therapy (Med Oncol 2010;27:291)
          • Chemotherapy
          • Radiation
        • Surgery: only necessary if complications (perforation, obstruction, bleeding) occur
        Gross description
        • Large mass with or without ulcer
        • Diffusely thickened gastric wall
        • Polypoid lesions
        Microscopic (histologic) description
        • Mass lesion with the following features
        • Sheets of neoplastic small lymphocytes
          • Irregular / angulated nuclear contours with inconspicuous nucleoli (centrocyte-like)
          • Relatively abundant pale cytoplasm (monocytoid appearance)
          • Cells with plasmacytic differentiation
            • Clonally related to the small lymphocytes
            • ~33% of gastric lymphomas
            • Intranuclear (Dutcher bodies) or intracytoplasmic (Russell bodies) immunoglobulin inclusions can be seen
            • Nuclear polymorphism may be noted
          • May see occasional transformed lymphocytes resembling centroblasts and immunoblasts
            • Larger nuclei with more open chromatin and prominent nucleoli
            • If solid or sheet-like proliferations of transformed cells → diffuse large B cell lymphoma (Arch Pathol Lab Med 2018;142:44)
        • Lymphoepithelial lesions (specific but not sensitive)
          • Infiltration of gastric glands by neoplastic lymphocytes (> 3 centrocyte-like cells causing epithelial damage)
          • Residual gastric glands are separated by the neoplastic lymphocytes
          • Oxyphilic change (eosinophilic degeneration) of epithelial cells
          • May lead to destruction of glands
        • Adjacent reactive germinal centers
          • Neoplastic cell colonization may resemble follicular lymphoma
        • Amyloid deposition can be seen (Am J Clin Pathol 2012;137:51)
        • H. pylori gastritis associated changes can often be present in biopsies from patients
          • Active or chronic inflammation
          • Diffuse chronic monocytic infiltrate, microerosions, large lymphoid follicles and regenerative foveolar hyperplasia
        Microscopic (histologic) images

        Contributed by Kwun Wah Wen, M.D., Ph.D.
        Dense lymphoplasmacytic infiltrate

        Dense lymphoplasmacytic infiltrate

        Destruction of gastric glands

        Destruction of gastric glands

        Focus of active inflammation

        Focus of active inflammation

        Marked plasmacytic differentiation

        Marked plasmacytic differentiation

        Lymphoepithelial lesion

        Lymphoepithelial lesion

        Cytologic features of lymphoma cells

        Cytologic features of lymphoma cells


        Diffusely positive for CD20

        Diffusely positive for CD20

        Numerous kappa restricted cells (κ:λ = > 8:1)

        Numerous kappa restricted cells (κ:λ = > 8:1)

        Rare lambda positive cells (κ:λ = > 8:1)

        Rare lambda positive cells (κ:λ = > 8:1)

        Low proliferative index

        Low proliferative index

        Positive stains
        • Neoplastic cells of MALT lymphoma: CD19, CD20, CD22, CD79a, CD43 variable, BCL2 (Arch Pathol Lab Med 2018;142:44)
        • Kappa / lambda: can be useful to demonstrate light chain restriction (especially if there are abundant neoplastic plasma cells)
        • Staining for CD21, CD23 and CD35 typically reveals disrupted and occasionally expanded meshwork of intermixed follicular dendritic cells, which may help to recognize colonized follicles
        • Keratin: can accentuate lymphoepithelial lesions
        • Congo red: can highlight amyloid deposition present in a subset of cases
        • MNDA staining: MALT (61 - 75%) versus follicular (< 10%) lymphoma (Am J Clin Pathol 2019;151:337)
        • IRTA1 antibody: potential MALT specific antibody
        Negative stains
        Molecular / cytogenetics description
        • PCR can help confirm clonally rearranged immunoglobulin genes in challenging cases (those that show no aberrant expression of CD43 and BCL2)
        • t(11;18)(q21;q21): BIRC3 (API2)-MALT1 (6 - 26%) (Haematologica 2019;104:e558)
          • Associated with resistance to H. pylori eradication therapy
        • t(14;18)(q32;q21): IGH-MALT1 (1 - 5%)
        • t(1;14)(p22:q32): BCL10-IGH (not seen in gastric cases)
        • t(3;14)(p13;q32): FOXP1-IGH (not seen in gastric cases)
        • Trisomy 3 (11%)
        • Trisomy 18 (6%)
        • TNFAIP3 deletion / hypermethylation
        • Point mutations in Ig genes
        Molecular / cytogenetics images

        Images hosted on other servers:
        t(11;18)(q21;q21) translocation

        t(11;18)(q21;q21) translocation

        Sample pathology report
        • Gastric ulcers, biopsy:
          • Extranodal marginal zone lymphoma (MALT lymphoma) (see comment)
          • Comment: H&E sections of the biopsy show patchy involvement by a dense and atypical lymphoplasmacytic infiltrate with lymphoepithelial lesions. The background stomach shows chronic active gastritis. No carcinoma, dysplasia, Helicobacter organisms or intestinal metaplasia is present.
          • Immunostains were performed and evaluated with the following results:
            • CD3: Highlights background small reactive T cells.
            • CD20: Highlights diffuse sheets of neoplastic B cells in some fragments.
            • CD21: Highlights rare residual dendritic cell meshworks underlying one lymphoid follicle.
            • Kappa: Highlights B cells and plasma cells with kappa restriction (kappa:lambda ratio > 8:1).
            • Lambda: Highlights a rare subset of B cells and plasma cells (kappa:lambda ratio > 8:1).
            • H. pylori: Negative.
            • CD5: Negative in B cells; positive in background T cells.
            • CD10: Negative in B cells.
            • CD43: Negative in B cells; positive in T cells.
            • BCL1: Negative for mantle cell lymphoma.
            • BCL2: Positive in B cells and T cells.
            • CD23: Negative in B cells.
            • Ki67: Low proliferative index.
            • AE1 / AE3: Negative for carcinoma.
          • The above findings support the interpretation of extranodal marginal zone lymphoma in the stomach.
        Differential diagnosis
        Board review style question #1
        You are asked to consult on a 55 year old patient with mucosa associated lymphoid tissue (MALT) lymphoma of the stomach. She is H. pylori positive. PET-CT does not show any other evidence of disease outside the stomach. The lesion is 5 - 6 cm and confined to fundus. A bone marrow biopsy is negative. Which of the following statements about MALT lymphoma is true?

        1. Combination chemotherapy is used as the first curative approach
        2. Gastric MALT may progress into diffuse large B cell lymphoma
        3. H. pylori infection is associated with gastric MALT in a minority of cases
        4. Most patients present with multi-organ involvement
        Board review style answer #1
        B. Gastric MALT may progress into diffuse large B cell lymphoma

        Comment Here

        Reference: MALT lymphoma
        Board review style question #2
        Which cytogenetic / molecular abnormality of gastric MALT lymphoma is associated with resistance to H. pylori antibiotic treatment?

        1. t(11;18)(q21;q21): BIRC3-MALT1
        2. t(14;18)(q32;q21): IGH-MALT1
        3. TNFAIP3 deletion / hypermethylation
        4. Trisomy 3
        Board review style answer #2
        A. t(11;18)(q21;q21): BIRC3-MALT1

        Comment Here

        Reference: MALT lymphoma
        Board review style question #3

        Which of the following is true regarding MALT lymphoma of the stomach?

        1. Evidence of H. pylori infection should only be sought in the presence of symptoms, such as dyspepsia, bloating, nausea
        2. Lymphomas with sheets of large cells should be diagnosed as high grade MALT lymphoma
        3. Neoplastic lymphocytes are positive for CD5 and cyclin D1
        4. Neoplastic lymphocytes ranging from small lymphocytic to monocytoid cells infiltrating into epithelial structures are characteristic for gastric MALT lymphoma
        Board review style answer #3
        D. Neoplastic lymphocytes ranging from small lymphocytic to monocytoid cells infiltrating into epithelial structures are characteristic for gastric MALT lymphoma

        Comment Here

        Reference: MALT lymphoma

        Features to report & grossing
        Table of Contents
        Features to report | Grossing
        Features to report
        • Surgical procedure
        • History of prior gastric surgery (recommended)
        • Anatomic site
        • Tumor histologic type and grade
        • Extent of tumor
        • Tumor size
        • Depth of invasion
        • Lymphovascular invasion
        • Perineural invasion (recommended)
        • Margin involvement (proximal, distal, radial); if negative, report distance of tumor to nearest margin
        • Lymph node involvement (total involved, total examined)
        • Tumor descriptors if indicated (m-multiple, r-recurrent, y-posttreatment)
        • HER2 status (if known, results pending)
        • Presence of distant metastasis (if known)
        • Other features (gastritis [Helicobacter pylori or other]), intestinal metaplasia, dysplasia (low grade, high grade), specific types of infection
        • See also CAP.org, search "cancer protocols"

        Grossing
        • Take sections from resection margins, tumors / lesions, deep margins to tumors, all lymph nodes
        • For malignant gastrectomy specimens, should assess 15+ lymph nodes

        Fundic gland polyp
        Definition / general
        • Benign cystic hyperplastic proliferation of oxyntic gland
        • Traditionally regarded as hamartomatous lesions; however, studies have shown alterations in the APC beta catenin pathway in both sporadic and syndromic cases, supporting a neoplastic nature of these polyps (Mod Path 2002;15:718)
        Essential features
        • Can be sporadic, associated with familial adenomatous polyposis (FAP) or arise as part of a familial condition confined to the stomach without polyposis coli
        Terminology
        • Also known as Elster glandular cysts and cystic hamartomatous epithelial polyps
        Sites
        • Typically restricted to the fundus and body of the stomach
        Etiology
        • Sporadic cases are usually secondary to use of proton pump inhibitors and are usually solitary but may be multiple (Hum Pathol 2000;31:684)
        • Syndromic cases can arise from FAP or gastric adenocarcinoma and proximal polyposis of the stomach (GAPPS) (Gut 2012;61:774)
        Clinical features
        • Most commonly encountered form of gastric polyps
        • Peak incidence in fifth or sixth decade of life in sporadic cases and second and third decade in FAP patients
        • Sporadic cases rarely harbor dysplasia (Gasteroenterol Hepatol 2009;7:849)
        • FAP associated fundic gland polyps arise at younger ages, are more numerous and are more likely to show low grade dysplasia
        • Patients with GAPPS present with fundic gland polyposis and gastric adenocarcinoma at a young age
        Case reports
        Treatment
        • Current guidelines do not recommend polypectomy or surveillance endoscopy for sporadic cases
        • Findings of multiple fundic gland polyps in a younger patient or fundic gland polyp with dysplasia should prompt colonic investigation for the possibility of familial adenomatous polyposis
        • In GAPPS, gastrectomy may be indicated
        Clinical images

        Images hosted on other servers:

        Pedunculated lesion

        Multiple polyps

        Gross description
        • Typically small and sessile, with a glassy surface, usually less than a centimeter
        Microscopic (histologic) description
        • Cystically dilated glands lined by chief cells, parietal cells and mucinous foveolar cells
        • Hyperplastic parietal cells with apocrine snouting is seen in patients on proton pump inhibitors
        • May rarely contain morules (Int J Clin Exp Pathol 2014;7:1241)
        • May harbor dysplasia
        Microscopic (histologic) images

        Contributed by Naziheh Assarzadegan, M.D.
        Missing Image Missing Image

        Cystically dilated glands

        Missing Image

        Fundic gland polyp with dysplasia



        Case #389

        Specialized gastric mucosa

        Scattered dilated fundic type glands

        Nuclear enlargement, hyperchromasia and stratification


        Fundic gland polyp with dysplasia

        Positive stains
        Molecular / cytogenetics description
        • Associated with APC beta catenin alteration in both sporadic and syndromic cases
        Sample pathology report
        • Stomach, fundus, polypectomy:
          • Fundic gland polyp
        • Stomach, fundus, resection:
          • Portion of stomach with numerous fundic gland polyps, some with focal low grade dysplasia (see comment)
          • Negative for high grade dysplasia or malignancy
          • Margins of resection unremarkable
          • Comment: Multiple fundic gland polyps can arise in several settings, including proton pump inhibitor use, familial adenomatous polyposis and gastric adenocarcinoma and proximal polyposis of the stomach.
        Differential diagnosis
        Board review style question #1

          Which of the following statements is true about gastric fundic gland polyps?

        1. Alterations of APC beta catenin pathway is seen in both sporadic and syndromic fundic gland polyps
        2. Dysplasia is a common finding in fundic gland polyps
        3. Only familial forms are associated with alteration in APC beta catenin pathway
        4. They are unequivocally benign and nonneoplastic
        Board review style answer #1
        A. Alterations of APC beta catenin pathway is seen in both sporadic and syndromic fundic gland polyps

        Comment Here

        Reference: Fundic gland polyp

        Gastric adenosquamous carcinoma
        Definition / general
        • Rare; neoplastic squamous component at least 25% of tumor
        • Behavior determined by glandular component
        Prognostic factors
        • Generally poor prognosis
        Microscopic (histologic) description
        • By definition, neoplastic squamous component at least 25% of tumor; generally transitional zone between squamous and glandular component; lymphovascular invasion common
        Positive stains

        Gastric antral vascular ectasia
        Definition / general
        • Antral vascular malformation with dilation of superficial mucosal and submucosal blood vessels, often with associated fibrin microthrombi
        Essential features
        • Accounts for up to 4% of nonvariceal gastrointestinal bleeding, usually chronic with associated iron deficiency
        • Histologically characterized by variable mucosal and submucosal vessel ectasia, fibrin thrombi, reactive foveolar changes and fibromuscular hyperplasia
        • May present clinically as striped (watermelon stomach) form in younger males with cirrhosis of the liver, diffuse punctate (honeycomb stomach) form in older females with autoimmune disorders or nodular form
        • Endoscopic ablative therapy such as argon plasma coagulation (APC) is first line treatment; antrectomy only if severe and refractory (due to high postoperative mortality rate)
        • Primary differential includes portal hypertensive gastropathy, which may present with vascular ectasia but lacks fibrin thrombi
        Terminology
        • Linear striped type referred to as watermelon stomach
        • Diffuse punctate type called honeycomb stomach
        • Originally described as erosive gastritis with venocapillary ectasia by Reider in 1953
        ICD coding
        • ICD-10:
          • K31.811 - angiodysplasia of stomach and duodenum with bleeding
          • K31.819 - angiodysplasia of stomach and duodenum without bleeding
        • ICD-11: DA43.4 - diffuse vascular ectasia of stomach
        Epidemiology
        Sites
        • Gastric antrum
        • Rare extra-antral sites include gastric cardia, duodenum, jejunum and rectum (Gut 1993;34:558)
        Pathophysiology
        • Pathophysiology is unknown, though several theories have been proposed:
          • Chronic liver dysfunction leads to increased blood concentrations of vasodilatory hormones and metabolites such as gastrin, VIP and PGE2 (World J Gastrointest Endosc 2013;5:6)
          • Mechanical stress from gastric peristalsis induces prolapse, antral mucosal trauma and vessel obstruction, resulting in vascular ectasia and fibromuscular hyperplasia
          • Autoimmune antibodies react with proteins that are present in the gastric mucosal and submucosal vessels
        Etiology
        • Unknown
        Clinical features
        Diagnosis
        • Esophagogastroduodenoscopy (EGD) is gold standard for endoscopy (Semin Arthritis Rheum 2020;50:938)
        • Antral biopsy may be useful to rule out portal hypertensive gastropathy if comorbid cirrhosis
          • Biopsy may not exclude GAVE, as lesions may be focal
        • Fecal occult blood tests
        Laboratory
        • Anemia
        • Iron deficiency
        • Anti-RNA polymerase III (RNAPIII) antibodies present in up to 71% of GAVE patients with systemic sclerosis (Semin Arthritis Rheum 2020;50:938)
          • Concomitant GAVE and anti-RNAPIII is strongly correlated with scleroderma renal crisis
        Radiology description
        Prognostic factors
        • Exact mortality is unknown but appears to be low (Gastroenterology Res 2021;14:104)
        • Recurrence rate is ~40%
        • Factors associated with poorer prognosis (Gastroenterology Res 2021;14:104):
          • Hemorrhage associated with higher mortality
          • Mortality increases with number of comorbidities
          • Hispanic status may be associated with poorer prognosis
          • Higher mortality rate if refractory to endoscopic ablation
        Case reports
        Treatment
        Clinical images

        Contributed by Tony El Jabbour, M.D.

        Diffuse punctate pattern



        Images hosted on other servers:

        Striped pattern

        Diffuse punctate pattern

        Nodular pattern

        Microscopic (histologic) description
        • Dilated mucosal capillaries
        • Reactive foveolar changes
        • Intravascular fibrin thrombi
        • Fibromuscular hyperplasia of lamina propria
        • Fibrohyalinosis (perivascular hyalinization)
        Microscopic (histologic) images

        Contributed by Hwajeong (Jenny) Lee, M.D.

        Antrum with reactive changes

        Fibrin thrombus

        Videos

        Histopathology of gastric antral vascular ectasia

        Sample pathology report
        • Antrum, biopsy:
          • Gastric antral mucosa with vascular dilation in the lamina propria with extravasation of red blood cells and rare fibrin thrombi (see comment)
          • Comment: These features could be seen in gastric antral vascular ectasia (GAVE) and portal hypertensive gastropathy. Correlation with the clinical history and endoscopic aspect of the mucosa at the site of the biopsy are necessary to establish the diagnosis.
        Differential diagnosis
        • Portal hypertensive gastropathy (World J Gastrointest Endosc 2013;5:6):
          • May present with vascular ectasia
          • Affects gastric fundus and corpus
          • Endoscopically presents with mosaic red point lesions, cherry red spots or black-brown spots
          • Lacks fibrin microthrombi
          • Responds to portal hypertensive therapies (beta blockers, portocaval shunt, etc.)
        • Telangiectasia:
        • Reactive gastropathy:
          • Appears with similar, if identical, reactive foveolar changes
          • May contain mucosal capillary ectasia
          • Typically lacks fibrin microthrombi
        • Hyperplastic polyps:
          • May mimic nodular GAVE on endoscopy
          • Contains elongated, cystically dilated foveolar epithelium
          • More prominent inflammation in lamina propria
        Board review style question #1

        The microphotograph above demonstrates a section obtained from a biopsy of the gastric antrum in a 71 year old woman presenting with chronic melena and iron deficiency anemia. Which of the following is true of this patient's disease?

        1. Antrectomy is associated with the most favorable postoperative outcomes
        2. Endoscopic ablative treatments such as argon plasma coagulation are first line
        3. It typically presents as acute heavy upper gastrointestinal bleeding
        4. On endoscopy, it usually presents as parallel red stripes in patients with comorbid cirrhosis
        Board review style answer #1
        B. Endoscopic ablative treatments such as argon plasma coagulation are first line

        Comment Here

        Reference: Gastric antral vascular ectasia
        Board review style question #2
        Which of the following is true of both gastric antral vascular ectasia (GAVE) and portal hypertensive gastropathy?

        1. Dilated mucosal capillaries are routinely present on histology
        2. Endoscopically presents as mosaic red point lesions and black-brown spots
        3. Histologically characterized by intravascular fibrin thrombi
        4. Responsive to antihypertensive therapies such as beta blockers
        Board review style answer #2
        A. Dilated mucosal capillaries are routinely present on histology

        Comment Here

        Reference: Gastric antral vascular ectasia

        Gastric carcinoma with lymphoid stroma
        Definition / general
        • Gastric carcinoma composed of small nests of cancer cells broadly distributed in a background of dense and prominent lymphoid stroma, histologicically similar to nasopharyngeal carcinoma (Arch Pathol Lab Med 2008;132:706)
        Essential features
        • Infrequent subtype of gastric cancer composed of small trabeculae and nests of epithelial cells embedded in a dense lymphoid infiltrate reminiscent of lymphoid tissue
        • Strongly associated with Epstein-Barr virus (EBV) infection and microsatellite instability (mutually exclusive)
        • PDL1 is usually overexpressed, making these tumors prone to immune checkpoint blockade therapy
        Terminology
        • Lymphoepithelioma-like carcinoma
        • Medullary carcinoma
        • Undifferentiated carcinoma with lymphoid stroma
        ICD coding
        • ICD-O: 8140/3 - adenocarcinoma, NOS
        Epidemiology
        Sites
        Pathophysiology
        Etiology
        Diagnosis
        Radiology description
        • CT scan: well circumscribed mass with a large thickness to length ratio with the low density stripe of the normal gastric wall abruptly terminated at the edge of the lesion (Medicine (Baltimore) 2019;98:e14839)
        Prognostic factors
        Case reports
        Treatment
        • Localized cases are prone to curative submucosal dissection (World J Gastroenterol 2014;20:1365)
        • High sensitivity to immune checkpoint blockade therapy with pembrolizumab (anti-PD1 antibody) has been observed in advanced EBV and microsatellite instability gastric cancer (overall response rate of 85.7% in microsatellite instability and overall response rate of 100% in EBV positive cases) (Nat Med 2018;24:1449)
        Gross description
        Microscopic (histologic) description
        Microscopic (histologic) images

        Contributed by Carolina Martinez-Ciarpaglini, M.D., Ph.D.

        Dense lymphoid infiltrate

        Trabecular pattern

        Tumor cells (high power)

        Primitive tubular pattern

        EBER expression

        Cytokeratin expresion


        T cell infiltrate (CD3)

        Positive stains
        Negative stains
        Molecular / cytogenetics description
        • For microsatellite instability evaluation in gastric cancer, the immunohistochemical study of MMR proteins and PCR show an excellent concordance (ESMO Open 2019;4:e000470)
        • EBV positive tumors show the higher prevalence of DNA hypermethylation among all gastric tumors (Nature 2014;513:202)
        • The main molecular alterations in EBV associated cases include hypermethylation of the promoter region of the gene CDKN2A (p16INK4A) and mutation in phosphatidylinositol-3-kinase catalytic subunit alpha (PIK3CA) in about 5 - 10% (Oncotarget 2016;7:32925, Nature 2014;513:202, ESMO Open 2019;4:e000470)
        • MLH1 hypermethylation is characteristic of microsatellite instability associated cases (Nature 2014;513:202)
        • Microsatellite instability cases show high tumor mutation burden and KRAS alterations (56%) (Am J Surg Pathol 2018;42:453)
        Sample pathology report
        • Gastric mass, upper body, endoscopic biopsy:
          • Gastric carcinoma with lymphoid stroma
          • In situ hybridization study for EBER is diffusely positive in all tumor cells
        Differential diagnosis
        Board review style question #1

          The image shows a gastric endoscopic biopsy of a well circumscribed tumor located in the antrum. Epstein-Barr in situ hybridization study was completely negative. Which molecular alteration is probably present in this lesion?

        1. BRAF mutation
        2. HER2 overexpression
        3. Microsatellite instability
        4. MYC translocation
        Board review style question #2
          Which of the following proteins is frequently overexpressed in gastric carcinoma with lymphoid stroma?

        1. HER2
        2. p16
        3. p53
        4. PDL1

        Gastric squamous cell carcinoma
        Definition / general
        • By definition, must be surrounded on all sides by gastric mucosa; otherwise may be esophageal carcinoma with gastric extension
        • Usually have small glandular component if searched for
        • Mucoepidermoid carcinoma: very rare
        • Undifferentiated carcinoma: very rare
        • Endodermal sinus tumor: very rare
        • Embryonal carcinoma: very rare
        Case reports

        Gastric undifferentiated carcinoma (pending)
        [Pending]

        Gastritis cystica polyposa
        Definition / general
        • Hyperplastic polyp containing foveolar or glandular epithelium in submucosa or muscularis propria
        • Also called polypoid mucosal prolapse
        Gross description
        • Usually on gastric side of gastroenterostomy stomas
        • Large sessile polyp grossly resembling hyperplastic polyp
        Microscopic (histologic) description
        • Resembles gastric hyperplastic polyp and reflux gastritis with pit hyperplasia, distortion, dilation
        • May contain submucosal cysts surrounded by disorganized smooth muscle
        • Increased mixed inflammatory cells in lamina propria
        Microscopic (histologic) images

        Contributed by @RaulSGonzalezMD on Twitter
        Gastritis cystica polyposa Gastritis cystica polyposa Gastritis cystica polyposa

        Gastritis cystica polyposa



        Image hosted on other server:

        Elongated, tortuous
        foveolae and hyperplastic
        and cystically dilated
        underlying pyloric gland

        Differential diagnosis

        Gastritis-features to report
        Definition / general
        • Features to report by international protocols
        Essential features
        • Chronic and active gastritis should be graded as mild, moderate and marked
        • The presence and extent of intestinal metaplasia (IM) should be documented
        • H. pylori should be reported as present or absent
        • Dysplasia if present should be graded as low or high grade
        Terminology
        • Chronic gastritis, chronic inflammation, active gastritis, chronic active gastritis
        ICD coding
        • ICD-11
          • DA42 - gastritis
            • DA42.0 - autoimmune gastritis
            • DA42.1 - Helicobacter pylori induced gastritis
            • DA42.2 - eosinophilic gastritis
            • DA42.3 - lymphocytic gastritis
            • DA42.4 - allergic gastritis
            • DA42.5 - gastritis due to duodenogastric reflux
            • DA42.7 - gastritis of unknown etiology with specific endoscopic or pathological features
            • DA42.8 - gastritis due to external causes
            • DA42.9 - gastric phlegmon
            • DA42.Y - other specified gastritis
            • DA42.Z - gastritis, unspecified
        Important features to report
        • Location / site
          • 5 biopsy specimens are essential
            • 2 from antrum (2 - 3 cm from the pylorus, 1 each from the lesser and greater curvature), 2 from the body (8 cm from the cardia, 1 each from the lesser and greater curvature) and incisura angularis
          • Biopsies from different sites should be separately identifiable when submitted to the laboratory
            • Note: it has proven useful to ink the antrum greater curve with red and incisura with black and place antrum lesser and greater curve and incisura on 1 slide, while body greater curve is inked red and placed together with lesser curve biopsy on another slide
            • Pathologist should be informed about endoscopic findings, clinical history and medications taken in the previous 2 months, especially NSAIDS and proton pump inhibitors (PPIs)
          • Optional additional biopsies from suspected lesions or according to local epidemiological conditions with respect to gastritis type and incidence of gastric cancer
        • Morphological variables
          • Graded (mild, moderate and marked)
            • H. pylori density
              • Essential reporting whether H. pylori is present or absent
              • Density grading is done for epidemiological associations and should be on the gastric epithelium and not in the intestinal metaplastic areas
            • Neutrophilic activity
              • Grading of neutrophilic density is done in context of chronic active gastritis due to H. pylori
              • Accumulated in the lamina propria, within the epithelium (glandular neck) and foveolar lumen (pit abscess)
              • Density is correlated to the extent of mucosal damage and intensity of H. pylori infection
            • Chronic inflammation
              • Grading of mononuclear leukocytes and plasma cells should be done
              • 2 - 5 lymphocytes, plasma cells and macrophages per high power field in the lamina propria or 2 - 3 lymphocytes or plasma cells in between the gastric foveolae is normally seen
              • Grading chronic inflammatory cells is important to monitor the pre and post antibacterial treatment, especially H. pylori
            • Atrophy
              • Loss of glandular tissue is termed as atrophy; in endoscopic biopsies, it may be challenging to identify atrophy especially in the presence of intestinal or pseudopyloric metaplasia
              • Visual analog scale should be followed to grade the atrophy
            • Intestinal metaplasia
              • Presence of intestinal type epithelium replete with goblet cells, with or without brush border, Paneth cells and columnar cells is known as IM
              • Routine subtyping of IM is not recommended in clinical set up and should be confined to research settings only
            • Operative link on gastritis assessment (OLGA) / operative link on intestinal metaplasia assessment (OLGIM)
              • Recently histological classification known as OLGA (operative link on gastritis assessment) was proposed to grade gastritis into stages with corresponding risks for gastric cancer (GC) (Dig Liver Dis 2008;40:8)
              • Due to poor interobserver agreement in grading atrophy, IM is currently being used to grade gastritis (OLGIM) (Gastrointest Endosc 2010;71:1150)
              • OLGA / OLGIM utilizes the percentage of atrophy / IM given by Sydney protocol in all of the 5 biopsies
                • Average sum of percentages of atrophy / IM in incisura angularis and antrum (lesser and greater curves) is regarded as cumulative atrophy / IM grade in antrum
                  • While the average sum of percentages of atrophy / IM in the body (lesser and greater curves) is regarded as cumulative atrophy / IM grade in the body
                • Stages III and IV are associated with gastric cancer
                • European Society of Gastrointestinal Endoscopy (ESGE) 2019 and Academy of Medicine, Singapore clinical guidelines recommends endoscopic surveillance every 3 years for patients with OLGA / OLGIM stage III and IV (Endoscopy 2019;51:365, Ann Acad Med Singap 2022;51:417)
            • Dysplasia, if present, should be documented as low or high grade according to the Vienna classification (Gut 2002;51:130)
          • Nongraded
            • Surface epithelial damage, mucus depletion and erosion especially in the context of H. pylori infection should be noted
              • Distinguish true erosions (fibrin deposits, neutrophilic infiltrate and regenerative changes) from detachment of surface epithelium due to mishandling and processing
            • Lymphoid aggregates are hallmarks of H. pylori infection; hence, not graded
              • Presence of lymphoid aggregates in the absence of H. pylori indicates either overlooking the presence of the bacterium, sampling error or H. pylori infection that has cleared
              • Irregular or large lymphoid aggregates or dense aggregates of lymphocytes warrant the diagnosis of mucosa associated lymphoid tissue (MALT)
            • Foveolar hyperplasia
              • Characterized by increased length and tortuosity of foveolae along with the expansion of the proliferative zone and increased nuclear size
              • It is most prominent in chemical gastritis
            • Pseudopyloric metaplasia (spasmolytic polypeptide expressing metaplasia)
              • These are metaplastic glands resembling pyloric glands that are observed in the body of the stomach
              • Pseudopyloric glands must be differentiated from the true antral glands especially if the location of the biopsy is not confirmed
              • True antral glands have G or the gastrin cells while these are absent in the pseudopyloric glands
              • Note: IHC staining for gastrin can be performed to ascertain the site in cases with pseudopyloric metaplasia
              • Pseudopyloric glands also secrete pepsinogen I and II and have enterochromaffin cells while the antral glands secrete only pepsinogen I
            • Pancreatic metaplasia
              • Characterized by replacement of the gastric glands by the pancreatic acinar cells
              • Pancreatic metaplasia is associated with IM and gastritis
            • Endocrine cell hyperplasia
              • Enterochromaffin-like cell hyperplasia is seen in autoimmune gastritis (AIG)
              • Hypochlorhydria or achlorhydria drives the G cells which secretes excessive gastrin, which in turn leads to the hyperplasia of the histamine producing enterochromaffin-like cells
              • Special stains such as Gromelius, IHC for chromogranin A and synaptophysin can be used to detect the enterochromaffin-like cell hyperplasia
        • Topography
          • Recommendations
            • Patterns of chronic gastritis should be documented as diffuse (both antrum and corpus), antral predominant or corpus predominant
            • Patterns of atrophy and IM should be categorized as multifocal or diffuse (multifocal atrophy and IM are characterized by patchy glandular loss and IM alternating with well preserved glandular layer)
            • Antrum usually exhibits 1 grade higher prominent gastritis than the corpus
            • Corpus predominant gastritis is usually the result of autoimmune and lymphocytic gastritis
            • Multifocal atrophic gastritis is mostly caused by H. pylori infection and increases the risk for gastric cancer
        • Etiology
          • If H. pylori is found in the biopsy, then it should be incorporated in the report as it is the major cause of gastritis; however, if the organism cannot be found in the specimen but has a positive serology test or urea breath test, then a diagnosis of chronic gastritis without any further qualification is warranted
          • Corpus predominant gastritis and atrophy without any accompanying metaplasia or atrophy in the antrum favors the diagnosis of autoimmune gastritis; however, final diagnosis should be confirmed with a clinicopathological correlation
        • Acute gastritis
          • Usage of the term acute gastritis is discouraged because of the confusion between acute and active gastritis
        • Special forms of gastritis
          • Chemical or reactive gastritis
            • Foveolar hyperplasia, mucosal oedema, smooth muscle proliferation in lamina propria with a minor increase in the inflammatory cells
            • Clinical information is required to make a diagnosis (e.g., bile reflux induced gastritis, NSAID associated chemical gastritis)
          • Lymphocytic gastritis
            • Characterized by > 25 intraepithelial lymphocytes per 100 cells
            • Usually associated with varioliform gastritis, Menetrier disease and celiac disease
          • Granulomatous gastritis
            • Granulomas can be seen in cases of Crohn's disease, tuberculosis, histoplasmosis, anisakiasis, sarcoidosis, etc.
          • Eosinophilic gastritis
            • Eosinophils are the dominant cell type with little or no increase in any other cell types
            • Stomach is the most common site and infiltration is highest in antrum
          • Collagenous gastritis
            • Thick band of collagen beneath the surface epithelium is seen in this gastritis
          • Radiation gastritis
            • Characterized by necrosis of fundal glands, edema and mononuclear infiltration
          • Infectious gastritis
            • Mycobacterium avium intracellulare, commonly seen in HIV patients, is diagnosed by accumulation of foamy histiocytes in lamina propria and ill defined granulomas without necrosis
            • Syphilitic gastritis, also seen commonly in HIV patients, is characterized by prominent mixed inflammatory infiltrate, with abundant plasma cells, mucosal ulcerations, mononuclear vasculitis of larger vessels in submucosa and muscularis mucosa
          • H. heilmanii gastritis
            • Rare compared with H. pylori gastritis
            • More commonly seen in children as compared with adults
            • Histologically shows fewer lymphoid aggregates and lesser neutrophils as compared with H. pylori
            • H. heilmanii is 3.5 - 7.5 microns in size with long tight spirals
          • Cytomegalovirus (CMV) gastritis
            • Commonly seen in immunocompromised patients and children
            • Gastric mucosa may appear normal or show erosion or shallow ulcers or pseudotumor (grossly nodular mucosa)
            • Depending on the host's immune response, patients may show numerous CMV inclusions in epithelial cells, endothelial cells and macrophages with little or no inflammatory response
            • Sometimes intense eosinophilic infiltration is seen, suggesting an allergic response
          • Fungal gastritis
            • Seen in immunocompromised subjects especially in HIV patients
            • Gastric candidiasis appears as erosions with fungal hyphae lying at the base of ulcer
          • Drug induced gastritis
            • Iron pill gastritis due to iron intake in iron deficiency anemia shows crystalline brown pigmented material deposition, erosion, ulceration and granulation tissues
            • Immune therapy induced gastritis is relatively rare but is increasingly being reported in the last 2 - 3 years
            • It is most caused by anti-PD1 drugs like pembrolizumab and nivolumab administered as a single agent or in combination
            • Histologically diffuse chronic active gastritis with increased lymphocytic infiltration, apoptosis are seen (Front Oncol 2023:13:1164236)
        Diagrams / tables

        Contributed by Supriya Srivastava, M.D., Ph.D.
        Updated Sydney protocol

        Updated Sydney protocol

        OLGIM staging

        OLGIM staging

        Microscopic (histologic) images

        Contributed by Supriya Srivastava, M.D., Ph.D.
        Mild chronic gastritis

        Mild chronic gastritis

        Moderate chronic gastritis

        Moderate chronic gastritis

        Marked chronic gastritis

        Marked chronic gastritis

        Mild intestinal metaplasia

        Mild intestinal metaplasia

        Moderate intestinal metaplasia

        Moderate intestinal metaplasia

        Marked intestinal metaplasia

        Marked intestinal metaplasia


        Chronic active gastritis

        Chronic active gastritis

        H. pylori gastritis

        H. pylori gastritis

        Autoimmune gastritis Autoimmune gastritis

        Autoimmune gastritis

        H. heilmanii gastritis H. heilmanii gastritis

        H. heilmanii gastritis

        Positive stains
        • Alcian blue at pH 2.5: diagnostic stain, however not routinely needed as gastric intestinal metaplasia can be readily identified in H&E; Alcian blue stains goblet cells bright blue in color due to the presence of sialomucins
        • IHC for H. pylori detection offers better sensitivity and specificity than the traditional histochemical stains (Adv Anat Pathol 2020;27:193)
        • Giemsa, modified Giemsa and Warthin-Starry for H. pylori and H. heilmanii detection
        • Synaptophysin and chromogranin A for neuroendocrine hyperplasia
        Sample pathology report
        • Stomach, antral and body biopsy:
          • Mild chronic gastritis with intestinal metaplasia (see comment)
          • Comment: Antrum greater curve (red inked) shows mild chronic gastritis with moderate intestinal metaplasia; no evidence of H. pylori, dysplasia or malignancy. Antrum lesser curve shows mild chronic gastritis with marked intestinal metaplasia; no evidence of H. pylori, dysplasia or malignancy. Incisura (black inked) shows mild chronic gastritis with mild intestinal metaplasia; no evidence of H. pylori, dysplasia or malignancy. Body greater curve (red inked) shows mild chronic gastritis; no evidence of intestinal metaplasia, H. pylori, dysplasia or malignancy. Body lesser curve shows mild chronic gastritis with mild intestinal metaplasia; no evidence of H. pylori, dysplasia or malignancy.
        Board review style question #1

        A 50 year old female working professional visited the general practitioner with complaints of fatigue, malaise and gastritis. In addition, she gave a history of diabetes since childhood. The general practitioner referred her to a gastroenterologist for endoscopic guided biopsy, which showed the picture above in the corpus. The antrum (lesser and greater curve) and incisura angularis showed features of mild chronic gastritis only. What could be the most likely diagnosis?

        1. Autoimmune gastritis
        2. Eosinophilic gastritis
        3. Lymphocytic gastritis
        4. Reactive gastropathy
        Board review style answer #1
        A. Autoimmune gastritis. In this context, histology likely shows features of autoimmune gastritis induced by the autoantibodies against the parietal cells and intrinsic factor leading to marked intestinal metaplasia (IM) in the body only. In addition, the patient shows features of anemia (fatigue and malaise) and associated autoimmune disorder (type 1 diabetes) which are features associated with autoimmune gastritis. Further laboratory work up is warranted to confirm the diagnosis. Answer C is incorrect because lymphocytic gastritis occurs in association with celiac disease and shows increased intraepithelial lymphocytes with expanded lamina propria. Answer B is incorrect because eosinophilic gastritis is associated with allergies, hay fever and often eczema with > 30 eosinophils per high power field. Answer D is incorrect as reactive gastropathy shows foveolar hyperplasia with corkscrew appearance of the gastric glands.

        Comment Here

        Reference: Gastritis-features to report
        Board review style question #2
        A 58 year old man had symptoms of reflux and a dull pain in the epigastrium for the past few days. He went to the general practitioner who referred him to a gastroenterologist for endoscopy and biopsy. Histology showed marked intestinal metaplasia (IM) (80%, 65% and 70%) in antrum lesser, greater curve and incisura respectively and mild IM (25%) in the body lesser curve. What would be the OLGIM stage for this patient?

        1. OLGIM I
        2. OLGIM II
        3. OLGIM III
        4. OLGIM IV
        Board review style answer #2
        C. OLGIM III. Answers A, B and D are incorrect because the stage in this patient is OLGIM III.The average intestinal metaplasia (IM) in antrum is (80+65+70/3) 72%. Average IM in body is (0+25/2) 12.5%. Using the OLGIM chart, the patient has marked IM (grade 3) in antrum and mild IM (grade I) in body; hence, he has stage III OLGIM. Patients with OLGIM stage III or IV have a higher risk of progression to gastric cancer. The ESGE 2019 and Academy of Medicine, Singapore clinical guidelines recommend endoscopic surveillance every 3 years for such patients.

        Comment Here

        Reference: Gastritis-features to report

        Gastroblastoma
        Definition / general
        Essential features
        • Rare, biphasic epitheliomesenchymal neoplasm of the stomach
        • Majority of cases have characteristic fusion gene, MALAT1::GLI1
        • Novel EWSR1::CTBP1 fusion was recently identified
        • Mostly seen in young adults
        ICD coding
        • ICD-O: 8976/3 - gastroblastoma
        • ICD-11: 2F70.1 & XH4VQ1 - neoplasms of uncertain behavior of stomach & gastroblastoma
        Epidemiology
        Sites
        Pathophysiology
        Etiology
        • Unknown
        Clinical features
        Diagnosis
        • Imaging followed by tissue diagnosis
        Radiology description
        Radiology images

        Images hosted on other servers:

        CT and MRI with multilobulated cystic mass

        Prognostic factors
        • Unknown at this time
        Case reports
        Treatment
        Clinical images

        Images hosted on other servers:

        Lobulated stomach mass

        Gross description
        Gross images

        Images hosted on other servers:

        Solid and cystic mass

        Variegated mass

        Transmural mass

        Microscopic (histologic) description
        • Biphasic tumor
          • Spindle cell component demonstrating sheets of monotonous spindled type cells
          • Epithelial component demonstrating cords / clusters of epithelial type cells, some with vague to well formed gland formation, which may contain inspissated material
        • Relatively low mitotic rate (Am J Surg Pathol 2009;33:1370)
        Microscopic (histologic) images

        Contributed by Raul S. Gonzalez, M.D. and Rondell P. Graham, M.B.B.S.

        Invasive tumor

        Epithelial component

        Nests of epithelioid cells

        Spindle cell component

        Positive stains
        Negative stains
        Electron microscopy description
        Electron microscopy images

        Images hosted on other servers:

        Desmosomes and microvilli between tumor cells

        Molecular / cytogenetics description
        Molecular / cytogenetics images

        Images hosted on other servers:

        RT-PCR, MALAT1::GLI1 fusion and sequencing

        GLI1 FISH
        breakapart and
        MALAT1::GLI1
        fusion FISH

        Sample pathology report
        • Stomach, partial gastrectomy:
          • Gastroblastoma (see comment)
          • Comment: H&E sections demonstrate a biphasic tumor with varied intermixed epithelial and stromal elements. The epithelial component is positive for keratin AE1 / AE3 while the stromal component is positive for vimentin. Both components are negative for KIT, CD34, desmin, S100, CD99 and SMA. Additional ancillary testing demonstrates the MALAT1::GLI1 fusion gene.
        Differential diagnosis
        Board review style question #1
        Which fusion gene typifies gastroblastoma?

        1. ASPL::TFE3
        2. BRAF::KIAA1549
        3. ETV6(TEL)::NTRK3
        4. MALAT1::GLI1
        5. SYT::SSX1
        Board review style answer #1
        D. MALAT1::GLI1

        Comment Here

        Reference: Gastroblastoma
        Board review style question #2

        A large gastric antral tumor is excised in a 15 year old boy. Histologically, you identify 2 differing cellular components with spindled and epithelioid morphology (see image above). KIT, DOG1, SMA, desmin and S100 are negative. Which tumor type could this be?

        1. Gastroblastoma
        2. Gastrointestinal stromal tumor
        3. Granular cell tumor
        4. Leiomyoma
        5. Schwannoma
        Board review style answer #2
        A. Gastroblastoma

        Comment Here

        Reference: Gastroblastoma

        GIST
        Definition / general
        • Most common mesenchymal tumor of the gastrointestinal tract
        • Arises from the interstitial cells of Cajal within the myenteric plexus of the muscularis propria
        • Micro / mini / subclinical GIST: minute growths (1 - 10 mm) of interstitial cells of Cajal / GIST-like cells (Am J Pathol 2002;160:1567)
        • Extragastrointestinal GIST: GIST arising outside of the gastrointestinal tract (omentum, mesentery, retroperitoneum or pleura)
        Essential features
        • Most common in stomach
        • Most due to mutations in proto-oncogene KIT (exon 11)
        • 3 histologic types: spindle, epithelioid and mixed
        • Prognosis: depends on tumor size, mitotic rate and site of origin
        • Treatment: surgical excision or imatinib
        Terminology
        • Gastrointestinal stromal tumor (GIST)
        • Historic terms:
          • Gastrointestinal smooth muscle tumor
          • Gastrointestinal autonomic nerve tumor
          • Leiomyoblastoma
          • Smooth muscle tumor of uncertain malignant potential
          • Gastrointestinal pacemaker cell tumor
        ICD coding
        Epidemiology
        Sites
        • Can occur anywhere along the tubular gastrointestinal tract
        • Stomach (60%) > jejunum and ileum (30%) > duodenum (4 - 5%) > rectum (4%) > colon and appendix (1 - 2%) > esophagus (< 1%) (Semin Diagn Pathol 2006;23:70)
        Pathophysiology
        • Activating mutations in the proto-oncogene KIT (~75%) (Science 1998;279:577) or platelet derived growth factor receptor-α (PDGFR-α) (~10%) (Science 2003;299:708)
          • Leads to constitutive phosphorylation of the receptor tyrosine kinase and activation of downstream pathways → cell proliferation and survival
          • KIT and PDGFRA mutations are mutually exclusive in GIST
        • Succinate dehydrogenase deficient GIST (Am J Surg Pathol 2010;34:636)
          • Young adults (before age 40)
          • Female preponderance (> 2:1)
          • Almost exclusively in stomach (predilection for distal stomach and antrum)
          • Pathophysiology:
            • SDH is an enzyme complex in the electron transport chain and Krebs (citric acid) cycle, composed of 4 subunits (SDHA, SDHB, SDHC, SDHD)
            • In the Krebs cycle, SDH catalyzes oxidation of succinate to fumarate
            • Mutations in 1 of the subunits (most commonly SDHA) results in succinate accumulation, increased transcription of HIF1α-regulated genes and decreased DNA methylation
          • Loss of SDHB immunoexpression
        • Recently identified, less well understood mutations
          • NF1 mutation
          • Mutations in the RAS / RAF / MEK pathway (ex. ETV1 transcription factor associated with GIST formation)
        Etiology
        • Unknown at this time
        • Most are sporadic
        • Small percentage are familial
        Clinical features
        • SDH deficient GIST
          • Carney triad: GIST, pulmonary chondroma, paraganglioma
            • Nonhereditary
            • SDHC promoter hypermethylation
            • Small percentage have germline SDH mutations
          • Carney-Stratakis syndrome: GIST and paraganglioma
            • Hereditary, autosomal dominant
            • Germline mutations in SDHB, SDHC or SDHD subunit
        • Neurofibromatosis (NF): 7% of patients with NF1 develop 1 or more GIST, usually in small bowel (Am J Surg Pathol 2005;29:1170, Am J Surg Pathol 2006;30:90, Hum Mol Genet 2006;15:1015)
        • Familial: germline mutations in KIT or PDGFRα autosomal dominant
          • Immunopositive for SDHB
        Diagnosis
        • Patients most commonly present with gastrointestinal bleeding or abdominal pain
        • May be an incidental finding during radiologic or endoscopic workup for other clinical issues
          • Endoscopy show as a subepithelial lesion
        • Definitive diagnosis cannot be made without histologic examination
        Laboratory
        • No significant laboratory findings
        Radiology description
        • Radiologic findings are variable, depending on size and time of presentation
        • CT usually shows a solid, heterogeneous mass (reflecting the presence of hemorrhage or cystic degeneration)
        • Endoscopic ultrasound reveals a hypoechoic solid mass
        Radiology images

        Images hosted on other servers:
        Missing Image Missing Image

        Gastric GIST

        Prognostic factors
        • GIST can have clinically malignant behavior
        • Prognosis depends upon tumor size, mitotic rate and site of origin (Semin Diagn Pathol 2006;23:70)
        • Intraoperative tumor rupture is also associated with poorer prognosis
        • Compete surgical resection: improved local recurrence rate and overall survival
          • Incomplete resection particularly in the area of the rectum, is associated with a higher risk of recurrence
        • 60 - 80% of patients with SDH deficient GIST developed distant metastasis; however, the NIH risk stratification criteria may not be appropriate for this subtype (Am J Surg Pathol 2016;40:1616)
        Gastric GIST: risk of disease progression (Semin Diagn Pathol 2006;23:70)
        Size ≤ 5 mitoses per 50 HPF > 5 mitoses per 50 HPF
         > 10 cm   Moderate   High 
         > 5 to ≤ 10 cm   Low   High 
         > 2 to ≤ 5 cm   Very low   Moderate 
         ≤ 2 cm   No   No 
        Case reports
        Treatment
        • Most GIST are treated with surgical resection
        • Imatinib mesylate (Gleevec): tyrosine kinase inhibitor of KIT and PDGFRα
          • Metastatic / recurrent GIST
        • Sunitinib malate (Sutent): a tyrosine kinase inhibitor of KIT, PDGFRα, VEGFR
          • Imatinib resistant GIST
        • SDH deficient tumors are less responsive to tyrosine kinase inhibitor
        Clinical images

        Images hosted on other servers:
        Missing Image

        Endoscopy

        Missing Image

        Intraoperative mass

        Missing Image

        Gastroscopy

        Gross description
        • Well circumscribed, intramural lesion, centered within the muscularis propria
        • Fleshy, tan-pink cut surfaces, which may show hemorrhage or cystic degeneration
        • Mean size 6 cm (0.4 - 40 cm) (Am J Surg Pathol 2005;29:52)
        Gross images

        Contributed by Riki Turri, PA (ASCP)
        Missing Image

        Stomach GIST, cross section

        Frozen section description
        • Spindle cell neoplasm
        Frozen section images

        Contributed by Phoenix D. Bell, M.S., M.D.
        Missing Image

        Bland spindle cells on frozen section

        Microscopic (histologic) description
        • 3 morphologic types: spindle (70%), epithelioid (20%), mixed (10%) (WHO Classification of Tumours Editorial Board: Digestive System Tumours (Medicine), 5th Edition, 2019)
          • Spindle:
            • Bland spindle cells with faintly eosinophilic cytoplasm in a syncytial pattern; elongated nuclei with inconspicuous nucleoli; artifactual paranuclear vacuoles common in stomach GIST"
            • Subtypes: sclerosing, palisaded, vacuolated, diffuse hypercellular, sarcomatoid features with significant nuclear atypia and mitotic activity
          • Epithelioid:
            • Round cells with clear to eosinophilic cytoplasm in sheets or nests; increased tendency for pleomorphism versus spindle type
            • Subtypes: sclerosing, discohesive, hypercellular, sarcomatous with significant atypia and mitotic activity
          • Mixed:
            • Tumor is composed of cells with spindle and epithelioid morphology
        • SDH deficient: epithelioid or mixed epithelioid / spindle cell morphology, multinodular pattern, minimal nuclear pleomorphism, occasional atypical mitoses
        • Dedifferentiated: anaplastic appearance with an unusual phenotype (may lose expression of KIT or may aberrantly express other markers such as cytokeratin)
        Microscopic (histologic) images

        Contributed by Phoenix D. Bell, M.S., M.D. and Jennifer Findeis-Hosey, M.D.
        Missing Image

        Spindle cell type

        Missing Image

        Stomach GIST (mixed type)

        Missing Image Missing Image

        Evidence of mitotic activity

        Missing Image Missing Image

        Prominent paranuclear vacuoles


        Missing Image Missing Image

        Stomach GIST, epithelioid type

        Missing Image Missing Image

        Stomach GIST, spindle cell type



        Contributed by Andrey Bychkov, M.D., Ph.D.
        Missing Image

        Core biopsy

        Missing Image

        Core biopsy with IHC

        Missing Image

        CD34

        Missing Image

        C-kit / CD117



        Contributed by Raul S. Gonzalez, M.D. (Case #523)

        SDH deficient GIST

        DOG1

        SDHB

        Virtual slides

        Images hosted on other servers:
        Missing Image

        Gastric GIST

        Cytology description
        • Endoscopic ultrasound guided fine needle aspiration may lend a preliminary diagnosis
          • Bland spindle to epithelioid cells arranged in fascicles
        Cytology images

        Contributed by Andrey Bychkov, M.D., Ph.D.
        Missing Image

        Highly cellular

        Missing Image

        Spindled

        Positive stains
        Negative stains
        • S100
        • SDHB (in SDH deficient GIST, loss of retention is an abnormal finding)
        Molecular / cytogenetics description
        Molecular / cytogenetics images

        Images hosted on other servers:
        Missing Image

        c-kit mutations

        Sample pathology report
        • Stomach, fundus, partial gastrectomy:
          • Gastrointestinal stromal tumor (GIST), mixed spindle cell and epithelioid type (see synoptic report)
        Differential diagnosis
        Board review style question #1

          The image above demonstrates a gastric mass in a 20 year old woman. Which of the following mutations is most likely seen in this entity?

        1. KIT
        2. NF1
        3. PDGFRA
        4. SDHA
        5. SDHC
        Board review style answer #1
        D. The image shows cells with round nuclei, abundant eosinophilic cytoplasm, inconspicuous nucleoli and minimal pleomorphism, representative of an epithelioid type GIST. This subtype is commonly seen in patients with an SDH deficient GIST, which are most often due to mutations in the SDHA subunit; however, rare mutations in the other subunits have also been identified. The majority of GIST result from mutations in the KIT (75%) or PDGFR-α (10%) proto-oncogenes. They are usually found in the stomach and are composed of a spindle, versus epithelioid, population. More recently, there have been reports suggesting patients with neurofibromatosis 1 (NF1) mutations are predisposed to GIST development.

        Comment Here

        Reference: Gastrointestinal stromal tumor (GIST)
        Board review style question #2
          A 60 year old man presents with abdominal pain and a CT reveals a mass within the gastrointestinal system, which is resected. Histopathologic examination reveals a submucosal mass composed of spindle cells with lightly eosinophilic cytoplasm arranged in a syncytial pattern. The pathologic diagnosis is a GIST, spindle cell type. Which of the following answer choices correlates with the most favorable prognosis for this patient?

        1. KIT mutation present
        2. Mass 11 cm in greatest dimension
        3. Mass located in the jejunum
        4. Mitotic rate ≥ 5 per 50 HPF
        5. R1 resection
        Board review style answer #2
        A. The main prognostic factors associated with GIST are mitotic rate, tumor size and tumor location. The extent of surgical resection and the presence of a KIT mutation also affect prognosis. GIST findings suggesting a more favorable prognosis include: stomach location, tumor size ≤ 2 cm, mitotic rate < 5 per 50 HPF, R0 (a resection with negative margins) and the presence of a KIT mutation.

        Comment Here

        Reference: Gastrointestinal stromal tumor (GIST)
        Board review style question #3
          Which of the following is true about succinate dehydrogenase deficient gastrointestinal stromal tumors (SDH deficient GISTs)?

        1. They almost always arise in the colon
        2. They are negative for KIT by immunohistochemistry
        3. They have a very poor prognosis
        4. They sometimes metastasize to lymph nodes
        Board review style answer #3
        D. They sometimes metastasize to lymph nodes. SDH deficient GISTs may metastasize to lymph nodes, which almost never happens in GISTs that arise through other molecular pathways. Despite this behavior, they have a relatively good prognosis. SDH deficient GISTs almost always arise in the stomach, not the colon. They are positive for KIT and DOG1 by immunohistochemistry, facilitating the diagnosis of GIST. They have a moderate prognosis, and patients often have a protracted clinical course with metastases to lymph nodes and distant organs.

        Comment Here

        Reference: Gastrointestinal stromal tumor (GIST)

        Glomus tumor
        Definition / general
        • Mesenchymal tumor composed of modified smooth muscle cells; neoplastic counterpart of perivascular glomus bodies
        • Common site is distal extremities; rare in GI tract; most common GI site is stomach
        Clinical features
        • More common in women, median age 55 years (range, 19-90 years, Am J Surg Pathol 2002;26:301)
        • Presents with bleeding (may be life threatening), ulcer symptoms or as incidental finding
        • Much less common than GIST
        • Usually benign, may metastasize to liver and cause death; malignant behavior more likely if > 5 cm, but cannot predict based on histology
        Radiology images

        Case #393

        53 year old man

        Case reports
        Gross description
        • 2-5 cm (range, 1.1 to 7 cm) intramural mass, usually antral
        • Circumscribed, often with overlying mucosal ulceration and multinodular
        Gross images

        Case #393

        53 year old man

        Microscopic (histologic) description
        • Multiple cellular nodules often separated by streaks of gastric smooth muscle
        • Glomus cells are round, sharply demarcated, with cytoplasmic clearing
        • Hyaline and myxoid change often in center of tumor
        • Mildly dilated pericytoma-like vessels
        • Vascular invasion and focal atypia common
        • 1-4 mitotic figures / 50 HPF
        Microscopic (histologic) images

        Case #393




        Contributed by @RaulSGonzalezMD on Twitter
        Glomus tumor Glomus tumor Glomus tumor

        Glomus tumor



        Images hosted on other servers:

        Epithelial cells (left), Calponin+ (right)

        Solid growth pattern

        Glomus cells and tumor

        Positive stains
        Negative stains
        Electron microscopy description
        • Cytoplasm packed with myofilaments with focal condensations
        • Resembles smooth muscle cells
        Differential diagnosis
        • Carcinoid: less prominent cell borders, coarser chromatin, keratin+, chromogranin+, synaptophysin+
        • Epithelioid GIST: pericellular clearing, polygonal and not oval / round, less prominent veins / capillaries
        • Hemangiopericytoma / solitary fibrous tumor: very rare in GI tract, actin-
        • Paraganglioma: zellballen surrounded by sustentacular cells; chromogranin+, synaptophysin+, S100+

        Graft versus host disease
        Definition / general
        Sites
        Pathophysiology
        Etiology
        Clinical features
        Diagnosis
        • Histologic criteria (grading) originally described in 1975 by Lerner et al. includes 4 histological grades for the GI biopsies (Transplant Proc 1974;6:367, Ann Intern Med 1994;120:143):

            Organ Involvement
          Grade Skin Liver Gut
          I Epidermal, basal cell, vacuolar degeneration < 25% small interlobular bile ducts abnormal (degeneration or necrosis) Single cell necrosis of epithelial cells
          II Grade I changes plus "eosinophilic bodies" 25% to 50% bile ducts abnormal Necrosis and loss of glands
          III Grade II changes plus separation of the dermal epidermal junction 50% to 75% bile ducts abnormal Focal microscopic mucosal denudation
          IV Frank epidermal denudation > 75% bile ducts abnormal Diffuse mucosal denudation

        • Currently, NIH consensus criteria incorporate timing of the presentation and clinical findings in addition to histologic criteria:
          • Classic acute GVHD: cases present within 100 days of HCT and display features of acute GVHD; diagnostic and distinctive features of chronic GVHD are absent
          • Persistent, recurrent, late onset acute GVHD: cases present greater than 100 days post-HCT with features of acute GVHD; diagnostic and distinctive features of chronic GVHD are absent
          • Classic chronic GVHD: cases may present at any time post-HCT; diagnostic and distinctive features of chronic GVHD are present and there are no features of acute GVHD
          • Overlap syndrome: cases may present at any time post-HCT with features of both chronic GVHD and acute GVHD; on occasion, this is colloquially referred to as "acute on chronic" GVHD (UpToDate: Clinical Manifestations, Diagnosis and Grading of Acute Graft Versus Host Disease [Accessed 1 August 2018])
        • Diagnosis of chronic GVHD requires the presence of at least one diagnostic clinical sign of chronic GVHD or the presence of at least one distinctive manifestation confirmed by pertinent biopsy or other relevant tests in the same or another organ
        • Pathology report recommendation:
          • All pathology reports should report both histologic features and a final diagnosis
          • Final diagnosis integrates the histopathologic results and the clinical context and is summarized in 1 of 4 categories:
        Endoscopy findings
        Prognostic factors
        Case reports
        Treatment
        • Steroids, mycophenolate mofetil, tacrolimus, cyclosporine, infliximab, daclizumab
        Microscopic (histologic) description
        Microscopic (histologic) images

        Contributed by Heidi D. Lehrke, D.O.

        Grade 3 GVHD, glandular apoptosis and crypt drop out, several apoptotic figures seen in field



        Images hosted on other servers:

        Hepatic GVHD

        Various images

        Flow cytometry description
        Videos

        Chronic graft versus host disease and the gastrointestinal tract

        Differential diagnosis
        Board review style question #1
        What is the differential diagnosis for gastrointestinal GVHD?

        1. Chemotherapy
        2. CMV infection
        3. Cryptosporidiosis
        4. H. pylori infection
        5. Mycophenolate mofetil treatment
        6. NSAIDs
        7. Oral sodium phosphate bowel preparation
        8. All of the above
        Board review style answer #1
        H. All of the above

        Comment Here

        Reference: Graft versus host disease
        Board review style question #2
        What type of cell is generally resistant to GVHD in the GI tract and could be mistaken for an infiltrating adenocarcinoma in a single cell pattern in a biopsy specimen?

        1. Esophageal mucosa squamous cells
        2. Gastric endocrine cells
        3. Gastric glandular cells
        4. Goblet cells
        5. Paneth cells
        6. None of the above
        7. All of the above
        Board review style answer #2
        B. Gastric endocrine cells

        Comment Here

        Reference: Graft versus host disease

        Granulomatous gastritis
        Definition / general
        • Causes:
          • Anisakiasis, common variable immunodeficiency, Crohn’s disease, foreign body, histoplasmosis, idiopathic, post-barium studies, sarcoidosis, tuberculosis, tumors (extravasated mucin), vasculitis
          • Rarely Langerhans cell histiocytosis

          • Idiopathic / isolated:
            • Usually older white men vs. sarcoid (often young black men / women), other causes of granulomatous disease must be excluded
            • Patients may later develop Crohn’s disease or sarcoidosis
            • A descriptive diagnosis such as gastric mucosa with non-necrotizing granulomatous inflammation and a recommendation for followup may be more prudent than a diagnosis of idiopathic granulomatous gastritis

        • Chronic granulomatous disease:
          • May present with distinct or poorly formed granulomas in children with pigment-laded macrophages
          • Associated with outlet obstruction (Am J Surg Pathol 1982;6:673)
        Microscopic (histologic) images

        Contributed by Chungja C. Shim, M.D.

        Gastric biopsy


        Helicobacter heilmannii
        Definition / general
        • Very rare compared to H. pylori (Arch Pathol Lab Med 1995;119:1149)
        • Patients usually symptomatic with dyspepsia, epigastric pain, acid reflux
        • Associated with lymphoid hyperplasia, lymphoma, gastric carcinoma, peptic ulcer disease
        • Less severe and fewer lymphoid aggregates than H. pylori gastritis (Mod Pathol 1999;12:534)
        • More common in children; due to contact with farm animals or household pets
        • Helical, 3.5 - 7.5 microns, 0.9 microns in diameter
        • Prefers gastric antrum
        • H. felis, H. fennelliae, H. cinaedi also associated with human disease
        Treatment
        • Same as H. pylori
        Microscopic (histologic) description
        • Long tight spirals; thicker and twice as long as H. pylori and usually visible with H&E stain
        • Changes similar to but less severe than H. pylori gastritis (lymphoid aggregates, scant neutrophils)
        Microscopic (histologic) images

        Contributed by @liverwei on Twitter
        Helicobacter heilmannii Helicobacter heilmannii Helicobacter heilmannii

        Helicobacter heilmannii

        Positive stains
        • H. pylori (polyclonal antibody cross-reacts, but not monoclonal antibody), Warthin Starry, Steiner, Diff-Quik
        Board review style question #1

        A 16 year old boy presents with dyspepsia and abdominal pain. Endoscopy shows nodular mucosa and histology (shown above) shows gastritis with prominent lymphoid aggregates and organisms on the mucosal surface (inset). What is your diagnosis?

        1. Atrophic gastritis
        2. Autoimmune gastritis
        3. Eosinophilic gastritis
        4. H. heilmannii gastritis
        5. H. pylori gastritis
        Board review style answer #1
        D. H. heilmannii gastritis

        Comment Here

        Reference: Helicobacter heilmannii

        Helicobacter pylori
        Definition / general
        • Helicobacter pylori gastritis is the most frequent and treatable form of gastritis, affecting more than half the world's population
        Essential features
        • Most patients are asymptomatic or have mild self limited dyspeptic symptoms but some present with abdominal pain with or without peptic ulcer disease
        • H. pylori organisms are Gram negative, helical or spiral shaped and flagellate
        • Diagnostic tests include invasive tests such as biopsy urease test, histology or bacterial culture and sensitivity test or noninvasive tests such as urea breath test, stool antigen test and serology
        • Chronic active gastritis with superficial band of lymphoplasmacytic cells with or without neutrophils is the most common histologic pattern of H. pylori gastritis
        • Bismuth quadruple, levofloxacin triple or rifabutin triple therapies are used for treatment
        • Upfront H. pylori testing is not recommended; GIPS recommendations for utility of ancillary stains when evaluating gastric biopsies for H. pylori should be followed (Am J Surg Pathol 2013;37:e12)
        • Complications of H. pylori gastritis include gastroduodenal ulcers, atrophy, intestinal metaplasia, gastric cancer and mucosa associated lymphoid tissue (MALT) lymphoma
        Terminology
        • Superficial gastritis
        • Type B (bacterial) gastritis
        ICD coding
        • ICD-10: B96.81 - Helicobacter pylori (H. pylori) as the cause of diseases classified elsewhere
        Epidemiology
        • More than half the world's population is infected (Gastroenterology 2017;153:420)
        • Africa has the highest pooled prevalence (70.1%); Northern America (37.1%) and Oceania (24.4%) have the lowest prevalence
        • H. pylori prevalence differs by ethnicity in the U.S.:
          • Non-Hispanic whites: 18.4 - 26.2%
          • Nonwhites: 34.5 - 61.6%
          • Native American / Alaskan Native: 75.0%
        • H. pylori prevalence reflects the level of urbanization, sanitation, access to clean water and varied socioeconomic status
        • In children, the most frequent transmission route is from mother to child (Adv Exp Med Biol 2019;1149:107)
        Sites
        Pathophysiology
        • Major virulence factors: CagA (cytotoxin associated gene A) and its pathogenicity island (Cag PAI) and VacA (vacuolating cytotoxin A)
          • CagA and Cag PAI:
            • Translocates to the host cytoplasm and binds to the inner surface of the cell membrane
            • Major effects: disrupts tight junctions, provokes proinflammatory and mitogenic response
          • VacA:
            • Cytochrome c release and activation of proapoptotic factor leading to apoptosis
        • Gastric inflammation is a result of H. pylori infection mediated upregulation of cytokines; most significant are:
          • CagA PAI mediated induction of NFκB and IL8 secretion
          • Reactive oxygen species and reactive nitrogen species produced by host gastric epithelial cells
        • Reference: Cancer Lett 2014;345:196
        Etiology
        • Pathogenic species in humans:
          • Helicobacter pylori, Helicobacter heilmannii
        • Taxonomy:
          • Campylobacterales (order); Helicobacteraceae (family); Helicobacter (genus)
        • Characteristic features:
          • Gram negative
          • Helical or spiral shaped
          • Flagellate
        • 3 physiologic states or forms of H. pylori:
          • Vegetative / proliferative: S shaped, culturable and viable, capable of replication
          • Dormant: C / U shaped, reversible state, not obviously viable but culturable, transformation induced at low pH (< 6)
          • Coccoid: nonviable
        Diagrams / tables

        Contributed by Tanner Storozuk, M.D. and Namrata Setia, M.D.
        Proposed algorithm for diagnosis

        Proposed algorithm for diagnosis

        Clinical features
        • Asymptomatic or mild self limited dyspeptic symptoms with transient hypochlorhydria - acute gastritis
        • Abdominal pain seen with chronic gastritis with or without peptic ulcer disease
        • Extragastrointestinal manifestations: iron deficiency anemia, idiopathic thrombocytopenic purpura, vitamin B12 deficiency
        Diagnosis
        • Diagnostic test of choice based on active bleeding due to suspected peptic ulcer, use of proton pump inhibitor therapy [PPI], antibiotics and bismuth
        • Proposed algorithm (see Diagrams / tables)
        Laboratory
        • Invasive (endoscopic) and noninvasive techniques:
          • Endoscopic diagnosis:
            • Biopsy urease test: sensitivity and specificity, 90% and 95%
            • Histology: sensitivity and specificity, 95% and 98%
            • Bacterial culture and sensitivity test: high specificity, low sensitivity as H. pylori is difficult to culture
          • Noninvasive testing:
            • Urea breath test: sensitivity and specificity, 88 - 95% and 95 - 100%
            • Stool antigen test: sensitivity and specificity, 94% and 97%
            • Serology: sensitivity and specificity, 85% and 79%
        • Eradication confirmation recommended; tests recommended: urea breath test, stool antigen testing or endoscopy based testing
        • Other infrequently used tests:
          • 13C urea assay: serodiagnostic test using a 13C urea assay
          • Polymerase chain reaction: used to detect low bacterial loads and identify specific mutations associated with antimicrobial resistance
        • Reference: UpToDate: Bacteriology and epidemiology of Helicobacter pylori infection [Accessed 17 August 2020]
        Radiology description
        • Not indicated; historically, presence of thickened gastric folds was the best radiographic criterion (Radiology 1995;195:763)
        Prognostic factors
        • ~20% of cases not cured with first course of treatment and need retreatment
        • Complications of H. pylori gastritis: gastroduodenal ulcers, atrophy, intestinal metaplasia, gastric cancer, MALT lymphoma
        Case reports
        • 12 year old girl with refractory iron deficiency anemia and H. pylori associated lymphocytic gastritis (J Pediatr Hematol Oncol 2013;35:321)
        • 14 year old girl with H. pylori gastritis and associated gastric low grade mucosa associated lymphoid lymphoma treated only for the H. pylori infection, resulting in complete resolution of her lymphoproliferative disease (Gastroenterology 1995;109:973)
        • 23 year old man with subacute combined degeneration of the spinal cord, low serum vitamin B12, anti-intrinsic factor antibodies and H. pylori gastritis (BMJ Case Rep 2013;2013:bcr201320038)
        • 66 year old man with reflux esophagitis and Russell body gastritis (Dig Liver Dis 2015;47:526)
        Treatment
        • Bismuth quadruple (PPI, bismuth subcitrate, tetracycline and metronidazole) or levofloxacin triple (PPI, levofloxacin, amoxicillin), rifabutin triple (PPI, rifabutin and amoxicillin) (Am J Gastroenterol 2017;112:212)
        • Decision made based on risk factors for macrolide resistance and the presence of a penicillin allergy
        Clinical images

        Contributed by Tanner Storozuk, M.D. and Namrata Setia, M.D.
        Erosions

        Erosions

        Ulcer

        Ulcer

        Nodular mucosa

        Nodular mucosa

        Gross description
        Microscopic (histologic) description
        • Chronic antral gastritis: infiltration of lamina propria by plasma cells, lymphocytes and a small number of eosinophils seen as a superficial band of inflammation
        • Active chronic antral gastritis: when the above is associated with neutrophils; this should prompt a search for organisms
        • Lymphoid follicles in antral mucosa are common but nonspecific
        • Variation in histologic patterns:
          • May present as corpus predominant gastritis or pangastritis
          • Duodenum may show foveolar metaplasia and duodenitis, with or without H. pylori organisms
          • Use of PPIs facilitates proximal migration of the organisms to the oxyntic mucosa; the organisms also migrate deeper into the oxyntic glands
          • May be associated with hyperplastic polyps; eradication of H. pylori is recommended in patients with hyperplastic polyps (Ann Intern Med 1998;129:712)
          • An acute gastritis - initial transient stage followed by either resolution or onset of chronic gastritis
          • Coccoid morphology of H. pylori is found most often in patients with recent PPI use or eradication therapy; this morphology of H. pylori has a diameter that is one half to one third the length of spiral forms (Am J Clin Pathol 2002;118:719)
        • Sequalae of H. pylori gastritis:
          • Atrophic antral and corpus gastritis
          • H. pylori related gastric ulcers: associated with pangastritis
          • H. pylori related duodenal ulcers: associated with antral gastritis, usually seen in the first part of duodenum, distal to the pylorus
          • Intestinal metaplasia often involving the incisura or the antrum
          • Lymphocytic gastritis
        • Histologic features with regression of H. pylori gastritis:
          • Neutrophils disappear
          • Regression of chronic inflammation over several years
          • Slow decrease in the number of lymphoid follicles
        • Sydney classification (Am J Surg Pathol 1996;20:1161):
          • Not used in routine practice
          • Records five histologic variables: chronic inflammation, activity, atrophy, intestinal metaplasia and H. pylori density; makes recommendations for obtaining optimal biopsy samples
        Microscopic (histologic) images

        Contributed by Tanner Storozuk, M.D. and Namrata Setia, M.D.
        Histology of <i>H. pylori</i> stomach body

        H. pylori in body

        Histology of H. pylori stomach antrum

        H. pylori in antrum

        Histology of <i>H. pylori</i> in duodenum

        H. pylori in duodenum

        Histology of <i>H. pylori</i> (left) versus <i>H. heilmannii</i> (right)

        Histology of H. pylori (left) versus H. heilmannii (right)

        Immunostain for <i>H. pylori</i>

        Immunostain for H. pylori

        Stomach body <i>H. pylori</i> gastritis

        Stomach body H. pylori gastritis

        Virtual slides

        Images hosted on other servers:

        H. pylori antral gastritis

        H. pylori body gastritis

        Positive stains
        • Easily recognized on routine H&E stain, upfront H. pylori immunohistochemistry not recommended (Am J Surg Pathol 2013;37:e12)
        • Immunohistochemistry and special stains: histochemical stains do not distinguish H. pylori from other bacteria and immunohistochemical antibodies against H. pylori cross react with H. heilmanni
          • Immunohistochemistry: most common, associated with low false positive rare, also stains coccoid forms
          • Silver based stains: Warthin-Starry, Steiner, El-Zimaity dual stain, Genta
          • Nonsilver based stains: Diff-Quik, Giemsa, Leung Alcian yellow
        • Gastrointestinal Pathology Society (GIPS) offers recommendations for ancillary stains in detecting H. pylori when evaluating gastric biopsies (Am J Surg Pathol 2013;37:e12)
        Electron microscopy description
        • On scanning electron microscopy, the bacterium shows a helical bacillary morphology with a minor axis / major axis ratio of ≤ 1 / 1.2
        • Coccoid form is spherical with a smooth surface and minor axis / major axis ratio > 1 / 1.2 (Microbiol Immunol 2018;62:221)
        Videos

        Histopathology, H. pylori gastritis

        Basic pathogenesis of H. pylori infection

        Sample pathology report
        • Stomach, upper gastrointestinal biopsies:
          • Active antral / body H. pylori gastritis (see comment)
          • Comment: There is no intestinal metaplasia or dysplasia in these biopsies.
        Differential diagnosis
        Board review style question #1

        A 55 year old man presents with abdominal pain and vomiting; imaging shows pneumoperitoneum and posterior duodenal wall perforation for which an exploratory laparotomy with antrectomy, proximal duodenectomy, Roux-en-Y gastrojejunostomy and truncal vagotomy is performed. The section of the gastric wall showed a dense band of superficial gastritis in the gastric antrum and body mucosa (as shown above) with several spiral shaped organisms in the gastric mucin. Which of the following is true about this gastritis?

        1. A corpus restricted chronic gastritis is a typical presentation of H. pylori gastritis
        2. A focally enhanced chronic active gastritis is a typical biopsy finding in H. pylori gastritis
        3. H. pylori organisms are twice as long and considerably thicker than H. heilmannii organisms
        4. It is associated with a risk of peptic ulcer disease, primary gastric lymphoma and gastric carcinoma
        5. The treatment of H. pylori gastritis is optional as the infection is asymptomatic in the vast majority of infected individuals
        Board review style answer #1
        D. H. pylori infection is associated with complications including gastroduodenal ulcers, atrophy, intestinal metaplasia, gastric cancer and mucosa associated lymphoid tissue (MALT) lymphoma. Choice A: a corpus restricted chronic gastritis is a typical presentation of autoimmune gastritis whereas an antral predominant gastritis is the most common pattern of H. pylori gastritis in the western world. Choice B: a superficial chronic active gastritis with a dense band of lymphoplasmacytic inflammation is typical for H. pylori gastritis. A focally enhanced chronic active gastritis is typical of Crohn's gastritis, especially when the findings are discrete and intense. Choice C: H. heilmannii organisms are twice as long and considerably thicker than H. pylori organisms. Choice E: according to AGA recommendations, all patients with a positive test of active infection with H. pylori should be offered treatment (Am J Gastroenterol 2018;113:1102).

        Comment Here

        Reference: Helicobacter pylori
        Board review style question #2
        In patients with active peptic ulcer bleeding, what is the best method to diagnose H. pylori infection?
        1. Bacterial culture
        2. Breath test
        3. Histology
        4. Serology
        5. Stool Antigen
        Board review style answer #2
        C. Histology

        Comment Here

        Reference: Helicobacter pylori
        Board review style question #3
        H. pylori immunohistochemistry may be used to help in diagnosis of H. pylori gastritis in which of the following scenarios?

        1. Active chronic antral gastritis when no organisms are identified on H&E
        2. For confirmation of H. pylori organisms that are clearly identifiable on H&E
        3. In every case to rule out H. pylori infection in symptomatic patients
        4. To distinguish between H. pylori and H. heilmannii organisms
        5. When the clinician requests to rule out H. pylori despite no other histologic features
        Board review style answer #3
        A. Active chronic antral gastritis when no organisms are identified on H&E

        Comment Here

        Reference: Helicobacter pylori

        Hepatoid
        Definition / general
        • Subtype with hepatocellular differentiation and poor prognosis
        Gross description
        • Nodular or massive growth, extensive venous invasion
        Microscopic (histologic) description
        • Glandular and hepatocellular differentiation
        • Abundant cytoplasmic glycogen, hyaline globules
        • May have tubulopapillary pattern with clear cells
        • May secrete bile
        Positive stains
        • AFP in serum
        • Hepatic paraffin antigen AFP (50%, positive in classic gastric adenocarcinomas also)
        • Polyclonal CEA (canalicular pattern)

        Hereditary diffuse gastric cancer
        Definition / general
        • Autosomal dominant associated cancer syndrome
        • Associated with diffuse gastric and invasive lobular breast cancers
        • Caused by inactivating germline mutations in CDH1 (E-cadherin)
        Essential features
        • Autosomal dominant associated cancer syndrome
        • Caused by inactivating germline mutations in CDH1 (E-cadherin)
        • Associated with poorly cohesive carcinoma (signet ring carcinoma)
        • Precursor lesions include signet ring carcinoma in situ and signet ring cells with pagetoid spread (pTis)
        • Loss of membranous E-cadherin immunohistochemical staining
        • Associated risk of developing lobular breast cancer
        Terminology
        • Older terms include: E-cadherin associated hereditary gastric cancer, familial diffuse gastric cancer, hereditary diffuse gastric adenocarcinoma
        Epidemiology
        Sites
        • No predilection for specific area of stomach
        Pathophysiology
        Etiology
        • CDH1 germline mutation
        Clinical features
        • Variable presentation
        • 40% lifetime risk of developing lobular breast cancer (JAMA Oncol 2015;1:23)
        • Risk of developing gastric cancer by the age of 80 years: ~ 70% in men and ~ 56% in women (JAMA Oncol 2015;1:23)
        Radiology description
        Case reports
        Treatment
        • Prophylactic total gastrectomy
        Gross description
        • Range:
          • No gross lesions identified
          • Linitis plastica
          • Multiple polyps
        • Summary of grossing a total gastrectomy specimen:
          • Open the specimen along the greater curvature
          • Ink margins
          • Record all measurements
          • Pin and allow to fix overnight
          • Record any lesions and distance to margins
          • Current recommendations include submitting the prophylactic gastrectomy specimen in toto (J Med Genet 2015;52:361)
          • Take pictures in order to map out the specimen as to where sections came and where lesions occur
        Gross images

        Images hosted on other servers:

        Total gastrectomy specimen

        Microscopic (histologic) description
        • Signet ring cell carcinoma in situ (pTis)
          • Signet ring cells within basal membrane
        • Signet ring cells with pagetoid spread (pTis)
          • Second row of signet rings cells beneath normal epithelium in a gastric gland within the basal membrane
        • Intramucosal signet ring carcinoma (pT1a)
          • Signet ring cells restricted to the lamina propria
        • Advanced HGDC (> pT1a)
          • Poorly cohesive carcinoma with signet ring cells
        • NB
          • If no foci of signet ring cell carcinoma or in situ component is identified in a prophylactic total gastrectomy specimen , it should not be reported as negative for carcinoma, but as 'no carcinoma found in xx% of mucosa examined' (J Med Genet 2015;52:361)
        Microscopic (histologic) images

        Contributed by Runjan Chetty, M.B.B.Ch., Ph.D. and Altaf Taher, M.B.B.S., M.D.

        Signet ring carcinoma in situ

        Loss of membranous E-cadherin staining




        Contributed by Raul S. Gonzalez, M.D.

        Signet ring carcinoma in situ

        Obvious poorly cohesive carcinoma

        Subtle poorly cohesive carcinoma

        Positive stains
        Negative stains
        Molecular / cytogenetics description
        Sample pathology report
        • Stomach, total gastrectomy:
          • Poorly cohesive carcinoma with signet ring features (see comment and synoptic report)
          • Comment: A poorly cohesive carcinoma with signet ring features is seen arising in a background of signet ring cell carcinoma in situ. The tumor cells are diffusely positive for pankeratin and demonstrate a loss of membranous E-cadherin staining. In the appropriate clinical context, genetic counseling is recommended to exclude hereditary diffuse gastric cancer, which is associated with a germline mutation in CDH1.
        Differential diagnosis
        Board review style question #1
        Which mutation is most commonly associated with hereditary diffuse gastric cancer?

        1. BRCA2
        2. CDH1
        3. NF1
        4. SMARCA4
        Board review style answer #1
        Board review style question #2
        The following morphology is associated with which answer?



        1. Loss of cytoplasmic keratin staining
        2. Loss of membranous E-cadherin staining
        3. Overexpression of CD68
        4. Overexpression of DOG1
        Board review style answer #2
        B. Loss of membranous E-cadherin staining

        Comment Here

        Reference: Hereditary diffuse gastric cancer

        Heterotopic pancreas / pancreatic acinar metaplasia
        Definition / general
        Case reports
        Gross description
        • Nipple-like projection (with duct emptying into gastric lumen), symmetric cone or round mass
        • Cut surface resembles normal pancreas, but may be cystic
        Gross images

        Contributed by Dennis J. Chute, M.D.

        Heterotopic pancreas in annulus

        Microscopic (histologic) description
        • Pancreatic acini and ducts usually present (total heterotopia), ducts often dilated, islets present in 30%
        • Rarely endocrine only (case report at Arch Pathol Lab Med 2002;126:464)
        • May have mucocele-like changes
        • Usually in submucosa or muscularis propria
        Microscopic (histologic) images

        Contributed by Dennis J. Chute, M.D.

        Heterotopic exocrine pancreas in annulus

        Positive stains
        • Lipase, trypsinogen, amylase

        Hyperplastic polyp
        Definition / general
        • The second most common gastric epithelial polyp encountered during endoscopy
        • Usually a solitary polyp, commonly found in gastric antrum
        • Associated with autoimmune type gastritis, Helicobacter pylori gastritis, postsurgery and bile reflux gastritis (Am J Surg Pathol 2001;25:500, Am J Surg Pathol 2010;34:1591)
        • Usually < 1 cm, more than 50% are < 0.5 cm; large polyps up to 12 cm can occur and mimic malignancy
        • Dysplasia is more likely when > 1 cm and > 2.5 cm (Gut Liver 2009;3:271, GE Port J Gastroenterol 2021;28:328)
          • Resembles other GI dysplasia, either low grade or high grade
        Essential features
        • Surface epithelium derived polyp
        • Elongated and architecturally distorted, irregular foveolar epithelium with cystic dilatations
        • Corkscrew appearance can occur
        • Lamina propria shows edema, congestion, variable acute and chronic inflammation
        • Smooth muscle strands can be seen extending from muscularis mucosae towards surface
        • Thick walled vessels may be present towards the base of the polyp
        Terminology
        ICD coding
        • ICD-10: K31.7 - polyp of stomach and duodenum
        Epidemiology
        Sites
        • Common site is gastric antrum but can occur anywhere in the stomach; recently body of the stomach has become more common due to autoimmune gastritis and the increased use of proton pump inhibitors (Am J Gastroenterol 2009;104:1524, Int J Mol Sci 2021;22:6548)
        • Hyperplastic polyps in pyloric and prepyloric regions may have features of prolapse associated polyps
        • Hyperplastic polyps at gastroesophageal junction are associated with Barrett esophagus in 33% of cases (Am J Surg Pathol 2011;35:1038)
        Pathophysiology
        Etiology
        Clinical features
        • Mean age: 65 years
        • In the last decade, patients aged 45 - 59 had twice as many gastric polyps as 10 years ago (World J Gastroenterol 2016;22:8883)
        • Dyspepsia, heartburn and abdominal pain; patients may have anemia due to upper gastrointestinal bleeding (Dig Dis Sci 2007;52:105)
        • Obstructive symptoms if near pylorus or gastroesophageal junction
        Diagnosis
        • Histopathology
        Case reports
        Treatment
        Gross description
        • Broad based polyp with smooth lobulated contour (Surg Pathol Clin 2020;13:431)
        • Surface erosion may occur in large polyps
        • Should be submitted entirely to rule out dysplasia or malignancy
        Microscopic (histologic) description
        • Elongated, architecturally distorted, irregular pits with outpouchings, cystic dilation and papillary formation
        • Foveolar epithelium with apical neutral mucin cap
          • Can show overt hypertrophic features with the formation of clustered pseudogoblet cells or even pseudo-signet ring cells, especially in damaged areas
        • Edematous lamina propria with lymphocytes, plasma cells and eosinophils
          • Erosion areas can also show neutrophils
        • Helicobacter pylori associated polyps: more likely show lymphoplasmacytic inflammation in lamina propria and neutrophils in the epithelium (Int J Surg Pathol 2016;24:704)
        • Smooth muscle strands can extend from muscularis mucosae to the surface
        • Thick walled blood vessels can be seen at the base
        • Intestinal metaplasia can be seen
        • Regenerative epithelial changes with focal mucin depletion, prominent nucleoli and hyperchromasia
        • Ulcerated areas can display increased mitotic activity in epithelium and granulation tissue with reactive fibroblasts and endothelial cells
        • Dysplasia is rare (4%); may be intestinal, foveolar or mixed (Histopathology 2021;78:106)
          • High grade dysplasia: complex architecture with cribriform formation or budding, back to back arrangement of glands
        Microscopic (histologic) images

        Contributed by Monica T. Garcia-Buitrago, M.D., Omar Aljuboori, M.B.B.S. and Andrey Bychkov, M.D., Ph.D.
        Dilated gastric pits

        Dilated gastric pits

        Tortuous gastric pits

        Tortuous gastric pits

        Edematous lamina propria

        Edematous lamina propria

        Pseudogoblet cells Pseudogoblet cells

        Pseudogoblet cells

        Dysplasia

        Dysplasia


        Dysplastic polyp

        Dysplastic polyp

        Dysplasia

        Dysplasia

        Adenocarcinoma

        Adenocarcinoma

        Polypoid lesion

        Polypoid lesion

        Hemorrhagic and ulcerated

        Hemorrhagic and ulcerated

        Cystic hyperplastic glands

        Cystic hyperplastic glands

        Molecular / cytogenetics description
        Sample pathology report
        • Stomach, esophagogastroduodenoscopy:
          • Gastric hyperplastic polyp (see comment)
          • Comment: Large gastric hyperplastic polyps typically arise in a background of damaged mucosa and are frequently associated with Helicobacter pylori or autoimmune gastritis. Follow up sampling of the flat mucosa from both the antrum and body / fundus in such patients may provide information on the setting in which the polyp has arisen.
        Differential diagnosis
        • Polypoid gastritis:
          • Normal architecture, no smooth muscle wisps
        • Polypoid foveolar hyperplasia:
          • No cystic dilation, no smooth muscle wisps
        • Gastritis cystica profunda:
          • Entrapped, distorted, dilated glands in muscularis
        • Fundic gland polyp:
          • Distorted glands and cysts lined by parietal and chief cells
        • Ménétrier disease:
          • Affects gastric body only; foveolar hyperplasia, atrophy of glands, adjacent epithelium normal
        • Cronkhite-Canada syndrome:
          • Affects entire stomach; foveolar hyperplasia, atrophy of glands, adjacent epithelium shows atrophy of glands and microcystic change
        • Juvenile polyposis:
          • Adjacent mucosa is normal as opposed to hyperplastic polyp, which often has chronic gastritis
        • Peutz-Jeghers polyposis:
          • Autosomal dominant inherited disorder with multiple hamartomatous polyps and mucocutaneous hyperpigmentation
        • Pyloric adenoma:
          • Closely packed tubular glands lined by cuboidal to low columnar epithelium with ground glass cytoplasm, bland nuclei and no well formed apical mucin cap
        • Inflammatory fibroid polyp:
          • Submucosal lesion composed of spindle and stellate stromal cells
        • Dysplasia must be differentiated from regenerative change:
          • Dysplasia is present at the surface; regenerative change shows surface maturation and atypia like changes only in the proliferative zones, not at the surface
          • Dysplasia is typically abrupt
          • Dysplasia has architecturally complex patterns (cribriform, budding and branching)
          • Dysplasia has nuclear pleomorphism, elongation and hyperchromasia without prominent nucleoli
          • Regenerative epithelium shows gradual change, lacks complex patterns, has hyperchromasia but with ovoid nuclei and prominent nucleoli
        Board review style question #1
        Which Helicobacter pylori protein is associated with a role in the development of gastric neoplasia?

        1. CagA
        2. PAR1
        3. SHP2
        4. SRC
        Board review style answer #1
        A. CagA. CagA in Helicobacter pylori can cause aberrant activation of SHP2, a human oncoprotein, in the stomach (Proc Natl Acad Sci USA 2008;105:1003). Answer B is incorrect because PAR1 is a human kinase inhibited by CagA. Answer C is incorrect because SHP2 is a human oncoprotein, not a Helicobacter pylori protein. Answer D is incorrect because SRC is a human tyrosine kinase that mediates CagA phosphorylation and cytoskeletal rearrangements.

        Comment Here

        Reference: Hyperplastic polyp
        Board review style question #2

        Which of the following polyps is seen in this picture?

        1. Fundic gland polyp
        2. Hyperplastic polyp
        3. Polypoid foveolar hyperplasia
        4. Polypoid gastritis
        Board review style answer #2
        B. Hyperplastic polyp. Gastric polyp showing elongated, tortuous and dilated gastric pits and edematous lamina propria (Surg Pathol Clin 2020;13:431). Answer A is incorrect because the polyp shows dilated pits lined by foveolar epithelium, not oxyntic epithelium. Answer C is incorrect because the polyp shows cystically dilated pits. Answer D is incorrect because the polyp has abnormal architecture and does not show predominant plasmacytic and neutrophilic inflammation.

        Comment Here

        Reference: Hyperplastic polyp

        IBD-related gastritis
        Crohn's disease
        • Clinical involvement rare, microscopic disease is more common
        • Gastric granulomas in 9-15%
        • High proportion of Crohn's patients show upper GI involvement,usually antral (Virchows Arch 1998;432:49)
        • Focal acute inflammation in a background of non-inflamed, H. pylori negative mucosa is suggestive of Crohn's disease (Am J Surg Pathol 1998;22:383, Gut 1996;38:379)
        • Active inflammation in 56% (some inflammatory changes in up to 70%), but only 10% are H. pylori positive
        Ulcerative colitis
        Microscopic (histologic) description
        • Most common upper GI inflammatory pattern is focal gastritis, followed by gastric basal mixed inflammation and superficial plasmacytosis (Am J Surg Pathol 2010;34:1672)
        • Variable changes in ulcerative colitis include multiple tiny shallow ulcers, crypt abscess formation, increased intraepithelial lymphocytes, villous blunting; microscopic findings parallel remissions and relapses of colonic inflammation
        Microscopic (histologic) images

        Images hosted on other servers:

        Focal lesion in
        atral mucosa
        resembling active
        Crohn's disease

        Differential diagnosis
        • Granulomas of Crohn's disease:
          • Foreign-body
          • Fungus
          • Granulomatous gastritis
          • Sarcoid
          • Tuberculosis
          • Tumor
          • Whipple’s disease

        Inflammatory fibroid polyp
        Definition / general
        • Benign neoplastic polyp that occurs anywhere in gastrointestinal tract but most commonly in stomach and small intestine
        Essential features
        • Benign mesenchymal gastrointestinal polyp that arises in the submucosa
        • Associated with PDGFRA mutation
        Terminology
        Sites
        • Usually arises in pylorus or distal antrum in stomach
        Etiology
        Clinical features
        • Peak incidence in sixth or seventh decade of life
        • No endoscopic surveillance is required after the histological diagnosis is confirmed
        • Rarely associated with adenocarcinoma or adenoma (Arch Pathol Lab Med 1988;112:829)
        Case reports
        • 79 year old woman with chronic gastritis and a 2 cm sessile gastric polyp (Case #421)
        Treatment
        • Local excision
        Gross description
        • Typically solitary, small and sessile
        • Median size 1.5 cm but can measure up to 9 cm
        Microscopic (histologic) description
        • Submucosal lesion composed of spindle and stellate stromal cells
        • Loose edematous stroma containing thin walled blood vessels with characteristic onion skin arrangement of spindled cells around vessels
        • Inflammatory infiltrate rich in eosinophils
        • Can infiltrate lamina propria
        • Minimal mitotic activity
        Microscopic (histologic) images

        Contributed by Naziheh Assarzadegan, M.D.

        Submucosal mass

        Spindle and stellate stromal cells

        Eosinophils



        Case #421

        Polypoid mass expanding the submucosa

        Mixed inflammatory infiltrate in fibromyxoid stroma, rich in eosinophils

        Loose, fibromyxoid spindled stroma

        CD34+ stromal cells with onion skinning

        Stroma is CD117-,
        with scattered
        CD117+ mast cells

        Positive stains
        Negative stains
        Molecular / cytogenetics description
        • Associated with activating mutation in the platelet derived growth factor receptor alpha (PDGFRA) gene, supporting a neoplastic origin (Histopathology 2012;61:59)
        Sample pathology report
        • Stomach, antrum, polypectomy:
          • Inflammatory fibroid polyp (3.1 cm), focally extending to deep margin (see comment)
          • Lateral margins unremarkable.
          • Comment: An immunohistochemical stain for CD34 is positive.
        Differential diagnosis
        Board review style question #1

        Which of the following statements is true about gastric inflammatory fibroid polyp?

        1. Associated with activating mutation in the platelet derived growth factor receptor alpha gene
        2. Benign and nonneoplastic
        3. Synonymous with inflammatory polyp
        4. Typically centered in the muscularis propria
        Board review style answer #1
        A. They are associated with activating mutation in the platelet derived growth factor receptor alpha gene

        Comment Here

        Reference: Inflammatory fibroid polyp

        Intestinal metaplasia
        Definition / general
        • Gastric intestinal metaplasia (IM) is a metaplastic condition wherein the normal gastric mucosa is replaced by intestinal type of mucosa replete with mucin producing goblet cells, with or without Paneth cells and absorptive cells
        • Diagnosis of intestinal metaplasia in stomach is based on the presence of goblet cells
        • Gastric intestinal metaplasia is a known high risk lesion for gastric carcinoma (GC)
        Essential features
        • In gastric intestinal metaplasia, native gastric mucosa is replaced with intestinal type of mucosa replete with goblet cells, with or without Paneth and absorptive cells
        • It is considered to be a precancerous lesion and is associated with gastric carcinoma
        • Prevalence of intestinal metaplasia is 25% according to a meta analysis; annually, 0.1 - 10% of gastric intestinal metaplasia can progress to gastric carcinoma
        • Commonly occurs as a response to chronic H. pylori infection, bile acid reflux, smoking or high salt intake
        • 2 types of gastric intestinal metaplasia are known: complete and incomplete
          • Incomplete type is associated with gastric carcinoma
        • Operative link for gastric intestinal metaplasia (OLGA / OLGIM) stages III / IV are considered high risk factors for gastric carcinoma
        Terminology
        • Gastric intestinal metaplasia (GIM)
        ICD coding
        • ICD-10:
          • K31.A - gastric intestinal metaplasia
            • K31.A0 - gastric intestinal metaplasia, unspecified
            • K31.A1 - gastric intestinal metaplasia without dysplasia
            • K31.A2 - gastric intestinal metaplasia with dysplasia
        Epidemiology
        • Worldwide prevalence of gastric intestinal metaplasia is 25% according to a meta analysis; ranging from 24 - 84% in Eastern countries to 7 - 25% in Western countries (JGH Open 2020;4:569)
        • Incidence increases with older age, male gender, Helicobacter pylori infection, family history of gastric cancer, bile reflux, smoking, high salt intake and a diet low in fruits, vegetables and vitamin C (Aliment Pharmacol Ther 2002;16:1209, Nat Rev Cancer 2017;17:594)
        • Annual rate of progression of intestinal metaplasia to gastric carcinoma varies from 0 - 10% (Helicobacter 2007;12:22)
        • When associated with high grade dysplasia, progression to gastric carcinoma is reported in 60 - 85% of cases within 4 - 48 months; with low grade dysplasia (LGD), the risk is much lower (Gastroenterology 2008;134:945)
        Sites
        • All parts of the stomach can be affected; but it is most frequently observed in antrum and along the lesser curvature of stomach
        Pathophysiology
        • Intestinal metaplasia results from a distinct interplay between host immune susceptibility, genetic response and environmental exposure; chronic mucosal inflammation leads to gastric atrophy, which leads to replacement of gastric mucosa with intestinal type epithelium
        • Recent animal models have shown that spasmolytic polypeptide expressing metaplasia (SPEM) is a precursor of intestinal metaplasia (Gastroenterol Hepatol (N Y) 2018;14:92)
        Etiology
        • Helicobacter pylori infection, bile reflux, smoking, high salt intake
        • Incidence increases with age > 50 years, male gender, positive family history of gastric cancer, immigrants from H. pylori endemic areas
        Diagrams / tables

        Contributed by Supriya Srivastava, M.D., Ph.D.

        Correa pathway

        Updated Sydney protocol

        OLGA / OLGIM staging

        Clinical features
        • Gastric intestinal metaplasia is usually asymptomatic; may present with acid reflux or heartburn due to underlying conditions like chronic acid reflux and H. pylori infection
        • Associated with autoimmune gastritis and pernicious anemia
        Diagnosis
        • Gold standard for diagnosis of gastric intestinal metaplasia is histology
          • According to updated Sydney protocol, endoscopic targeted biopsies were recommended initially for grading and recently for staging of intestinal metaplasia
            • It recommends at least 5 biopsies: 2 from antrum (1 each from lesser and greater curve, 3 cm proximal to the pylorus), 2 from body (lesser curve, 4 cm proximal to the incisura and midpoint of the greater curve) and 1 from the incisura (Am J Surg Pathol 1996;20:1161)
        • Japan, South Korea and Singapore are a few of the countries that have endoscopic screening programs for early detection of precancerous lesions and gastric carcinoma (Gut 2022;71:854, Dig Endosc 2022;34:412)
        Laboratory
        Radiology description
        • Endoscopic findings: white light endoscopy shows grayish white, slightly elevated plaques surrounded by patchy pink and pale areas
        • Image enhanced magnification endoscopy: groove type pattern for gastric body, light blue crest and marginal blue band for gastric antrum
        • Narrow band imaging: tubular or tubulovillous glands with light blue crests
        • Chromoendoscopy with methylene blue: blue colored irregular marks, blue round and tubular pits or blue villi or blue small pits
        • Reference: Cancers (Basel) 2021;13:6242
        Prognostic factors
        • The following show high association with gastric carcinoma and warrant endoscopic surveillance (Endoscopy 2019;51:365):
          • Incomplete (type III) type intestinal metaplasia
          • OLGA / OLGIM stages III / IV
          • Extensive intestinal metaplasia (intestinal metaplasia involving both antrum and body)
          • Persistent H. pylori infection
          • First degree family history of gastric cancer
          • Immigrants from areas of high incidence of H. pylori
          • Age > 50 years
          • Smoking
        Case reports
        Treatment
        • Currently no definitive treatment for gastric intestinal metaplasia
        • American Association of Gastroenterologists (AGA 2020) recommends treatment and eradication of H. pylori to manage intestinal metaplasia; H. pylori eradication cannot reverse gastric intestinal metaplasia but it may slow down the progression to gastric carcinoma (Gastroenterology 2020;158:693)
        • AGA further recommends endoscopic surveillance every 3 - 5 years in high risk gastric intestinal metaplasia (i.e., incomplete intestinal metaplasia, extensive intestinal metaplasia, family history of gastric carcinoma, racial / ethnic minorities and immigrants from high incidence areas)
        • British Society of Gastroenterologists (BSG 2019) recommends endoscopic surveillance every 3 years for extensive intestinal metaplasia or intestinal metaplasia limited to antrum with a positive family history or persistent H. pylori infection, annually if low grade dysplasia or every 6 months if persistent high grade dysplasia (HGD) with no plan to treat (Gut Liver 2019;13:596)
        • European Society of Gastrointestinal Endoscopy (ESGE 2019) recommends endoscopic surveillance as follows (Endoscopy 2019;51:365):
          • Every 1 - 2 years with positive family history of gastric carcinoma plus either extensive intestinal metaplasia or OLGA / OLGIM stages III / IV
          • Every 3 years if extensive intestinal metaplasia or OLGA / OLGIM stages III / IV
          • Consider every 3 years for focal intestinal metaplasia plus family history of gastric carcinoma, incomplete intestinal metaplasia or persistent H. pylori gastritis
        • According to the latest Academy of Medicine, Singapore Clinical guidelines (2022), endoscopic surveillance is recommended every 3 years with OLGA / OLGIM stages III / IV and every 2 years with more than 2 risk factors, such as smoking, age > 50 years, incomplete intestinal metaplasia, persistent H. pylori infection, first degree family history of gastric cancer (Ann Acad Med Singap 2022;51:417)
        • COX2 inhibitors are not recommended
        • Use of dietary supplement with antioxidants is not supported as a therapy
        • Endoscopic submucosal dissection (ESD) or endoscopic mucosal resection (EMR) are recommended for gastric intestinal metaplasia associated with visible dysplasia or early gastric carcinoma
        Clinical images

        Images hosted on other servers:
        Missing Image Missing Image

        High resolution white light endoscopy

        Missing Image

        Whitish mucosal patches

        Gross description
        • Patchy erythema or slightly elevated, grayish white plaque
        Microscopic (histologic) description
        • Intestinal metaplasia can be identified by the replacement of gastric columnar mucosa with intestinal type of mucosa comprised of goblet cells, with or without Paneth cells and absorptive brush border
        • Usually occurs in a background of chronic gastritis or H. pylori gastritis
        • It can be classified as complete (type I) and incomplete (II and III) subtypes
        • Complete (type I) intestinal metaplasia (Cancer Res 1999;59:1003):
          • Resembles small intestinal mucosa with straight crypt architecture, well formed goblet cells, Paneth cells and absorptive brush border
          • Goblet cells secrete sialomucins or occasionally sulphomucins
          • It has been reported that the complete type of intestinal metaplasia shows weak or no expression of MUC1 and MUC5AC and the absorptive brush border express the enzyme sucrase, trehalase, maltase and intestinal type alkaline phosphatase
        • Incomplete (type II and III):
          • Histologically resembles either small or large intestinal mucosa and shows irregular architecture, variable size goblet cells and columnar mucosa in various stages of differentiation; Paneth cells and absorptive brush border may or may not be seen
          • In type II intestinal metaplasia, goblet cells secrete sialomucins while the columnar mucosa may secrete sialomucins or neutral mucins
          • In type III intestinal metaplasia, sulphomucins predominate in the columnar mucosa while sialo or sulphomucins predominate in goblet cells
          • Incomplete intestinal metaplasia shows overexpression of MUC1 and MUC5AC and secretes sucrase, maltase but not trehalase and alkaline phosphatase
          • Type III intestinal metaplasia shows highest association with gastric cancer
        • Intestinal metaplasia is graded as mild (< 30%), moderate (31 - 60%) and marked (61 - 100%) according to the updated Sydney classification
        • Intestinal metaplasia is staged using the OLGA / OLGIM system; it is based on the grades of intestinal metaplasia in the antrum (including incisura) and body
        • In autoimmune gastritis, extensive (marked) gastric intestinal metaplasia is seen restricted to the body (oxyntic mucosa) in a background of moderate to marked chronic inflammation, loss of oxyntic glands and accompanied with SPEM or pseudopyloric metaplasia (presence of pseudopyloric or antral glands in body of stomach) (World J Gastroenterol 2015;21:12179)
        Microscopic (histologic) images

        Contributed by Supriya Srivastava, M.D., Ph.D.

        Goblet cells and H. pylori

        Intestinal metaplasia grading

        PAS / Alcian blue and HID / Alcian blue

        MUC2

        CDX2

        Cytology description
        Cytology images

        Images hosted on other servers:
        Missing Image

        Areas of intestinal metaplasia of the stomach

        Positive stains
        • Alcian blue (AB) at pH 2.5: diagnostic stain but not routinely needed as gastric intestinal metaplasia can be readily identified in H&E; AB stains goblet cells bright blue due to the presence of sialomucins
        • Periodic acid-Schiff Alcian blue (PAS AB): stains the acidic or sialomucins as purple and neutral mucins as magenta
        • High iron diamine Alcian blue (HID AB): stains the acidic or sialomucins as blue and sulfated mucin as brownish black in color; not routinely performed due to toxic and carcinogenic reagents
        • MUC2 IHC: de novo expression in the cytoplasm of goblet cells (Virchows Arch 2002;440:311)
        • CDX2: nuclear positivity of goblet cells and columnar mucosa (Gut Liver 2012;6:71)
        • Intestinal markers like CD10, villin, Das1 are variably positive in gastric intestinal metaplasia
        Negative stains
        Electron microscopy description
        • Microvilli border with interspersed goblet cells
        Molecular / cytogenetics description
        • Compared to gastric carcinoma, gastric intestinal metaplasia exhibits lower mutational burden, recurrent mutation in FBXW7 gene, chromosome 8q amplification, focal DNA hypermethylation and shortened telomeres (Cancer Cell 2018;33:137)
        • CDX2 / CDX1, TFF3, villin, OCT1 and PDX1 genes are upregulated in intestinal metaplasia (J Gastroenterol Hepatol 2009;24:193)
        Sample pathology report
        • Stomach, biopsy:
          • Gastric mucosa shows chronic gastritis with mild / moderate / marked intestinal metaplasia (see comment)
          • Comment: No evidence of H. pylori, dysplasia or malignancy.
        Differential diagnosis
        • Pseudogoblet cells:
          • These are columnar cells that are goblet or barrel shaped and arranged in a continuous linear fashion
          • On the contrary, true goblet cells are rounded in shape and are dispersed singly
          • AB stain can be used to distinguish pseudogoblet cells (bluish hue may be present sometimes) and true goblet cells (distinct blue colored goblet cells)
        • Barrett esophagus (BE)
          • Preneoplastic condition observed in the lower one - third of the esophagus
          • Metaplastic condition where the squamous epithelium of the distal esophagus is replaced by the intestinal epithelium with goblet cells
          • Histologically, both gastric intestinal metaplasia and Barrett esophagus appear similar, the difference being the location (stomach versus distal esophagus)
          • While gastric intestinal metaplasia occurs in a background of H. pylori commonly, Barrett esophagus has no causal association with H. pylori
          • Incomplete type of intestinal metaplasia is more commonly observed in Barrett esophagus as compared to gastric intestinal metaplasia (Mod Pathol 2004;17:62)
          • Predisposes to dysplasia and esophageal adenocarcinoma
        Board review style question #1

        A 65 year old man presented to clinic with complaints of dyspepsia and acid reflux. An endoscopic biopsy was done and the sample was sent for histopathological examination. Based on the histopathological image above, what is the diagnosis?

        1. Chronic gastritis only
        2. H. pylori gastritis with intestinal metaplasia
        3. Intestinal metaplasia
        4. Reactive gastropathy
        Board review style answer #1
        B. H. pylori gastritis with intestinal metaplasia. Intestinal metaplasia most commonly occurs in the background of H. pylori infection. Intestinal metaplasia can be recognized readily by the presence of well formed goblet cells.

        Comment Here

        Reference: Intestinal metaplasia
        Board review style question #2
        Which of the following is a high risk factor for gastric intestinal metaplasia?

        1. Age < 50 years
        2. Complete intestinal metaplasia
        3. Extensive intestinal metaplasia
        4. Mild / focal intestinal metaplasia
        Board review style answer #2
        C. Extensive intestinal metaplasia is one of the high risk factors for gastric intestinal metaplasia. It refers to intestinal metaplasia present in both antrum and body.

        Comment Here

        Reference: Intestinal metaplasia

        Intestinal type adenocarcinoma
        Definition / general
        • Tubular and papillary histological subtypes of gastric adenocarcinoma were historically grouped together under the term intestinal adenocarcinoma
        • Tubular adenocarcinoma is composed of neoplastic ducts of varying size and represents the most common subtype of this cancer
        • Papillary adenocarcinoma comprises finger-like projections of well differentiated epithelial cells arranged around fibrovascular cores
        Essential features
        • Intestinal type (tubular and papillary) adenocarcinoma has a more favorable prognosis than other histological subtypes of gastric carcinoma
        • Commonly associated with atrophic gastritis and gastric epithelial dysplasia
        • May express HER2 or PDL1, both of which offer additional treatment options
        Terminology
        ICD coding
        • ICD-O: 8144/3 - stomach / adenocarcinoma, intestinal type
        • ICD-10: C16 - malignant neoplasm of stomach
        • ICD-11: 2B72 - malignant neoplasms of stomach
        Epidemiology
        Sites
        • In the highest incidence areas (see Epidemiology), the most common site is the distal stomach
        • In northern Europe and the U.S., about half of the cases are located in the gastric cardia or fundus (Curr Gastroenterol Rep 2017;19:36)
        • Metastases commonly occur in lymph nodes, liver, lungs, bones, peritoneum and less commonly in the ovary (Krukenberg tumor is a term used for bilateral ovarian involvement)
        Pathophysiology
        • Gastric atrophy caused by Helicobacter pylori associated gastritis or autoimmune gastritis leads to gastric hypochlorhydria and then to bacterial overgrowth in the stomach; this results in the generation of carcinogenic agents within the stomach, leading to neoplastic transformation within the gastric epithelium
        • An increasing number of genetic and epigenetic alterations are implicated in the pathogenesis of gastric adenocarcinoma
        • In the literature, 4 molecular subtypes of gastric adenocarcinoma have been described, each with distinct genomic features
          1. EBV positive: DNA hypermethylation, PIK3CA mutations, amplification of PDL1, PDL2 and JAK2
          2. Microsatellite unstable: high mutation rates including of mismatch repair genes, such as MLH1
          3. Genomically stable (associated with diffuse histology): inactivating mutations in the E-cadherin gene (CDH1) and mutations / fusions of RHOA
          4. Chromosomal instability (associated with intestinal histology): TP53 mutation and amplification of receptor tyrosine kinases (Nature 2014;513:202)
        • Tumor spread occurs via direct invasion of adjacent structures, lymphatic or hematogenous spread or serosal dissemination
        Etiology
        • Risk factors include Helicobacter associated gastritis and autoimmune gastritis as well as cigarette smoking, working in the rubber manufacturing industry, Xray radiation and gamma radiation
        • There is equivocal evidence for additional risk factors including EBV infection, dietary factors, asbestos, N nitroso and lead compounds
        • Familial / hereditary tubular and papillary adenocarcinoma form a minority of cases and, unlike hereditary diffuse gastric cancer which is causally linked to CDH1 gene mutations, lack genetic testing guidelines when not associated with specific hereditary cancer susceptibility syndromes (Transl Gastroenterol Hepatol 2019;4:55)
        • Notably, gastric adenocarcinoma and proximal polyposis of the stomach (GAPPS) is a recently described rare autosomal dominant familial gastric cancer (Best Pract Res Clin Gastroenterol 2021;50-51:101728)
        Clinical features
        • Symptoms include weight loss, abdominal pain, dyspepsia, early satiety, vomiting and in more proximal cancers, dysphagia
        • May be asymptomatic (especially when at an early stage) or present with anemia only
        • Signs of advanced disease on examination include cachexia, a palpable abdominal mass, an enlarged left supraclavicular node (Virchow node), left axillary node (Irish node) or periumbilical node (Sister Mary Joseph nodule)
        • Common complications include upper gastrointestinal bleeds and gastric outlet obstruction
        Diagnosis
        • Upper gastrointestinal endoscopy commonly reveals a tumor mass or malignant ulcer within the stomach and allows biopsy for histological diagnosis, classification and evaluation of molecular biomarkers
        • CT scan of the abdomen may reveal a tumor mass or malignant ulcer within the stomach; this should prompt endoscopy and biopsy
        • CT scan of the chest, abdomen and pelvis is also used to detect metastatic spread and guide staging (see Radiology description)
        Laboratory
        • Histological examination is the key test for confirming the diagnosis
        • Full blood count may show an iron deficiency anemia
        • Biochemistry to assess renal and liver function may guide appropriate treatment options
        Radiology description
        • CT scanning and endoscopic ultrasonography (EUS) are important for radiological staging of the tumor (e.g., determining the operability of the primary tumor and looking for distant metastases)
        • Additional investigations that can be considered to aid the detection of metastatic disease that may be missed by CT scanning, include positron emission tomography (PET) scans or a staging laparoscopy with peritoneal cytology (see Cytology description)
        • See Diagnosis for a description of the role of radiology in the diagnosis of the primary lesion
        Radiology images

        Images hosted on other servers:
        Ulcerating mass and lymphadenopathy

        Ulcerating mass and lymphadenopathy

        Prognostic factors
        • Tumor stage is the most important prognostic factor, especially nodal status; higher stages are associated with worse prognosis (Br J Cancer 2001;84:1602)
        • Site of tumor: proximal gastric cancers have a worse prognosis than more distal cancers (Ann Surg 2000;232:353)
        • Ulcerated tumors have a worse prognosis than polypoid (Pathol Oncol Res 2016;22:197)
        • Histological subtype of adenocarcinoma: intestinal type has a favorable prognosis over other subtypes (J Gastrointest Oncol 2017;8:1026)
        • High grade / poorly differentiated tumors have a worse prognosis (Cancer 2000;89:1418)
        • Presence of perineural, lymphatic and vascular invasion is associated with a worse prognosis
        • Response to neoadjuvant treatment is associated with a better prognosis
        Case reports
        • 49 year old woman with gastric tubular adenocarcinoma and SMAD4 germline pathogenic variant related juvenile polyposis syndrome (Am J Case Rep 2021;22:e932241)
        • 61 year old man with cholangiocarcinoma combined with moderately differentiated gastric adenocarcinoma (Medicine (Baltimore) 2019;98:e16332)
        • 65 year old man with synchronous gastric adenocarcinoma and perigastric lymph node metastatic squamous cell carcinoma with unknown primary (J Minim Invasive Surg 2020;23:99)
        • 84 year old man with gastric adenocarcinoma of fundic gland (chief cell predominant type) coexisting with well differentiated intestinal adenocarcinoma (Medicine (Baltimore) 2021;100:e25861)
        Treatment
        • Tumors identified at a very early stage (i.e., pTis or pT1) may be amenable to local excision at upper gastrointestinal endoscopy
        • Partial or complete gastrectomy may be required for tumors that are more locally advanced; this would almost always be associated with a regional lymphadenectomy
        • Chemotherapy (with or without radiotherapy) may be given in an adjuvant or neoadjuvant setting for tumors assessed as at high risk of progressing with distant metastasis; this may include trastuzumab (Herceptin) for tumors shown to be HER2 positive on immunohistochemistry with or without in situ hybridization
        • Immunotherapy has more recently become a treatment option (e.g., for tumors that are found to express PDL1 at above a treatment threshold level on immunohistochemistry) (Curr Oncol 2022;29:1559)
        Clinical images

        Images hosted on other servers:
        Gastric cancer at endoscopy

        Gastric cancer at endoscopy

        Gross description
        • Tumors can vary in size from barely visible on macroscopic examination to several centimeters across
        • Typical appearance would be as an ulcer with raised everted edges
        • Alternative appearances would include an exophytic mass that may be polypoid in nature or an area of gastric wall thickening / stricturing
        Gross images

        Contributed by @Andrew_Fltv on Twitter
        Contributed by @Andrew_Fltv on Twitter (see original post here)"> Intestinal type adenocarcinomaContributed by @Andrew_Fltv on Twitter (see original post here)"> Intestinal type adenocarcinoma

        Intestinal type adenocarcinoma



        Images hosted on other servers:
        Gastric cancer resection specimen

        Gastric cancer resection specimen

        Frozen section description
        • Frozen section examination may be required if previously undetected distant lesions are identified during attempted surgical resection (e.g., within the peritoneum or liver)
        • Appearance of the tumor on frozen section would mirror that seen in permanent sections (see Microscopic (histologic) description)
        Microscopic (histologic) description
        • Features:
          • Tubular adenocarcinoma comprises haphazardly arranged neoplastic ducts of varying size and shape, often forming cribriform structures; the stroma is typically desmoplastic
          • Areas of necrosis may be present
          • Anastomosing / irregularly fused glands are present in a subgroup of cases and are associated with low grade cytological atypia, a poorly differentiated adenocarcinoma component and frequent RHOA and CDH1 mutations (Mod Pathol 2019;32:568)
          • Tubular adenocarcinoma is the most common histological subtype of gastric adenocarcinoma
          • Papillary adenocarcinoma comprises neoplastic epithelial cells, commonly well differentiated, arranged around fibrovascular cores forming elongated finger-like projections
          • Papillary adenocarcinoma is a rare histological subtype of gastric adenocarcinoma
        • Grading:
          • Graded as low (formerly well differentiated) or high grade (formerly poorly differentiated) according to the World Health Organization Classification, 2019
        • Staging (TNM 8th edition):
          • T stage is dependent on how deeply the tumor has infiltrated the stomach wall; i.e., carcinoma in situ (Tis), invasion of the lamina propria or muscularis mucosae (T1a), submucosa (T1b), muscularis propria (T2), subserosa (T3), serosa / visceral peritoneum (T4a) or adjacent structures (T4b)
          • N stage is dependent on how many regional lymph nodes are involved; i.e., none (N0), 1 - 2 nodes (N1), 3 - 6 nodes (N2), 7 - 15 nodes (N3a) or > 15 nodes (N3b)
          • M stage is dependent on the presence of distant metastases; i.e., not present (M0) or present (M1) (Gastric Cancer 2018;21:643)
        • Tumor regression grading (TRG):
          • Tumor regression may be seen in resection specimens following neoadjuvant therapy
          • Fibroinflammatory granulation tissue with foamy macrophages and dystrophic calcifications or acellular mucin lakes replace areas previously infiltrated by tumor and residual malignant cells may show marked nuclear atypia
          • Evidence of tumor regression is associated with better prognosis and is commonly graded (Front Oncol 2013;3:262)
          • The Mandard classification of TRG is dependent on the ratio of therapy induced fibrosis to residual tumor; i.e., no detectable tumor (TRG 1), scattered tumor cells (TRG 2), tumor cells with preponderance of fibrosis (TRG 3), fibrosis with preponderance of tumor cells (TRG 4) or tumor without changes of regression (TRG 5)
        Microscopic (histologic) images

        Contributed by Adrian C. Bateman, M.B.B.S., M.D.
        Tubular adenocarcinoma

        Tubular adenocarcinoma

        Tumor regression

        Tumor regression

        HER2

        HER2

        PDL1

        PDL1



        Contributed by @Andrew_Fltv on Twitter
        Contributed by @Andrew_Fltv on Twitter (see original post here)"> Intestinal type adenocarcinomaContributed by @Andrew_Fltv on Twitter (see original post here)"> Intestinal type adenocarcinoma

        Intestinal type adenocarcinoma

        Cytology description
        • Peritoneal lavage cytology may be used in the staging of gastric cancer
        • Cytological features include a high N:C ratio and irregularly shaped and variably sized nuclei with dark chromatin
        Positive stains
        • Immunohistochemistry is not usually required to make the diagnosis
        • Immunohistochemistry is used to confirm the HER2 status (with HER2 in situ hybridization if required), although HER2 expression is heterogenous in 40% of gastric cancers and multiple biopsies are recommended to obtain a meaningful result
        • Immunohistochemistry is also important for confirming the PDL1 status
        • Immunohistochemistry may also be performed for mismatch repair deficiency which carries a better prognosis than patients that are mismatch repair proficient (Br J Surg 2018;105:159)
        Molecular / cytogenetics description
        • Next generation sequencing for microsatellite instability (MSI), high tumor mutational burden (TMB) or NTRK gene fusions can be performed to guide targeted therapeutic agents as per the National Comprehensive Cancer Network Guidance version 2.2022 (NCCN Guidelines: Gastric Cancer [Accessed 9 February 2023])
        Sample pathology report
        • Stomach, endoscopic mucosal resection:
          • Invasive tubular adenocarcinoma, pT1a, N0 (see comment and synoptic report)
          • Comment: This specimen comprises nonspecialized gastric mucosa and some submucosa. Moderately differentiated tubular pattern adenocarcinoma is present, composed of irregular glands with nuclear atypia. The tumor measures 12 mm in width and invades into the muscularis mucosae but does not extend into the included submucosa. There is no evidence of lymphovascular or perineural invasion. The tumor lies 3 mm from the peripheral mucosal margin and 2 mm clear of the deep margin. The adjacent mucosa shows chronic gastritis with intestinal metaplasia and foci of high grade dysplasia.
        Differential diagnosis
        Board review style question #1

        Which of the following conditions may be a differential diagnosis of well / moderately differentiated tubular or papillary gastric adenocarcinoma on biopsy?

        1. Epithelioid pattern gastrointestinal stromal tumor
        2. Invasive ductal carcinoma of the breast
        3. Marginal zone lymphoma
        4. Small cell carcinoma of the lung
        5. Synovial sarcoma
        Board review style answer #1
        B. Invasive ductal carcinoma of the breast. Metastatic invasive ductal carcinoma of the breast commonly possesses a tubular type growth pattern and therefore could mimic tubular pattern gastric adenocarcinoma. In contrast, epithelioid pattern gastrointestinal stromal tumors will appear less pleomorphic and will show no tubular or papillary growth patterns and the immunohistochemical expression profile will differ from adenocarcinoma. Marginal cell lymphoma comprises sheets of small lymphoid cells, sometimes showing plasmacytic differentiation and lymphoepithelial lesions. Small cell carcinoma comprises sheets of epithelial cells with inconspicuous cytoplasm, exhibiting nuclear molding and showing a high mitotic and apoptotic index, often with areas of necrosis. Synovial sarcoma is usually a biphasic tumor with a predominant spindle cell component and characteristic immunohistochemical expression profile.

        Comment Here

        Reference: Intestinal type adenocarcinoma
        Board review style question #2
        Which of the following is a risk factor for gastric adenocarcinoma?

        1. Autoimmune gastritis
        2. Bile reflux into the stomach
        3. Fundic gland polyps
        4. Nonsteroidal anti-inflammatory drug use
        5. Peptic ulceration of the stomach
        Board review style answer #2
        A. Autoimmune gastritis leads to destruction of gastric parietal cells, hypochlorhydria and bacterial overgrowth, with the resultant production of carcinogens within the stomach. In contrast, bile reflux and nonsteroidal anti-inflammatory drugs lead to a reactive / chemical gastropathy but are not associated with gastric atrophy / adenocarcinoma. Fundic gland polyps only very rarely contain areas of dysplasia. Peptic ulceration most commonly occurs due to Helicobacter associated gastritis but is not an independent risk factor for adenocarcinoma.

        Comment Here

        Reference: Intestinal type adenocarcinoma

        Iron pill gastritis / pill gastritis
        Definition / general
        • Corrosive injury to gastric mucosa secondary to oral iron supplement intake
        Essential features
        • History of oral iron supplement use for iron deficiency anemia
        • Nausea, vomiting and epigastric pain
        • Mucosal injury with crystalline brown pigmented material deposition
        Terminology
        • Pill gastritis
        ICD coding
        • ICD-10:
          • K29.60 - other gastritis without bleeding
          • T45.4X5A - adverse effect of iron and its compounds, initial encounter
        Epidemiology
        Sites
        • Stomach
        • Gastroesophageal junction
        • Esophagus and duodenum
        Pathophysiology
        • Deposition of brown-black inorganic iron crystals on the surface and formation of reactive oxygen metabolites that have corrosive, thrombotic and inflammatory properties (Gastroenterology Res 2017;10:138)
        Etiology
        • Iatrogenic oral iron supplements (pills or tablets)
        Clinical features
        Diagnosis
        • Upper endoscopy with biopsy
        Laboratory
        • Microcytic hypochromic anemia
        Prognostic factors
        Case reports
        Treatment
        • Discontinuing use of the oral iron medication (ACG Case Rep J 2013;1:13)
        • Switch to less toxic form, such as liquid iron preparation
        • Intravenous iron supplementation
        Clinical images

        Contributed by Monica Garcia-Buitrago, M.D.
        Iron pill gastritis

        Gastric mucosa

        Gross description
        Microscopic (histologic) description
        • Mucosal injury in the form of erosion, ulceration and granulation tissue, according to the extent of the injury (Arch Pathol Lab Med 2004;128:821)
        • Acute inflammation in surrounding tissue, inflammatory exudate and reactive epithelial changes
        • Crystalline, metallic, brown-black pigmented material deposition in the superficial mucosa and ulcer bed
        • Iron is refractile but does not polarize
        Microscopic (histologic) images

        Contributed by Monica Garcia-Buitrago, M.D. and Domenika Ortiz, M.D.
        Brown crystalline iron deposition Brown crystalline iron deposition Brown crystalline iron deposition

        Brown crystalline iron deposition

        Prussian blue stain Prussian blue stain

        Prussian blue stain


        Mucosal calcinosis

        Mucosal calcinosis

        Mucosal calcinosis von Kossa stain

        Mucosal calcinosis von Kossa stain

        Nonspecific pill gastritis

        Nonspecific pill gastritis

        Negative stains
        Sample pathology report
        • Stomach, biopsy:
          • Acute gastritis with ulceration and crystalline, brown pigmented material deposition, consistent with iron pill gastritis (see comment)
          • Comment: A Prussian blue stain confirmed that the brown pigmented material is crystalline iron deposits.
        Differential diagnosis
        Board review style question #1

        A microphotograph of a gastric biopsy of a 68 year old woman with a history of iron deficiency anemia is shown above. What is the most likely diagnosis?

        1. Autoimmune gastritis
        2. Helicobacter pylori gastritis
        3. Iron pill gastritis
        4. Proton pump inhibitor (PPI) changes
        5. Reactive gastropathy
        Board review style answer #1
        C. Iron pill gastritis. Gastric mucosa with acute inflammation, mucosal erosion and crystalline brown pigmented material deposition.

        Comment Here

        Reference: Iron pill gastritis / pill gastritis
        Board review style question #2

        An 82 year old patient with a history of iron deficiency anemia on oral iron supplement presented with vague epigastric pain and erythema on endoscopy. What is the most appropriate special stain for the biopsy specimen?

        1. Congo red
        2. Grocott methenamine silver stain
        3. Periodic acid-Schiff stain
        4. Prussian blue stain
        5. Warthin starry stain
        Board review style answer #2
        D. Prussian blue stain, which highlights the crystalline iron deposition in the superficial gastric mucosa

        Comment Here

        Reference: Iron pill gastritis / pill gastritis

        Juvenile polyp
        Definition / general
        • Also called retention polyp
        • Part of generalized juvenile polyposis coli or rare gastric subtype, 20 - 50% of patients with syndrome have gastric polyps
        • Syndrome due to genetic dysregulation of TGF-β pathway
        • Associated with increased risk of colorectal and gastric carcinoma
        Gross description
        • Smooth surfaced, 1 - 2 cm, short narrow stalk
        Microscopic (histologic) description
        • Irregular cysts in lamina propria with normal gastric epithelium
        • Surface and foveolar hyperplasia, may have stromal hemorrhage, surface ulceration and chronic inflammation due to torsion
        Microscopic (histologic) images

        Image hosted on other server:

        Gastric juvenile polyp

        Differential diagnosis
        • Hyperplastic polyp: usually background mucosal disease, may need clinical or family history for definitive diagnosis

        Lanthanum carbonate deposition
        Definition / general
        • Lanthanum carbonate is an oral phosphate binder used to treat hyperphosphatemia in patients with end stage renal disease
        • Recent reports have indicated that the medication can deposit throughout the gastrointestinal tract mucosa, most commonly in the stomach but also in the small intestine and colon (Am J Surg Pathol 2015;39:767, Int J Surg Pathol 2016;24:89)
        Essential features
        • Lanthanum may become embedded in the mucosa following digestion by gastric acid; deposition may be detected years after cessation of therapy (Pathol Int 2017;67:389)
        • Lanthanum deposition can be detected radiographically but is most often observed microscopically on biopsy tissue
        Epidemiology
        • Patients with end stage renal disease
        Gross description
        • Appears as aggregates of brown purple amorphous material in the mucosa, engulfed by epithelioid histiocytes
        • Overlying mucosa may be intact or eroded
        • Identity of the material can be confirmed by energy dispersive Xray spectrometry, if necessary
        Microscopic (histologic) images

        Case #442

        Lanthanum carbonate in stomach

        Von Kossa

        Negative stains
        Sample pathology report
        • Stomach, fundus, biopsy:
          • Stomach mucosal with reactive change and focal lanthanum carbonate deposition (see comment)
          • Comment: The patient’s history of end stage renal disease and use of oral lanthanum carbonate deposition is noted. The deposited material is negative on von Kossa special stain.
        Differential diagnosis
        Board review style question #1
        A 35 year old man with no history of renal failure undergoes upper endoscopy and colonoscopy for abdominal pain. Crystalline material is observed deposited in his gastric mucosa. What is the most likely identity of this material?

        1. Calcium
        2. Iron
        3. Kayexalate
        4. Lanthanum
        5. OsmoPrep
        Board review style answer #1
        E. OsmoPrep, used to prepare the bowel for endoscopy

        Comment Here

        Reference: Lanthanum carbonate deposition

        Leiomyoma
        Definition / general
        • Benign smooth muscle tumor, rare in stomach (50× less than GIST)
        Microscopic (histologic) description
        • Smooth muscle tumor with minimal atypia; no / rare mitoses
        Positive stains
        Negative stains
        Electron microscopy description
        • Pinocytotic vesicles, subplasmalemmal dense patches, cytoplasmic microfilaments with focal densities

        Leiomyosarcoma
        Definition / general
        • Malignant smooth muscle sarcoma that arises from the muscularis propria of the stomach
        • Accounts for 1% of all malignant tumors of the stomach
        Essential features
        • Rare malignant neoplasm with atypia, mitoses and necrosis histologically
        • Positive for SMA and desmin
        • Intermediate to poor prognosis
        Epidemiology
        Sites
        Clinical features
        Prognostic factors
        Case reports
        • 50 year old woman with hypertension and diabetes presented with a submucosal gastric mass (Case #438)
        Gross images

        Images hosted on other servers:

        Large gastric leiomyosarcoma (> 8 cm)

        Microscopic (histologic) description
        Microscopic (histologic) images

        Contributed by Raul S. Gonzalez, M.D.

        Leiomyosarcoma



        Case #438

        H&E


        SMA

        MSA

        Desmin

        Ki67

        Positive stains
        Negative stains
        Sample pathology report
        • Stomach, antrum, resection:
          • Leiomyosarcoma (6.4 cm) (see comment)
          • Margins of resection unremarkable.
          • Two benign lymph nodes.
          • Comment: The tumor shows prominent nuclear atypia, numerous mitotic figures and regions of necrosis. Immunohistochemical stains for SMA and desmin are positive in the tumor. Leiomyosarcomas are rare in the stomach and risk factors for progression in this location remain poorly defined.
        Differential diagnosis
        Board review style question #1
        Which of the following is a poor prognostic indicator for leiomyosarcoma of the stomach?

        1. Greater than 20 mitoses per 10 high power fields
        2. Ki67 proliferative index above 50%
        3. Size greater than 5 cm
        4. Tumor necrosis
        Board review style answer #1
        C. Size greater than 5 cm

        Comment Here

        Reference: Leiomyosarcoma

        Lymphocytic gastritis
        Definition / general
        • A pattern of gastric mucosal injury characterized by increased intraepithelial lymphocytes (> 25 per 100 epithelial cells) and increased chronic inflammatory cells in the lamina propria
        Essential features
        • Associated with Helicobacter pylori infection and celiac disease, among other etiologies
        • Variable clinical presentation
        • Uncertain pathogenesis
        Terminology
        • The older entity “varioliform gastritis” (endoscopic mucosal appearance of small nodules with central erosions and enlarged rugal folds) appears to represent a subset of lymphocytic gastritis (J Pathol 1989;158:19)
        ICD coding
        • ICD-10: K52.89 - other specified noninfective gastroenteritis and colitis
        Epidemiology
        Sites
        Pathophysiology
        • Uncertain pathogenesis overall
        • Due to its association with celiac disease and H. pylori, it has been proposed to be a local immune response to luminal antigens such as gliadin and Helicobacter antigens (Am J Surg Pathol 1999;23:153)
        Clinical features
        • Dyspepsia, abdominal pain and iron deficiency anemia have been reported but presenting symptoms are variable (Pathol Case Rev 2008;13:167)
        • Predominately recurrent vomiting, epigastric pain or chronic diarrhea is seen in pediatric patients (J Pediatr 1994;124:57)
        • Cases associated with endoscopic varioliform gastritis may present with weight loss and anorexia (Gut 1990;31:282)
        • Cases associated with Ménétrier disease may present with protein losing enteropathy (Hum Pathol 1991;22:379)
        Diagnosis
        • Established by gastric biopsy
        Case reports
        Treatment
        Clinical images

        Images hosted on other servers:
        Missing Image

        Varioliform gastritis:
        multiple nodules with
        central mucosal
        atrophy / erosion

        Gross description
        • Small nodules with central erosions and enlarged rugal folds (“varioliform gastritis”)
          • Early studies suggested this finding in 80% of cases
          • Subsequent articles reported incidence of 4% to 30% (Pathol Case Rev 2008;13:167)
        • Additional findings: small elevated plaques, superficial erosions, nodular appearance, thickened folds
        • Unremarkable appearance in up to 50% of cases (J Clin Pathol 1995;48:939, Gut 1988;29:1258)
        • Hypertrophic gastric folds has been described in cases of Ménétrier disease (Hum Pathol 1991;22:379)
        Microscopic (histologic) description
        • Defined by increased gastric intraepithelial lymphocytes (> 25 per 100 epithelial cells)
          • Intraepithelial lymphocytosis is typically greater in the surface epithelium
          • Most cases show around 35 - 75 surface intraepithelial lymphocytes per 100 epithelial cells (Gut 1990;31:282)
        • Lymphoplasmacytic expansion of the lamina propria can be additionally seen
        • Intraepithelial neutrophils may be seen, especially in the setting of H. pylori infection or mucosal erosion
        • Lymphoepithelial lesions are not present
        Microscopic (histologic) images

        Contributed by Matthew Morrow, M.D.
        Missing Image

        Increased intraepithelial lymphocytes

        Missing Image Missing Image Missing Image Missing Image

        Increased intraepithelial lymphocytes, expanded lamina propria

        Missing Image

        Biopsy from area of nodular gastric mucosa



        Contributed by @RaulSGonzalezMD on Twitter
        bit.ly/3O97D3J #pathology #gipath #PathTwitter #PathOutPic"
        Contributed by @RaulSGonzalezMD on Twitter (see original post here)"> Lymphocytic gastritis

        Lymphocytic gastritis

        Positive stains
        • Lymphocytes are predominately CD3 positive T cells with CD8 co-expression
        Sample pathology report
        • Stomach, biopsy:
          • Corpus and antral mucosa with chronic inactive gastritis with increased intraepithelial lymphocytes (see comment)
          • Immunostain for Helicobacter species is negative
          • Comment: The finding of intraepithelial lymphocytosis is consistent with lymphocytic gastritis, which may be associated with celiac disease, H. pylori gastritis, viral infection, Crohn disease, certain medications and other etiologies
        Differential diagnosis
        • Gastric lymphoma
          • Often causes a mass lesion, unlike lymphocytic gastritis
          • Patchy increased intraepithelial lymphocytes can be seen in MALT lymphoma (Korean J Pathol 2007;41:289)
          • Lymphocytes in MALT lymphoma are CD20+ B cells
          • Lymphoma may display an infiltrative pattern with lymphoepithelial lesions and architectural distortion
          • Cytologic atypia may be present
        • Helicobacter pylori gastritis
          • Curved slender bacteria present in the superficial mucus layer and along the surface of gastric epithelial cells
          • Neutrophils and germinal centers may be seen
          • Chronic gastritis, such as mononuclear expansion of the lamina propria seen in lymphocytic gastritis, should prompt scrutiny for Helicobacter organisms
          • Helicobacter immunostain, Giemsa and silver stains highlight organisms
        Additional references
        Board review style question #1

          The image above is from an antral biopsy from a 9 year old boy. What additional histological finding is most likely to be present in this patient?

        1. Increased intraepithelial eosinophils with eosinophilic microabscesses in the upper esophagus
        2. Curved bacteria rods present in the superficial mucus layer of the stomach
        3. Duodenal increased intraepithelial lymphocytes, crypt hyperplasia and villous blunting
        4. Duodenal villous blunting, crypt hyperplasia and focal surface epithelial “tufting”
        Board review style answer #1
        C. Lymphocytic gastritis is most commonly seen in a background of celiac disease in children (intraepithelial lymphocytes, crypt hyperplasia and villous blunting in the duodenum) and may portend a more severe disease course. Helicobacter infection (choice B) would be more likely in an adult patient.

        Comment Here

        Reference: Lymphocytic gastritis
        Board review style question #2
        Aside from increased intraepithelial lymphocytes, what other histologic feature in the stomach is commonly described in lymphocytic gastritis?

        1. Lymphoepithelial lesions
        2. Lymphoplasmacytic expansion of the lamina propria
        3. Loss of parietal cells
        4. Increased subepithelial collagen deposition
        Board review style answer #2
        B. Lymphoplasmacytic expansion of the lamina propria is also commonly seen in lymphocytic gastritis.

        Comment Here

        Reference: Lymphocytic gastritis

        Menetrier disease
        Definition / general
        • A hyperplastic gastropathy described by Menetrier in 1888 as polyadenomes en nappe
        • Giant mucosal folds involving fundus and body, usually (not always) spares antrum
        Clinical features
        • 75% men
        • Mean age 30 - 50s; chronic and severe
        • Children: uncommon; often have CMV or other infection, peripheral eosinophilia
        • Peripheral edema present due to mucosal protein loss, variable weight loss, diarrhea
        • Low acid production even after stimulation
        • Likely due to excessive transforming growth factor alpha with increased signalling of EGFR
        • Grossly and radiologically resembles Zollinger-Ellison syndrome

        Localized Menetrier disease:
        • Rare localized hyperplastic gastropathy, associated with stomach adenocarcinoma (Am J Surg Pathol 1997;21:1334)
        • Symptoms: upper abdominal discomfort, loss of appetite, weight loss, anemia, occasionally hypoproteinemia
        • Gross description: circumscribed area of giant folds, well demarcated from surrounding normal appearing mucosa, usually in body or antrum
        • Micro description: increase in epithelial cell mass of mucous cells with long, sometimes cystically dilated foveola, mild inflammatory infiltrate
        Treatment
        Gross description
        • Usually affects greater curvature of stomach
        • Markedly hypertrophic gastric folds resembling cerebral convolutions
        • Abrupt transition to normal mucosa
        Microscopic (histologic) description
        • Marked foveolar hyperplasia, tortuous (corkscrew) and cystically dilated foveolar glands
        • May extend into muscularis mucosa
        • Atrophic glandular compartment
        • Edematous and mildly inflamed lamina propria, may have increased intraepithelial lymphocytes
        Differential diagnosis

        Mixed neuroendocrine-nonneuroendocrine
        Definition / general
        • Rare; mixed neuroendocrine and nonneuroendocrine carcinoma
        • Cells have dual differentiation, with mucus and neuroendocrine granules in cytoplasm of same cells

        Molecular pathology
        Definition / general
        Essential features
        The Cancer Genome Atlas (TCGA) network classification
        • EBV positive tumors (~9%)
          • Gastric fundus or body
          • Male prevalence
          • Extensive DNA promoter hypermethylation
          • CDKN2A (p16INK4A) promoter hypermethylation
          • PIK3CA, ARID1A and BCOR mutations
          • Overexpression of PDL1 and PDL2 (assessable with IHC)
        • Tumors with MSI (~22%)
          • Preferred localization in the gastric antrum
          • Associated with intestinal histotype
          • Elderly age of onset
          • More favorable prognosis than the other molecular subtypes
          • MLH1 gene promoter hypermethylation
          • High mutational burden
          • In contrast to MSI colorectal adenocarcinoma, there is no association with BRAF mutations
          • Can be part of the spectrum of inherited malignancies such as Lynch syndrome
        • Genomically stable tumors (~20%)
          • Distal localization
          • Poorly cohesive histotype
          • Younger patients in comparison to other molecular subtypes
          • Worst prognosis among the 4 TCGA subtypes
          • Low copy number alterations
          • Low mutational burden
          • ARID1, RHOA and CDH1 mutations
          • CLDN18-ARHGAP26 fusions in 15%
        • Tumors with chromosomal instability (CIN) (~50%)
          • Frequently located at the gastroesophageal junction / cardia
          • Most are classified as intestinal type
          • DNA aneuploidy and highly variable chromosomal copy numbers
          • Frequent mutations of the tumor suppressor TP53
          • Frequent genomic amplifications of receptor tyrosine kinases (RTKs) / RAS pathway, including epidermal growth factor receptor (EGFR), ERBB2 (HER2), ERBB3, MET proto-oncogene (MET), fibroblast growth factor receptor 2 (FGFR2), vascular endothelial growth factor A (VEGFA) and KRAS
        • Reference: Nature 2014;513:202
        The Asian Cancer Research Group (ACRG) classification
        • Microsatellite unstable (MSI) tumors (~23%)
          • Preferred location in the gastric antrum
          • Intestinal histology
          • Often diagnosed at an early stage
          • Best overall prognosis
          • Silencing of MLH1 gene
          • DNA methylation signature
          • Presence of hypermutation
          • Mutations of ARID1A, KRAS and ALK
          • Alterations affecting the PI3K-PTEN-mTOR pathway
          • Overexpression of PDL1 (assessable with IHC)
        • Epithelial to mesenchymal-like type tumors (microsatellite stable [MSS] / EMT) (~15%)
          • Poorly cohesive histotype
          • Younger age at presentation compared to all other subtypes
          • Higher TNM stages at presentation (III / IV)
          • Higher frequency of peritoneal metastases
          • Worst prognosis
          • CDH1 loss of expression
          • Lower number of mutation events when compared to the other MSS groups
        • MSS / TP53 positive tumors (~26%)
          • Intestinal histology
          • Male predominance
          • Frequent EBV infection
          • Frequent mutations in ARID1A, PIK3CA, SMAD4 and APC
        • MSS / TP53 negative tumors (~36%)
          • Intestinal histology
          • Male predominance
          • Highest prevalence of TP53 and RHOA mutations ("TP53 negative" refers to loss of function, not presence of mutation)
          • Mutations in APC, ARID1A, KRAS, PIK3CA and SMAD4
        • Reference: Nat Med 2015;21:449
        Sites
        Clinical features
        Diagnosis
        Prognostic factors
        Treatment
        Microscopic (histologic) description
        Microscopic (histologic) images

        Contributed by Matteo Fassan, M.D., Ph.D.

        HER2 3+ positive gastric adenocarcinoma

        EBER positive gastric adenocarcinoma

        MLH1 negative gastric adenocarcinoma

        Positive stains
        Board review style question #1

        What is an important molecular characteristic associated with microsatellite unstable gastric carcinomas?

        1. Frequent amplification of the ERBB2 (HER2) gene
        2. High frequency of BRAF mutations
        3. High mutational burden
        4. Poor prognosis
        5. Poorly cohesive histotype
        Board review style answer #1
        C. High mutational burden. Alterations in the DNA mismatch repair machinery results in the accumulation of frame shift mutations (either through insertions or deletions) with a subsequent increased mutational burden.

        Comment Here

        Reference: Molecular pathology of stomach cancer

        Mucosal calcinosis
        Definition / general
        • Associated with organ transplant patients or ulcer patients taking aluminum containing antacids or sucralfate (Am J Surg Pathol 1993;17:45)
        Microscopic (histologic) description
        • Calcium deposits resemble CMV inclusions, 40 - 250 microns, just beneath surface epithelium at foveolar tips
        Microscopic (histologic) images

        Contributed by Robin Pike, M.D.
        Granular material in lamina propria Granular material in lamina propria

        Granular material in lamina propria

        Positive von Kossa stain

        Positive von Kossa stain


        Neuroendocrine-classification
        Table of Contents
        Classification | Positive stains
        Classification
        2019 WHO classification
        • Neuroendocrine tumor (NET):
          • NET, grade 1 (G1): well differentiated low grade
          • NET, G2: well differentiated intermediate grade
          • NET, G3: well differentiated high grade
        • Neuroendocrine carcinoma, small cell type (SCNEC)
        • Neuroendocrine carcinoma, large cell type (LCNEC)
        • Mixed neuroendocrine - nonneuroendocrine neoplasm (MiNEN)
        • Grading of well differentiated neuroendocrine tumors:
          • Well differentiated NET, G1: mitotic rate (mitoses/mm2) < 2 and Ki67 < 3%
          • Well differentiated NET, G2: mitotic rate (mitoses/mm2) 2 - 20, Ki67 < 3 - 20% (use highest grade to categorize)
          • Well differentiated NET, G3: mitotic rate (mitoses/mm2) > 20, Ki67 > 20% (use highest grade to categorize)
        • Poorly differentiated neuroendocrine carcinoma:
          • Mitotic rate (mitoses/2 mm2) > 20 or Ki67 index > 20%
        Positive stains

        Oxyntic gland adenoma
        Definition / general
        • Benign gastric neoplasm composed of gland forming epithelial cells with predominantly chief cell differentiation resembling oxyntic glands
        • Risk of progression to gastric adenocarcinoma of fundic gland type with submucosal invasion
        Essential features
        Terminology
        • Many terms have been proposed for this polyp and its related invasive lesion, including:
          • Oxyntic gland polyp / adenoma
          • Oxyntic gland neoplasm
          • Gastric dysplasia of chief cell predominant type
          • Chief cell predominant gastric polyp
          • Chief cell hyperplasia with structural and nuclear atypia
          • Chief cell proliferation of the gastric mucosa
          • Chief cell adenoma
          • Gastric neoplasia of fundic gland (chief cell predominant) type
          • Gastric adenocarcinoma of fundic gland type
          • Gastric adenocarcinoma of fundic gland mucosa type
          • Gastric adenocarcinoma with chief cell differentiation
          • Chief cell predominant adenocarcinoma
        ICD coding
        • ICD-O: 8210/2 - adenomatous polyp, high grade dysplasia
        • ICD-O: 8210/0 - adenomatous polyp, low grade dysplasia
        • ICD-11: 2E92.1 & XH3DV3 - benign neoplasm of stomach and adenoma, NOS
        Epidemiology
        Sites
        • Stomach, usually upper third (80%)
        Etiology
        • Unknown
        • Not associated with Helicobacter pylori infection, atrophic gastritis or intestinal metaplasia
        • Questionable association with acid reduction therapy in patients with gastrointestinal reflux disease (World J Gastroenterol 2016;22:10523)
        Clinical features
        • Usually incidental endoscopic finding
        Diagnosis
        • Endoscopy
        Prognostic factors
        • No recurrence following endoscopic excision in all reported cases
        • Risk of progression to submucosal invasion (gastric adenocarcinoma of fundic gland type) (World J Gastroenterol 2016;22:10523)
        • No risk of nodal or distant metastasis
        Case reports
        • 49 year old man who was incidentally found to have a polypoid whitish elevation with a smooth surface in the gastric fundus measuring 1.1 cm (Case Rep Pathol 2016;2016:8646927)
        • 73 year old man with 6 mm slightly elevated, yellowish lesion with irregular surface vessels detected at the greater curvature of the upper body. (World J Gastroenterol 2015;21:5099)
        Treatment
        • Complete excision
        Clinical images

        Contributed by Natalia Liu, M.D. and Hanlin L. Wang, M.D., Ph.D.
        Flat raised nodule

        Flat raised nodule

        Gross description
        Microscopic (histologic) description
        • Usually arising within the lower zone of oxyntic mucosa
        • Clusters of irregular glands typically separated by radiating wisps of smooth muscle (Mod Pathol 2020;33:206)
        • Dilated, branched, fused and anastomosed glands can be seen
        • Composed predominantly of columnar epithelial cells with chief cell differentiation characterized by basophilic cytoplasm and relatively uniform round nuclei (World J Gastroenterol 2016;22:10523)
        • Can have minor components of parietal cells and mucus cells
        • No more than mild nuclear atypia (World J Gastroenterol 2016;22:10523)
        • No mitosis, necrosis, desmoplasia or lymphovascular invasion
        • Mucosal surface may be spared and covered by gastric foveola and nonneoplastic oxyntic glands
        • If submucosal invasion is present, should be classified as gastric adenocarcinoma of fundic gland type
        Microscopic (histologic) images

        Contributed by Natalia Liu, M.D. and Hanlin L. Wang, M.D., Ph.D.
        Architecture

        Low power architecture

        Irregular glands

        Irregular glands

        Predominantly chief cell differentiation

        Predominantly chief cell differentiation



        Contributed by @liverwei on Twitter
        Oxyntic gland polyp Oxyntic gland polyp

        Oxyntic gland polyp

        Oxyntic gland polyp Oxyntic gland polyp

        Oxyntic gland polyp

        Cytology description
        • Bland cytologic features, uniform round nuclei, inconspicuous nucleoli, abundant basophilic cytoplasm (Mod Pathol 2020;33:206)
        Molecular / cytogenetics description
        • Missense or nonsense mutations in Wnt / beta catenin signaling pathway (APC, CTNNB1, AXIN1, AXIN2) in ~50% of cases (Hum Pathol 2013;44:2438)
        • GNAS mutations in 19% of cases, almost mutually exclusive with mutations in Wnt / beta catenin pathway (Hum Pathol 2014;45:2488)
        Sample pathology report
        • Stomach, polyp, biopsy:
          • Oxyntic gland adenoma (see comment)
          • Comment: The biopsy shows bland appearing glandular proliferation in the lower portion of the oxyntic mucosa. The epithelial cells lining the glandular structures exhibit features of chief cell differentiation. No features of invasive carcinoma are identified but there is insufficient submucosal tissue present for evaluation. The findings are consistent with a diagnosis of oxyntic gland adenoma, which is regarded as a benign lesion. Complete excision of the lesion is considered curative as no recurrence or metastasis has been documented in reported cases in the literature. Given its potential risk of progression to gastric adenocarcinoma of fundic gland type, endoscopic follow up is recommended.
        Differential diagnosis
        • Well differentiated adenocarcinoma:
          • More prominent glandular anastomosis with lateral extension
          • More prominent nuclear atypia
          • Increased mitotic activity
          • Desmoplasia
          • Presence of intestinal metaplasia
          • Presence of dystrophic goblet cells
        • Well differentiated neuroendocrine tumor:
          • Nested or organoid growth pattern
          • Salt and pepper nuclear features
          • Positive immunostains for both chromogranin and synaptophysin
        • Pyloric gland adenoma:
          • Tightly packed pyloric type tubular glands
          • Cuboidal to low columnar epithelial cells showing pale or lightly eosinophilic cytoplasm with a ground glass appearance
          • Positive immunostains for both MUC5AC and MUC6
        • Fundic gland polyp:
          • Cystically dilated oxyntic glands lined by hypertrophic or flattened parietal cells admixed with chief cells and mucus cells
        Board review style question #1

        Which of the following is true about this gastric polyp?

        1. GNAS mutations occur in majority of this type of polyp
        2. It carries a risk of progression to submucosal invasion
        3. It is predominantly composed of columnar epithelial cells with parietal cell differentiation
        4. It is usually seen in the gastric antrum
        5. It stains positive for both MUC5AC and chromogranin
        Board review style answer #1
        B. It carries a risk of progression to submucosal invasion

        Comment Here

        Reference: Oxyntic gland adenoma
        Board review style answer #2
        What histologic feature is typically seen in an oxyntic gland adenoma?

        1. It frequently shows cystically dilated glands lined by hypertrophic parietal cells
        2. It frequently shows H. pylori gastritis in background gastric mucosa
        3. It frequently shows irregular glands separated by smooth muscle bundles
        4. It frequently shows lymphovascular invasion
        5. It frequently shows mitotic figures
        Board review style answer #2
        C. It frequently shows irregular glands separated by smooth muscle bundles

        Comment Here

        Reference: Oxyntic gland adenoma

        Peptic ulcer disease
        Definition / general
        • Ulcer:
          • Breach in muscularis mucosa of GI tract
          • Erosion is more superficial breach
          • Ulcers begin as erosions, but not all erosions progress to ulcers

          Peptic ulcer:
          • Chronic, usually solitary, due to acid-peptic juices

        • Causes:
          • Mucosal injury due to Helicobacter pylori infection, NSAID use, Zollinger-Ellison syndrome (multiple peptic ulcerations in stomach, duodenum and jejunum due to excess gastrin secretion by a tumor), ischemia, bile / pancreatic juice reflux
          • Alcohol, smoking, COPD and corticosteroids use may exacerbate peptic ulcer disease and impair healing
          • Hyperacidity present in minority of duodenal ulcers and only rarely in gastric ulcers
          • H. pylori has nearly universal association for duodenal ulcers; present in 65% of gastric ulcers and 90% of gastric ulcers not related to NSAID use or Zollinger-Ellison syndrome
        Clinical features
        • Incidence in US of 4 million, 350,000 new cases / year
        • 3,000 deaths per year
        • Affects 10% of American men, 4% of women (M/F = 3:1 for duodenal ulcers, 1.5-2:1 for gastric ulcers)
        • Incidence has decreased recently for duodenal ulcers, not for gastric ulcers
        • Usually in pyloric-type mucosa along lesser curvature
        • 20% have coexisting duodenal ulcer
        • 5% are multiple
        • Mean age 50 years, but may occur in children
        • Multiple (~10) biopsies recommended to rule out malignancy

        • Symptoms:
          • Epigastric burning
          • Pain worse at night, within 1-3 hours after meals
          • Pain may decrease with food / alkali
          • Perforation associated with pain in back, left upper quadrant, chest
          • Usually impairs life but doesn’t shorten it
          • Heals in 15 years without treatment versus weeks with treatment
          • Complications: perforation, hemorrhage, obstruction, surgery
        Pathophysiology
        • H. pylori related ulcers:
          • Produces urease (to protect it from acid), protease (breaks down glycoproteins in gastric mucus), phospholipase (damages epithelial cells, may release leukotrienes)
          • Attracts neutrophils that produce myeloperoxidase (turns HCl into hypochlorous acid, combines with NH3 to form monochloramine)
          • Both hypochlorous acid and monochloramine destroy mammalian cells

        • NSAID related ulcers:
        Sites
        • Duodenum, antrum, lesser curvature near incisura most common site in stomach, GE junction, margins of gastrojejunostomy, adjacent to Meckel diverticulum containing ectopic gastric mucosa, lower esophagus
        • 98% in stomach or duodenum
        • Duodenum:stomach 4:1
        • Gastric ulcers on greater curvature or not in antrum are more likely to be related to NSAID use, ulceration may also be caused by tumors, infections (other than H. pylori) especially in immunocompromised patients
        Treatment
        • Antibiotics if H. pylori induced promote healing of ulcers and reduce greatly recurrences, proton pump inhibitors, discontinuation of NSAIDS and corticosteroids, smoking cessation
        Gross description
        • 90% < 4 cm, 50% < 2cm, clean base (due to peptic enzymes), surrounded by erythematous mucosa, may see blood vessel in ulcer base
        • Usually sharply punched out defect with straight walls, NO heaped up margins
        • Size doesn’t predict malignancy
        Microscopic (histologic) description
        • Generally associated with H. pylori gastritis, may see reactive gastropathy if history of NSAID use
        • Ulcer may be transmural or limited to mucosa and submucosa
        • Muscle wall replaced by fibrous tissue
        • Serosal fibrosis
        • Hyperplasia of adjacent lymph nodes
        • Proximal mucosa may be overhanging
        • Distal mucosa may have ladder-like configuration
        • Accompanied by active and chronic inflammation, unless NSAID related

        • Active ulcers have 4 prototypical zones:
          • Surface neutrophils, bacteria, necrotic debris and possibly Candida
          • Fibrinoid necrosis at base and margins
          • Granulation tissue with chronic inflammatory cells
          • Fibrous or collagenous scars in muscularis propria with thickened blood vessels showing endarteritis obliterans

        • Healing ulcers:
          • Have regenerating epithelium over the surface
          • May have intestinal metaplasia, marked reactive changes
          • Rarely exhibits hyalinization (severe thickening, usually of submucosa, Arch Pathol Lab Med 1982;106:472)
        Microscopic (histologic) images

        Images hosted on other servers:

        Various images

        Differential diagnosis
        • Acute gastric ulcers due to severe systemic stress
        • Carcinoma (radiologically)

        Peutz-Jeghers
        Definition / general
        • Autosomal dominant disorder with mucocutaneous pigmentation and multiple GI hamartomatous polyps, most common in small intestine
        • Due to mutation in STKII / LKBI tumor suppressor gene related to TGF-β signalling pathway
        • Usually children / teenagers
        • Male = female
        • 20% of Peutz-Jeghers patients have hamartomatous gastric polyps
        • Rarely associated with dysplasia, gastric adenocarcinoma, but may occur at young age (Arch Pathol Lab Med 1982;106:517)
        Gross description
        • 1 - 3 cm, short broad stalk, coarsely lobulated
        Microscopic (histologic) description
        • Core of finely arborizing branches of smooth muscle from muscularis mucosa
        • Covered by normal but often disorganized mucosa
        • Usually no prominent inflammation
        • Pseudoinvasion in 10% (no atypia, normal epithelial cell subtypes, brush border, hemosiderin deposition, intramural mucinous cysts, Am J Surg Pathol 1987;11:743)
        Microscopic (histologic) images

        Images hosted on other servers:

        Gastric Peutz-Jeghers polyps


        Plexiform fibromyxoma
        Definition / general
        Essential features
        • Benign mesenchymal neoplasm that predominantly arises in the gastric antrum and pyloric region
        • Multinodular and plexiform architecture with arborizing vasculature at low power
        • Cytologically bland spindle cells in myxoid or fibromyxoid stroma
        • Rare or no mitotic figures; low proliferation index in most cases
        • Metastases and recurrence following complete excision have not been described
        Terminology
        • Also known as plexiform angiomyxoid myofibroblastic tumor
        ICD coding
        • ICD-O: 8811/0 - plexiform fibromyxoma
        • ICD-11: 2E92.1 - benign neoplasm of stomach
        Epidemiology
        Sites
        Pathophysiology
        • Recurrent MALAT1-GLI1 translocation or GLI1 upregulation have been identified in a subset of plexiform fibromyxomas (J Pathol 2016;239:335)
        Clinical features
        • Symptoms include upper gastrointestinal bleeding, abdominal distension, abdominal pain, nausea, hematemesis, anemia, weight loss, obstruction
        • Some cases are incidental endoscopic finding in asymptomatic patients
        Case reports
        Treatment
        Clinical images

        Contributed by Yujun Gan, M.D., Ph.D. and Xiuli Liu, M.D., Ph.D.

        Large gastric antral mass

        Large mural hypoechoic mass

        FNA of the mass

        Gross description
        • Presents most commonly as a solitary, circumscribed, multilobulated mass centered in the muscularis propria
        • Overlying mucosa may be smooth and unremarkable but sometimes is ulcerated
        • Tumor size varies from 1.5 cm up to 15 cm (Am J Surg Pathol 2009;33:1624)
        Gross images

        Contributed by Yujun Gan, M.D., Ph.D. and Xiuli Liu, M.D., Ph.D.

        Antral mass

        Myxoid and cystic degeneration

        Microscopic (histologic) description
        Microscopic (histologic) images

        Contributed by Yujun Gan, M.D., Ph.D. and Xiuli Liu, M.D., Ph.D.

        Hypocellular and myxoid tumor

        Bland spindle cell proliferation

        Fibrillary stroma

        Spindle and myxoid tumor

        Smooth muscle actin immunoreactivity



        Contributed by Raul S. Gonzalez, M.D.

        Plexiform architecture

        Bland myxoid tumor

        Negative stains
        Sample pathology report
        • Stomach, antrum, endoscopic submucosal dissection:
          • Plexiform fibromyoxoma, completely excised (see comment)
          • Comment: There is a bland spindle cell proliferation within myxoid stroma. Prominent vasculature transverses the lesion. No significant pleomorphism is noted. No mitotic figures or necrosis are noted. The tumor cells are positive for smooth muscle actin but negative for CD34, c-kit / CD117, desmin, S100 and DOG1. The overall histology and immunophenotype are most consistent with plexiform fibromyxoma. Metastases and recurrence following complete excision of plexiform fibromyxoma have not been reported.
        Differential diagnosis
        • GIST:
        • Inflammatory fibroid polyp:
          • Primarily arises in the submucosa
          • Has more inflammation and often demonstrates a concentric growth of spindle cells around small blood vessels
          • In most cases, the spindle cells are CD34 positive
        • Neurofibroma:
          • Neurofibroma can demonstrate a plexiform growth pattern
          • Consists of multiple cell types including S100 positive schwann cells intermixed with CD34 positive spindled fibroblasts and EMA positive perineurial cells
        • Schwannoma:
          • Comprises diversely arranged tumor cells that often form a microtrabecular structure against a background of collagen
          • Characteristic histological features include intralesional lymphoplasmacytic inflammation and lymphoid cuff with infiltrative margin
          • Schwannoma cells are S100 and SOX10 positive
        • Inflammatory myofibroblastic tumor (IMT):
          • Often affects children or young adults
          • Most common locations are mesentery and omentum
          • IMT comprises spindled to stellate myofibroblastic cells with a lymphoplasmacytic background
          • About 50% of cases harbor rearrangements of anaplastic lymphoma kinase (ALK) gene and thus are positive for ALK by immunohistochemistry
        • Fibromatosis:
        Board review style question #1

        Which of the following is true about plexiform fibromyxoma?

        1. Complete resection of the lesion provides cure
        2. Metastases are common after complete resection
        3. Most plexiform fibromyxomas are malignant
        4. Plexiform fibromyxoma only occurs in the stomach
        5. Recurrent translocation of genes have not been reported
        Board review style answer #1
        A. Complete resection of the lesion provides cure

        Comment Here

        Reference: Plexiform fibromyxoma
        Board review style question #2
        A 71 year old woman presented with vague upper abdominal pain. EGD examination reveals a 4.5 cm mural mass in the antrum. The patient underwent partial gastrectomy. Gross examination reveals a multilobular mural mass. The tumor shows a bland spindle cell proliferation in a variably myxoid stroma. The tumor cells are positive for smooth muscle actin but negative for CD34, CD117, desmin, ALK, S100 or DOG1. Which of the following is most likely the correct diagnosis?

        1. Gastrointestinal stromal tumor
        2. Inflammatory fibroid polyp
        3. Neurofibroma
        4. Plexiform fibromyxoma
        5. Schwannoma
        Board review style answer #2
        D. Plexiform fibromyxoma

        Comment Here

        Reference: Plexiform fibromyxoma

        Poorly differentiated neuroendocrine carcinoma
        Definition / general
        • Mean age 70 years (range 44 - 92 years), 70% men
        • Poorer prognosis than nonneuroendocrine carcinomas
        • May have a poor outcome in MEN1 patients (Am J Surg Pathol 1997;21:1075)
        • Small cell and large cell neuroendocrine carcinomas have similar prognosis (Arch Pathol Lab Med 1998;122:1010)
        • Small cell variant resembles counterpart in lung; has aggressive clinical course but treatment may be effective (J Surg Oncol 2012;106:994)
        • Most gastric carcinomas contain scattered cells (small percentage of tumor) with neuroendocrine differentiation (Arch Pathol Lab Med 1996;120:478) - don't call them neuroendocrine carcinomas
        Case reports
        Gross description
        • Ulcerated or fungated
        Microscopic (histologic) description
        • Solid, organoid, trabecular, pseudoglandular, spindle cell or rosette-like patterns
        • Small cell or large cell patterns
        Microscopic (histologic) images

        Images hosted on other servers:

        Small, round
        lymphocyte-like
        cells with hyper-
        chromatic nuclei


        Portal hypertensive gastropathy
        Definition / general
        • Portal hypertensive gastropathy (PHG) refers to the pathologic effects of increased portal venous pressure on the mucosal surface of the stomach
        Essential features
        • Portal hypertensive gastropathy is mainly a clinical and endoscopic diagnosis; histologic findings should be correlated with the clinical diagnosis
        • Histologic features are nonspecific and show congested capillaries and venules in the gastric mucosa, mainly in the body and fundus, without significant inflammation or fibrin thrombi
        Terminology
        • Congestive gastropathy: historic term first described by McCormack et al. in 1985 (Gut 1985;26:1226)
        ICD coding
        • ICD-10: K31.89 - other diseases of stomach and duodenum
        Epidemiology
        • 20 - 98% prevalence in patients with liver cirrhosis (Gastroenterology 2000;119:181)
        • Wide range of prevalence is related to the variability of the characterization criteria of the disease
        • Risk factors:
          • Schistosomiasis
          • Heart failure
        Sites
        Pathophysiology
        • Poorly understood
        • Portal hypertension: increased pressure in high resistance venous system
        • Backing up of blood in the collateral circulation of the gastrointestinal tract, including the proximal stomach (venous congestion and bleeding)
        • Vascular and mucosal changes in the stomach
        • Additional theories include mucosal injury, hypoxia, cytokine production and inflammation (Gastroenterology 1992;102:2066)
        Etiology
        • Portal hypertension:
          • Increased sinusoidal and portal venous pressure (cirrhosis)
          • Presinusoidal diseases, like portal vein thrombosis and veno-occlusive disorders
          • Postsinusoidal etiology, like heart failure
        • No well documented correlation between severity of portal hypertension and development of portal hypertensive gastropathy (Gastroenterology 1992;102:994)
        Clinical features
        • Patients with portal hypertensive gastropathy usually present with chronic gastrointestinal bleeding that can result in iron deficiency anemia (Gastroenterology 2000;119:181)
        • Acute gastrointestinal bleeding is less common
        • Portal hypertensive gastropathy can be associated with esophageal varices
        • Classification systems are based on endoscopic findings
        Diagnosis
        • Clinical: association with cirrhosis
        • Upper gastrointestinal endoscopy (i.e., EGD) findings:
          • Mild cases: snake skin, mosaic-like pattern of stomach mucosa
          • Severe cases: bulging red to brown marks
          • Specificity of endoscopic findings for portal hypertensive gastropathy diagnosis were reported as follows (World J Gastrointest Endosc 2013;5:323):
            • Mosaic-like pattern: 100% sensitivity and 92% specificity
            • Red point lesions: 48% sensitivity and 91% specificity
            • Cherry red spots: 39% sensitivity and 96% specificity
            • Confluent red marks: 22% sensitivity and 100% specificity
        • Capsule endoscopy:
        • Radiologic findings (double contrast upper gastrointestinal imaging study):
        Radiology description
        Radiology images

        Images hosted on other servers:

        Gastric serpentine mucosal folds

        Prognostic factors
        • Poor prognosis is associated with severe cases presenting with acute bleeding
        • Rarely fatal bleeding has been reported (J Clin Diagn Res 2016;10:HD01)
        • Cirrhotic patients with severe portal hypertensive gastropathy associated with higher portal pressure had worse prognosis than patients with mild portal hypertensive gastropathy (Dig Dis Sci 2010;55:3561)
        Case reports
        Treatment
        • Iron supplementation for chronic gastrointestinal bleeding
        • Beta blockers and octreotide to decrease portal blood pressure
        • Transjugular intrahepatic portosystemic shunt (TIPS) procedure (Hepatology 1995;21:1011)
        • Endoscopic therapy (sclerotherapy or coagulation therapy):
        Clinical images

        Contributed by Michael Schoech, M.D.

        Endoscopy: mosaic-like pattern

        Microscopic (histologic) description
        • No specific histologic diagnostic criteria for portal hypertensive gastropathy
        • Histologic diagnosis is correlated with clinical history (portal hypertension) and endoscopic findings
        • Prominent congested blood vessels (capillaries and venules) in the lamina propria
        • Lamina propria edema or fibrosis
        • Congested, dilated and tortuous mucosal and submucosal venules (Gut 1985;26:1226)
        • Features of reactive gastropathy can be seen
        • The above features can only be seen in the submucosa; with variable degrees, a normal gastric biopsy does not exclude the clinical diagnosis of portal hypertensive gastropathy (Am J Clin Pathol 2022;158:632)
        • Absence of significant inflammation and fibrin thrombi
        Microscopic (histologic) images

        Contributed by Divya Sharma, M.D.

        Congestion and reactive gastropathy

        Congested blood vessels

        Lamina propria fibrosis

        Ectatic blood vessels

        Sample pathology report
        • Stomach, biopsy:
          • Portal hypertensive gastropathy (see comment)
          • Comment: Mild reactive gastropathy with mucosal congestion and dilated vessels within lamina propria is consistent with portal hypertensive gastropathy.
          • Negative for intestinal metaplasia, dysplasia or malignancy.
        Differential diagnosis
        Board review style question #1
        A 58 year old man with a history of hepatitis C cirrhosis and prior bleeding varices presented with chronic chest pain, dyspepsia and anemia. Esophagogastroduodenoscopy (EGD) showed mucosal red spots in the stomach fundus. Which of the following histologic features is expected to be noted on biopsy?

        1. Active chronic inflammation
        2. Congested capillaries in the lamina propria without significant inflammation
        3. Ectatic blood vessels and fibrin thrombi
        4. Extensive intestinal metaplasia
        Board review style answer #1
        B. Congested capillaries in the lamina propria without significant inflammation. The clinical history suggests portal hypertension (cirrhosis and bleeding varices), which raises concern of portal hypertensive gastropathy. EGD findings show mucosal red spots or mosaic pattern that support the diagnosis.

        Comment Here

        Reference: Portal hypertensive gastropathy
        Board review style question #2

        The figure above is from a stomach biopsy in a 63 year old woman with history of recurrent deep vein thrombosis. Esophagogastroduodenoscopy (EGD) shows mosaic-like pattern of the stomach mucosa without bleeding. Which of the following is a possible etiology of the endoscopic findings?

        1. Atrophic gastritis
        2. Chronic hypertension
        3. Inflammatory bowel disease
        4. Portal vein thrombosis
        Board review style answer #2
        D. Portal vein thrombosis. The patient history and the EGD findings of the stomach suggest portal hypertension due to veno-occlusive disorder. Portal vein thrombosis can result in backing up of collateral circulation in the upper gastrointestinal tract causing portal hypertensive gastropathy. The biopsy of the stomach shows diffuse congestion of mucosal capillaries, supporting portal hypertensive gastropathy.

        Comment Here

        Reference: Portal hypertensive gastropathy

        Proton pump inhibitors
        Definition / general
        • Includes omeprazole, lansoprazole, dexlansoprazole, esomeprazole, rabeprazole
        • Used to treat gastroesophageal reflux disease (GERD)
        • Patients may develop G cell and enterochromaffin cell-like hyperplasia, secondary to drug induced achlorhydria
        • Long-term therapy associated with fundic gland polyps (sessile, multiple, < 1.0 cm) in 17% after 3 months, in 35% after 5 months treatment
        • Polyps regress when treatment stops (Hum Pathol 2000;31:684)
        Microscopic (histologic) description
        • Glandular luminal dilatation with swelling and bulging of the superficial cytoplasm of parietal cells (so-called parietal cell protrusion)
        • Glands lined by cells with serrated rather than a smooth border

        Pyloric gland adenoma
        Definition / general
        • Grossly visible neoplastic lesion in the stomach, wherein the native glands are replaced by closely packed pyloric glands lined by cuboidal to low columnar epithelium
          • 40 - 50% associated with high grade dysplasia
        Essential features
        • Closely packed tubular glands lined by cuboidal to low columnar epithelium with ground glass cytoplasm, bland nuclei and no well formed apical mucin cap
        • 60% observed in corpus of the stomach, followed by cardia and antrum
        • Associated with autoimmune gastritis, familial adenomatosis polyposis and Lynch syndrome
        • Commonly associated with dysplasia
        Terminology
        • Pyloric gland tubular adenoma; gastric differentiated adenoma (German); pyloric gland intracystic papillary neoplasm (J Clin Pathol 2014;67:883)
        ICD coding
        • ICD-O: 8210/0 - adenomatous polyp
        • ICD-10: K31.7 - polyp of stomach and duodenum
        • ICD-11: 2E92.1&XH3DV3 - benign neoplasm of stomach and adenoma NOS
        Epidemiology
        • Accounts for 3% of all gastric epithelial polyps (Virchows Archive 2003;442:317)
        • Slightly more common in females as compared with males (Am J Surg Pathol 2009;33:186)
        • Mean age: 70 years
        • No predilection for a specific geographic location
        • Strongly associated with autoimmune gastritis; some association with familial adenomatosis polyposis
        Sites
        • In stomach, most commonly seen in fundus / corpus, followed by cardia and antrum
        • Other sites: reported frequencies are variable in different studies; however, one of the largest series of 373 cases of pyloric gland adenoma (PGA) reports the following frequencies - duodenum including duodenal bulb (11%), gallbladder (4.3%), pancreas, bile duct (1.4%), pancreatic duct (0.3%), rectum (1.1%), esophagus (2.4%), cervix (rare) (J Clin Pathol 2014;67:883)
        Pathophysiology
        • Pyloric gland metaplasia (in the stomach) / gastric heterotopia (elsewhere) is the precursor lesion of pyloric gland adenoma (J Clin Pathol 2014;67:883)
        Etiology
        Clinical features
        • Usually asymptomatic but may present with gastric distention, epigastric pain or heartburn
        • Background mucosa is that of autoimmune gastritis or Helicobacter pylori gastritis
        • Its predominance in females reflects its association with autoimmune gastritis
        • Sporadic PGA tends to occur in younger patients and without any inflammatory background, sometimes in association with fundic gland polyp
        Diagnosis
        • Diagnosis is made by endoscopy and histology
        Prognostic factors
        • Since it is associated with high grade dysplasia and gastric adenocarcinoma, all PGAs should be resected
        • Prognosis depends on the associated dysplasia and adenocarcinoma
        • The neoplastic risk increases with the associated autoimmune gastritis, larger mean polyp size and tubulovillous architecture; however, after a complete endoscopic resection, the rate of local recurrence is low (< 10%) (Histopathology 2018;72:1007)
        Case reports
        Treatment
        • Endoscopic mucosal resection, surgical resection
        Clinical images

        Contributed by Supriya Srivastava M.D., Ph.D.
        Endoscopic finding

        Endoscopic appearance



        Images hosted on other servers:

        Endoscopy

        Gross description
        • Polyp, sessile or pedunculated or masses ranging from 1 - 10 cm in size
        • Usually single
        Gross images

        Images hosted on other servers:

        Intraoperative image

        Microscopic (histologic) description
        • Closely packed tubular glands (pyloric glands / pseudopyloric glands) with low cuboidal to columnar epithelium; cytoplasm is eosinophilic with ground glass appearance; round nuclei, basally located without any prominent nucleoli
        • PGA can occur without dysplasia, with low grade dysplasia and with high grade dysplasia; PGA may also be associated with adenocarcinoma (12 - 47%) but submucosal invasion is observed in fewer than 10% of cases
        • PGA without dysplasia: small tubular glands, closely packed, lined by bland cuboidal to columnar epithelium with basally arranged round nuclei with inconspicuous nucleoli
        • PGA with low grade dysplasia: slightly irregularly arranged tubular glands with slightly elongated nuclei with mild hyperchromasia with some stratification
        • PGA with high grade dysplasia: architectural distortion such as irregularly arranged occasional cribriform glands with nuclear crowding, loss of polarity, vesicular nuclei and prominent nucleoli and few mitoses
        • PGA associated with adenocarcinoma: invasive irregular complex glands, loss of nuclear polarity, high nuclear:cytoplasmic ratio, vesicular nuclei with irregular nuclear membranes and numerous mitoses
        Microscopic (histologic) images

        Contributed by Supriya Srivastava M.D., Ph.D.
        Antral polyp

        Antral polyp

        PGA without dysplasia

        PGA without dysplasia

        MUC5AC staining

        MUC5AC staining

        MUC6 staining

        MUC6 staining

        MUC2 staining

        MUC2 staining


        PGA with dysplasia

        High grade dysplasia

        High grade dysplasia

        Low grade dysplasia

        Low grade dysplasia

        PGA with and without dysplasia

        PGA with and without dysplasia

        Positive stains
        Negative stains
        Molecular / cytogenetics description
        Sample pathology report
        • Stomach, polypectomy:
          • Pyloric gland adenoma (see comment)
          • Comment: Histology shows tightly packed pyloric glands lined by cuboidal or low columnar epithelium with ground glass eosinophilic cytoplasm, basally located nuclei and absent apical mucin.
        Differential diagnosis
        Board review style question #1

        A polypoid lesion of the stomach was resected in a 75 year old man. Based on the histological feature provided in the photomicrograph above, which of the following statements is correct?

        1. Cells have an apical mucin cap
        2. GNAS mutation is commonly observed
        3. MUC2 expression is frequent in this lesion
        4. TP53 mutations are commonly seen
        Board review style answer #1
        B. KRAS and GNAS mutations are commonly seen in pyloric gland adenoma

        Comment Here

        Reference: Pyloric gland adenoma
        Board review style question #2
        Which of the following statements is true about pyloric gland adenoma?

        1. It is most commonly observed in antrum of the stomach
        2. It is commonly observed in a background of autoimmune gastritis
        3. It is rarely associated with dysplasia
        4. PASD is extensively positive in the neoplastic glands
        Board review style answer #2
        B. Pyloric gland adenoma is commonly observed in a background of autoimmune gastritis

        Comment Here

        Reference: Pyloric gland adenoma

        Pyloric stenosis
        Definition / general
        • Congenital: common congenital abnormality (1 per 300-900 births)
          • 75% male, onset at 3-12 weeks, high concordance in monozygotic twins, associated with Turner syndrome, trisomy 18, esophageal atresia
        • Acquired: 80% men, hypertrophy of pyloric circular muscle fibers that ends at duodenum
          • Associated with antral gastritis or pyloric ulcer
        Clinical features
        • Congenital: persistent projectile non-bilious vomiting in second week of life; regurgitation, visible peristalsis
        Treatment
        • Congenital: pyloromyotomy (splitting of pyloric muscle)
        Gross description
        • Congenital: thickened pyloric muscle resembling a fusiform mass, 3-5 cm, that occludes the pyloric channel
        Gross images

        Image hosted on other server:

        Thickened muscularis propria

        Microscopic (histologic) description
        • Congenital: edema and inflammatory changes in mucosa or submucosa; thickening primarily of circular muscle, which terminates abruptly distally
        Differential diagnosis

        Reactive (chemical) gastropathy
        Definition / general
        • Gastric mucosal pattern of injury induced by endogenous (bile / pancreatic reflux) or exogenous (chronic use of nonsteroidal anti-inflammatory drugs [NSAIDs], acetylsalicylic acid, ethanol or chemotherapeutics) agents (Pathologe 2001;22:44)
        • Second most common diagnosis on gastric biopsy (Arch Pathol Lab Med 2007;131:86)
        Essential features
        • Gastric epithelial pattern of injury in response to an endogenous or exogenous irritant
        • Histology is characterized by foveolar hyperplasia, mucin depletion in surface epithelial cells and lamina propria expansion by fibromuscular and capillary hyperplasia
        • No prominent inflammatory cells on histology
        Terminology
        ICD coding
        • ICD-10: K31.9 - disease of stomach and duodenum, unspecified
        Epidemiology
        Sites
        • Gastric surface epithelium
        • Antral mucosa > oxyntic mucosa, typically
        Pathophysiology
        • Insult to gastric surface epithelium results in excessive cellular exfoliation, leading to foveolar hyperplasia
        • NSAID induced:
          • Permeation of gastric cell membranes, due to their nature as weak organic acids, allows direct access to mucosal lining
          • Inhibition of cyclooxygenase enzymes prevents synthesis of prostaglandin from arachidonic acid and production of leukotrienes
          • Disruption of cellular proliferative / reparative processes, microcirculation and growth factor expression
          • Reduced secretion of bicarbonate and mucin results in diminished hydrophobic nature of gastric surface mucous gel layer
          • References: Arch Pathol Lab Med 2007;131:86, Mediators Inflamm 2013;2013:258209
        • Bile reflux induced:
        Etiology
        Clinical features
        • Potential indigestion, epigastric discomfort, diarrhea, bloating, nausea, vomiting
        • May be asymptomatic
        • Minor correlation with chronic inactive gastritis, Helicobacter negative chronic active gastritis and intestinal metaplasia
        • Patients with reactive gastropathy are more likely to have duodenal peptic injury and intraepithelial lymphocytosis
        • Other significant associations: H. pylori gastritis, active ileitis, focal active colitis and collagenous colitis (Aliment Pharmacol Ther 2012;36:736)
        • High rates of concurrent conditions in certain ethnic groups: Hispanics - diabetes, African Americans - hypertension, Caucasians - tobacco / alcohol use (Gastrointest Tumors 2021;8:115)
        Postchemotherapy / radiation therapy changes
        Diagnosis
        • Endoscopic observation of mucosal erythema or friability
        • Endoscopic biopsies of gastric antrum or corpus
        • Definitive diagnosis via identification of histopathologic criteria (Pathol Res Pract 2014;210:847)
        Prognostic factors
        • Prognosis is good and recurrence rate is generally low with removal of offending agent
        • If gastropathy is sequela of gastrectomy, it may contribute to pathogenesis of stump carcinoma / remnant gastric cancer (Eur J Gastroenterol Hepatol 2003;15:35)
        Case reports
        Treatment
        • Removal of offending agent
        • Prevention via combination therapy of NSAIDs with gastroprotective agent(s) or alternative use of selective COX2 inhibitors (Mediators Inflamm 2013;2013:258209)
        Clinical images

        Images hosted on other servers:

        Bile with erythematous mucosa

        Erythema / edema
        after choledocho-
        duodeno
        anastomosis

        Gross description
        Microscopic (histologic) description
        • Lack of significant edema or inflammation
        • Foveolar hyperplasia: tortuosity / corkscrew appearance with a minimum twofold elongation of gastric cardiac foveolae and pits
        • Mucin poor columnar cells with enlarged, hyperchromatic nuclei
        • Lamina propria expansion with fibromuscular proliferation and congested, ectatic capillaries
        • Chemotherapy related changes
          • More epithelial damage and apoptosis but often no effect (Arch Pathol Lab Med 1991;115:807)
          • May have large hyperchromatic nuclei that can mimic virally infected cells or bizarre nuclear atypia
          • May see reduction in tumor cellularity, increase in dense fibrosis, formation of large mucin pools with lymphocytes and macrophages
          • No atypical mitoses, no cytoplasmic eosinophilia, no vacuolization, no intestinal metaplasia, no irregular glandular microcystic change
          • In signet ring carcinomas, see smaller intracytoplasmic mucin vacuoles
        • Radiation related changes
          • Ectatic vessels and lamina propria fibrosis
        • References: Gastrointest Tumors 2021;8:115, J Clin Pathol 2002;55:352, Arch Pathol Lab Med 2007;131:86
        Microscopic (histologic) images

        Contributed by Aaron R. Huber, D.O.
        Reactive epithelium

        Reactive epithelium

        Epithelial injury without inflammation

        Epithelial injury without inflammation

        Negative stains
        Videos

        Gastritis versus gastropathy

        Sample pathology report
        • Stomach, antrum, biopsy:
          • Reactive (chemical) gastropathy
          • No Helicobacter organisms identified on routine H&E stained sections
        Differential diagnosis
        Board review style question #1

        A 48 year old woman with rheumatoid arthritis presents with dyspepsia and bloating. Endoscopy shows erythematous mucosa. Histopathology is significant for serrated gastric pits, foveolar hyperplasia, paucity of mucin in surface epithelial cells and ectatic superficial capillaries (see image). What is the diagnosis?

        1. Gastric antral vascular ectasia
        2. H. pylori gastritis
        3. Low grade dysplasia
        4. Reactive gastropathy
        Board review style answer #1
        D. Reactive gastropathy. Histology of low grade dysplasia includes intestinal metaplasia, mucosal atrophy and pencillate, hyperchromatic nuclei within tall columnar epithelium. Gastric vascular ectasia will also have distended capillaries but with fibrin thrombi and without foveolar hyperplasia. Histology of H. pylori gastritis should emphasize more inflammation, especially epithelial neutrophils and plasma cells in the lamina propria.

        Comment Here

        Reference: Reactive (chemical) gastropathy
        Board review style question #2
        A patient suffers severe reflux following sleeve gastrectomy and is diagnosed with a gastric injury pattern characterized by papillary foveolar extension, expanded lamina propria lacking inflammation and mucin depleted superficial cells. What condition does this potentially predispose her to?

        1. Gastric mucosal atrophy
        2. Gastric outlet obstruction
        3. Gastric ulcer
        4. Remnant gastric cancer
        Board review style answer #2
        D. Remnant gastric cancer. The other answer options are more closely associated with chronic gastritis. The described histology is classic for reactive (chemical) gastropathy, which can contribute to the pathogenesis of stump / remnant gastric cancer.

        Comment Here

        Reference: Reactive (chemical) gastropathy

        Russell body gastritis
        Definition / general
        • Russell body gastritis, first described in 1998, is a reactive gastric mucosal infiltration of plasma cells filled with cytoplasmic Russell bodies (Am J Hematol 2010;85:951)
        • Recently recognized pseudotumoral lesion of the gastric mucosa with unknown etiology; characterized by the accumulation of numerous plasma cells containing Russell bodies (RBs) with the expression of kappa and lambda light chains (J Gastrointestin Liver Dis 2012;21:97)
        • Often associated with H. pylori infection and malignancies such as lymphoplasmacytic lymphoma, plasmacytoma, signet ring cell carcinoma (Pathol Res Pract 2007;203:457, Virchows Arch 2005;446:463)
        Essential features
        • Russell bodies, first described by Russell in 1890, are considered to represent aggregates of immunoglobulins resulting from secretory disturbance of plasma cells
        • Plasma cells with Russell bodies can be seen in a variety of mucosal surfaces, including gastrointestinal tract in association with chronic inflammation such as H. pylori gastritis.
        ICD coding
        • 2017 ICD-10-CM Diagnosis Code K29.70 - Gastritis, unspecified, without bleeding
        Epidemiology
        • Only 39 cases involving the gastrointestinal tract have been reported in English literature, which include Russell body gastritis, duodenitis and esophagitis (Int J Surg Pathol 2015;23:667)
        Sites
        Pathophysiology
        • Russell bodies represent a cellular response to overstimulation of plasma cells, leading to the accumulation of abundant, nondegradable, condensed immunoglobulin in dilated rough endoplasmic reticulum cisternae (Int J Surg Pathol 2015;23:667)
        • Considering the other cases reported in the literature, it seems as if H. pylori plays an active role in the etiology of the condition, either by promoting it or by modulating the extension of the disease locally (J Gastrointestin Liver Dis 2012;21:97)
        Clinical features
        • Asymptomatic or symptoms related to H. pylori or associated malignancy
        Diagnosis
        • Histology showing diffuse infiltration with plasma cell filled with Russell bodies
        • Exclusion of other associated conditions
        Radiology description
        Prognostic factors
        • Although RBG is by itself a benign condition, its long term effect, such as its possible increased risk for the development of neoplasia, is unknown (J Gastrointestin Liver Dis 2012;21:97)
        Case reports
        Treatment
        • Treatment of H. pylori usually results in the resolution of Russel bodies in cases associated with H. pylori
        Gross description
        Gross images

        Images hosted on other servers:

        Large, irregular and deep
        ulcerated mass lesion
        with central ulceration
        at the incisura angularis

        Microscopic (histologic) description
        • Plasma cell infiltrates with extensive formation of Russell bodies (Mott cells) within the lamina propria of the antral mucosa
        • Intracytoplasmic inclusions, homogeneous, mainly round to oval, which push the nucleus toward the periphery
        Microscopic (histologic) images

        Case #410

        H&E


        Positive for CD138 stain

        Positive stains
        Negative stains

        Schwannoma
        Definition / general
        • Well circumscribed, benign nerve sheath tumor (also called neurilemmoma), much less common (~ 1 / 50) in stomach than GIST
        Case reports
        • 37 year old man with incidentally detected gastric mass (Case #455)
        • 46 year old woman with stomach mass (Case #347)
        Microscopic (histologic) description
        • Well circumscribed but not encapsulated, with interlacing bundles of spindle cells and collagen
        • Often nuclear pallisading, Verocay bodies, hyalinized vessels
        • May have nuclear atypia, inflammatory cells, peripheral cuff of lymphoid aggregates
        • No mitotic figures, no skenoid fibers, no epithelioid features
        Microscopic (histologic) images

        Contributed by @RaulSGonzalezMD on Twitter and Cases #455 and #347
        Schwannoma Schwannoma Schwannoma Schwannoma

        Schwannoma



        Microcystic variant of schwannoma in stomach


        S100, microcystic variant

        CD117, microcystic variant

        46 year old woman

        Positive stains
        Negative stains
        Board review style question #1
        Which of the following describes the majority of schwannomas arising in the gastrointestinal tract?

        1. Both encapsulated and circumscribed
        2. Circumscribed but not encapsulated
        3. Encapsulated but not circumscribed
        4. Neither encapsulated nor circumscribed
        Board review style answer #1
        B. Circumscribed but not encapsulated

        Comment Here

        Reference: Schwannoma

        Staging-carcinoma
        Pathologic TNM staging of carcinomas of the stomach, AJCC 8th edition
        Definition / general
        • Includes gastric carcinoma, gastric neuroendocrine carcinoma, gastroesophageal junction carcinoma with an epicenter more than 2 cm into the stomach, cardia carcinoma
        Essential features
        ICD coding
        • C16.9: Malignant neoplasm of stomach, unspecified
        Primary tumor (pT)
        • TX: Primary tumor cannot be assessed
        • T0: No evidence of primary tumor
        • Tis: Carcinoma in situ: intraepithelial tumor without invasion of the lamina propria; high grade dysplasia
        • T1: Tumor invades the lamina propria, muscularis mucosae or submucosa
          • T1a: Tumor invades the lamina propria or muscularis mucosae
          • T1b: Tumor invades the submucosa
        • T2: Tumor invades the muscularis propria
        • T3: Tumor penetrates the subserosal connective tissue without invasion of the visceral peritoneum or adjacent structures
        • T4: Tumor invades the serosa (visceral peritoneum) or adjacent structures
          • T4a: Tumor invades the serosa (visceral peritoneum)
          • T4b: Tumor invades adjacent structures/organs
        Regional lymph nodes (pN)
        • NX: Regional lymph nodes cannot be assessed
        • N0: No regional lymph node metastasis
        • N1: Metastasis in one or two regional lymph nodes
        • N2: Metastasis in three to six regional lymph nodes
        • N3: Metastasis in seven or more regional lymph nodes
          • N3a: Metastasis in seven to 15 regional lymph nodes
          • N3b: Metastasis in 16 or more regional lymph nodes

        Note:
        • Regional lymph nodes include the greater curvature, greater omental, lesser curvature, lesser omental, right and left peracardial (cardioesophageal), suprapyloric, gastroduodenal, infrapyloric, gastroepiploic, left gastric artery, celiac artery, common hepatic artery, hepatoduodenal, portal, splenic artery, and splenic hilum nodes
        Distant metastasis (pM)
        • M0: No distant metastasis
        • M1: Distant metastasis
        Prefixes
        • y: preoperative radiotherapy or chemotherapy
        Stage grouping
        Clinical staging
        Stage 0: Tis N0 M0
        Stage I: T1-2 N0 M0
        Stage IIA: T1-2 N1-3 M0
        Stage IIB: T3-4a N0 M0
        Stage III: T3-4a N1-3 M0
        Stage IVA: T4b N1-3 M0
        Stage IVB: any T any N M1


        Pathologic staging
        Stage 0: Tis N0 M0
        Stage IA: T1 N0 M0
        Stage IB: T1 N1 M0
        T2 N0 M0
        Stage IIA: T1 N2 M0
        T2 N1 M0
        T3 N0 M0
        Stage IIB: T1 N3a M0
        T2 N2 M0
        T3 N1 M0
        T4a N0 M0
        Stage IIIA: T2 N3a M0
        T3 N2 M0
        T4a N1-2 M0
        T4b N0 M0
        Stage IIIB: T1-2 N3b M0
        T3-4a N3a M0
        T4b N1-2 M0
        Stage IIIC: T3-4a N3b M0
        T4b N3a-3b M0
        Stage IV: any T any N M1


        Pathologic staging following neoadjuvant therapy
        Stage I: T1-2 N0 M0
        T1 N1 M0
        Stage II: T3-4a N0 M0
        T2-3 N1 M0
        T1-2 N2 M0
        T1 N3 M0
        Stage III: T4b N0 M0
        T4a-4b N1 M0
        T3, 4a, 4b N2 M0
        T2, 3, 4a, 4b N3 M0
        Stage IV: any T any N M1
        Registry data collection variables
        • Tumor location
        • Serum CEA
        • Serum CA 19-9
        • Clinical staging modalities (endoscopy and biopsy, EUS, EUS-FNA, CT, PET/CT)
        • Tumor length
        • Depth of invasion
        • Number of suspicious malignant lymph nodes on baseline radiologic images
        • Number of suspicious malignant lymph nodes by EUS assessment
        • Location of suspicious nodes (clinical)
        • Location of suspicious nodes (pathological)
        • Number of tumor deposits
        • Lymphovascular invasion
        • Neural invasion
        • Extranodal extension
        • HER2 status (positive or negative)
        • Microsatellite instability (MSI) status
        • Surgical margin (negative, microscopic, macroscopic)
        • Sites of metastasis, if applicable
        • Type of surgery
        Histologic grade
        • GX: Grade cannot be assessed
        • G1: Well differentiated
        • G2: Moderately differentiated
        • G3: Poorly differentiated
        Histopathologic type
        • Intraepithelial neoplasia (dysplasia), high grade
        • Adenocarcinoma, NOS
        • Adenocarcinoma, intestinal type
        • Carcinoma, diffuse type
        • Papillary adenocarcinoma
        • Tubular adenocarcinoma
        • Mucinous adenocarcinoma
        • Parietal cell carcinoma
        • Signet ring cell carcinoma
        • Mixed adenocarcinoma
        • Adenosquamous carcinoma
        • Carcinoma with lymphoid stroma (medullary carcinoma)
        • Hepatoid adenocarcinoma
        • Squamous cell carcinoma, NOS
        • Lymphoepithelial carcinoma
        • Medullary carcinoma, NOS
        • Undifferentiated carcinoma
        • Neuroendocrine carcinoma (NEC)
        • Large cell neuroendocrine carcinoma (NEC)
        • Small cell neuroendocrine carcinoma (NEC)
        • Mixed adenoneuroendocrine carcinoma
        Board review style question #1
          Based on AJCC 8th edition criteria, a tumor located within the gastric cardia but involving the gastroesophageal junction would be staged as an esophageal carcinoma if its epicenter is no more than what distance from the gastroesophageal junction?

        1. 2 cm
        2. 3 cm
        3. 4 cm
        4. 5 cm
        Board review style answer #1
        A. 2 cm

        Comment Here

        Reference: Staging-carcinoma

        Staging-GIST

        Pathologic TNM staging of gastrointestinal stromal tumors, AJCC 8th edition
        Definition / general
        • Sporadic nonpediatric gastrointestinal stromal tumors (GISTs) of any gastrointestinal organ are covered by this staging system
        • Pediatric, familial and syndromic GISTs are not staged using AJCC criteria
        Essential features
        ICD coding
        • C49.A0: Gastrointestinal stromal tumor, unspecified site
        Primary tumor (pT)
        • TX: Primary tumor cannot be assessed
        • T0: No evidence of primary tumor
        • T1: Tumor ≤ 2 cm
        • T2: Tumor > 2 cm but ≤ 5 cm
        • T3: Tumor > 5 cm but ≤ 10 cm
        • T4: Tumor > 10 cm
        Regional lymph nodes (pN)
        • N0: No regional lymph node metastasis or unknown lymph node status
        • N1: Regional lymph node metastasis

        Notes:
        • Nodal metastasis is very rare in sporadic nonpediatric GISTs
        • Regional lymph nodes vary depending on site of origin
        Distant metastasis (pM)
        • M0: No distant metastasis
        • M1: Distant metastasis
        Mitotic rate
        • Low: 5 or fewer mitoses per 5 mm2
        • High: More than 5 mitoses per 5 mm2
        Prefixes
        • y: Preoperative radiotherapy or chemotherapy
        • r: Recurrent tumor stage
        Stage grouping
        Gastric and omental GIST
        Stage IA: T1 - 2 N0 M0 Low mitotic rate
        Stage IB: T3 N0 M0 Low mitotic rate
        Stage II: T1 - 2 N0 M0 High mitotic rate
        T4 N0 M0 Low mitotic rate
        Stage IIIA: T3 N0 M0 High mitotic rate
        Stage IIIB: T4 N0 M0 High mitotic rate
        Stage IVA: Any T N1 M0 Any mitotic rate
        Any T Any N M1 Any mitotic rate

        Esophageal, small intestinal, colorectal, mesenteric and peritoneal GIST
        Stage I: T1 - 2 N0 M0 Low mitotic rate
        Stage II: T3 N0 M0 Low mitotic rate
        Stage IIIA: T4 N0 M0 Low mitotic rate
        T1 N0 M0 High mitotic rate
        Stage IIIB: T2 - 4 N0 M0 High mitotic rate
        Stage IVA: Any T N1 M0 Any mitotic rate
        Any T Any N M1 Any mitotic rate
        Registry data collection variables
        • Tumor size
        • Tumor site (esophagus, stomach, duodenum, jejunum / ileum, rectum, extraintestinal)
        • Tumor mitotic rate
        • Tumor rupture
        • Tumor metastasis (liver, peritoneum, other)
        • Tumor KIT immunohistochemistry
        • Tumor mutational status of KIT and PDGFRA (if known)
        Histologic grade
        • Same as mitotic rate (see above)
        Histopathologic type
        • Spindle cell, epithelioid and mixed cell types
        Survival data
        Gastric GISTs
        Stage Tumor size (cm) Mitotic rateObserved rate of
        progressive disease
        Stage IA: ≤ 5 Low 0 - 2%
        Stage IB: > 5 - 10 Low 3 - 4%
        Stage II: ≤ 2 High Insufficient data
        > 2 - 5 High 16%
        > 10 Low 12%
        Stage IIIA: > 5 - 10 High 55%
        Stage IIIB: > 10 High 86%

        Small intestinal GISTs
        Stage Tumor size (cm) Mitotic rateObserved rate of
        progressive disease
        Stage IA: ≤ 5 Low 0 - 4%
        Stage II: > 5 - 10 Low 24%
        Stage IIIA: ≥ 10 Low 52%
        ≤ 2 High 50%
        Stage IIIB: > 2 - 5 High 73%
        > 5 - 10 High 85%
        > 10 High 90%
        Board review style question #1
        Using AJCC 8th edition criteria, which of the following neoplasms may be staged as pN0 even if no lymph nodes are received for pathologic examination?

        1. Appendiceal goblet cell carcinoid
        2. Gastric neuroendocrine carcinoma
        3. Gastrointestinal stromal tumor
        4. Rectal adenocarcinoma with neoadjuvant treatment
        Board review style answer #1
        C. Gastrointestinal stromal tumor

        Comment Here

        Reference: Staging-GIST

        Staging-neuroendocrine tumors

        Pathologic TNM staging of neuroendocrine tumors of the stomach, AJCC 8th edition
        Definition / general
        • Well differentiated neuroendocrine tumors of the stomach are covered by this staging system
        • Not covered by this staging system are poorly differentiated neuroendocrine carcinomas of the stomach (click here)
        ICD coding
        • C7A.092: Malignant carcinoid tumor of the stomach
        Primary tumor (pT)
        • TX: Primary tumor cannot be assessed
        • T0: No evidence of primary tumor
        • T1: Invades the lamina propria or submucosa and is ≤ 1 cm in size
        • T2: Invades the muscularis propria or is > 1 cm in size
        • T3: Invades through the muscularis propria into subserosal tissue without penetration of overlying serosa
        • T4: Invades visceral peritoneum (serosa) or other organs or adjacent structures
        Regional lymph nodes (pN)
        • NX: Regional lymph nodes cannot be assessed
        • N0: No regional lymph node metastasis has occurred
        • N1: Regional lymph node metastasis

        Notes:
        • Regional lymph nodes depend on the tumor site within the stomach:
          • Greater curvature: greater curvature, greater omental, gastroduodenal, gastroepiploic, pyloric, and pancreaticoduodenal nodes
          • Pancreatic and splenic areas: pancreaticolienal, peripancreatic, and splenic nodes
          • Lesser curvature: lesser curvature, lesser omental, left gastric, cardioesophageal, common hepatic, celiac, and hepatoduodenal nodes
        Distant metastasis (pM)
        • M0: No distant metastasis
        • M1: Distant metastasis
          • M1a: Metastasis confined to liver
          • M1b: Metastasis in at least one extrahepatic site (e.g., lung, ovary, nonregional lymph node, peritoneum, bone)
          • M1c: Both hepatic and extrahepatic metastasis
        Prefixes
        • (m): multiple primary lesions (provide stage for the most advanced lesion)
        • r: recurrent tumor stage
        Stage grouping
        Stage I: T1 N0 M0
        Stage II: T2 - 3  N0 M0
        Stage III: T4 N0 M0
        any T N1 M0
        Stage IV:any T any N  M1
        Registry data collection variables
        • Size of tumor (value or unknown)
        • Depth of invasion
        • Nodal status and number of nodes involved, if applicable
        • Sites of metastasis, if applicable
        • Ki67 index
        • Mitotic count
        • Histologic grading (from Ki67 and mitotic count): G1, G2, G3
        • Preoperative pancreastatin level
        • Preoperative gastrin level
        • Preoperative CgA level
        • Type of gastric neuroendocrine tumor (NET) (I, II, or III)
        Histologic grade
        • Grading is not formally part of the staging system
        • Most pathologists use the European Neuroendocrine Tumor Society (ENETS) /WHO grading criteria:
          • Grade 1: Mitotic rate < 2 per 10 high power fields and Ki67 rate < 3%
          • Grade 2: Mitotic rate 2 - 20 per 10 high power fields or Ki67 rate 3 - 20%
          • Grade 3: Mitotic rate > 20 per 10 high power fields or Ki67 rate > 20%
        Histopathologic type
        • Neuroendocrine tumor (NET) G1 (carcinoid)
        • Neuroendocrine tumor (NET) G2
        Board review style question #1
          Based on AJCC 8th edition criteria, a well differentiated neuroendocrine tumor of the stomach that metastasized to regional lymph nodes and also to the lung would be staged as which of the following?

        1. pM0
        2. pM1a
        3. pM1b
        4. pM1c
        Board review style answer #1

        Well differentiated neuroendocrine tumor
        Definition / general
        • The 5th edition of the WHO 2019 Digestive Tumors Classification definition of neuroendocrine tumor (NET): well differentiated, grade 1, 2 or 3 based on mitotic rate per 2 mm2 (based on counting 10 mm2 and taking the average) and Ki67 immunohistochemical index (counted in ≥ 500 cells in the area of highest staining); grade assigned by whichever value is higher (see Diagrams / tables)
        • Must lack features of carcinoma, which includes poorly differentiated morphology, tumoral necrosis, high N/C ratio and prominent nucleoli
        Essential features
        • Type I is the most common, followed by type III, with type II being very rare
        • Type I is typically indolent and type II and III have a higher malignant potential (Surg Pathol Clin 2020;13:377)
        • Grade 3 neoplasms are no longer automatically categorized as a carcinoma per the updated 2019 WHO classification
        Terminology
        • Historically referred to as carcinoid
        • Can be a component of a mixed neuroendocrine nonneuroendocrine neoplasm (MiNEN); must represent ≥ 30% of tumor (Korean J Radiol 2019;20:5)
        ICD coding
        • ICD-10: D3A.092 - benign carcinoid tumor of the stomach
        Epidemiology
        • Gastric NETs often occur in the setting of certain diseases (Surg Pathol Clin 2020;13:377):
          • Type I:
            • Derived from enterochromaffin-like (ECL) cells
            • Occurs in a setting of chronic atrophic gastritis (type A) and hypergastrinemia
            • More frequent in women
          • Type II:
            • Derived from ECL cells
            • Occurs in a setting of hypergastrinemia due to Zollinger-Ellison syndrome
            • Can be seen in patients with multiple endocrine neoplasia type 1 (MEN1) syndrome
            • Affects men and women equally
          • Type III:
            • Sporadic
            • More frequent in men
          • Type IV:
            • Discussed in the literature but not currently recognized by the WHO
          • Serotonin producing EC cell NET:
            • Rare and usually nonfunctional
          • Gastrin producing G cell NET:
            • Usually nonfunctional but can cause Zollinger-Ellison syndrome and is then referred to as a gastric gastrinoma
        • All types tend to occur in the age range of 50 - 60 years
        Sites
        • Type I and II are more common in the gastric body and are often multifocal; usually < 2 cm
        • Type III occurs anywhere in the stomach and is usually unifocal; usually > 2 cm
        Pathophysiology
        • May arise in the setting of autoimmune gastritis (type I), Zollinger-Ellison syndrome (type II) or in the absence of a known precursor (type III) (Surg Pathol Clin 2020;13:377)
        Etiology
        Diagrams / tables

        Gastric NET grade Mitotic rate (per 2 mm2) Ki67 rate
        Grade 1 < 2 < 3%
        Grade 2 2 - 20 3 - 20%
        Grade 3 > 20 > 20%
        Clinical features
        • Tumor functionality is based on clinical symptoms, not on immunohistochemical expression of the hormone (Surg Pathol Clin 2020;13:377)
        • These tumors do not typically cause symptoms secondary to hormone secretion
        • Serum chromogranin A is used as biomarker to assess the bulk of disease and monitor treatment (World J Gastrointest Oncol 2020;12:791)
        Diagnosis
        • Typically found on endoscopic exam
        • Neuroendocrine neoplasms test (NETest) is a multianalyte liquid biopsy that measures neuroendocrine tumor gene expression in blood and can be used as a diagnostic and disease surveillance test (Endocrinol Metab Clin North Am 2018;47:485)
        Laboratory
        • Type I and type II will typically have elevated serum gastrin levels
        Prognostic factors
        Case reports
        • 37 year old woman presented with upper gastrointestinal bleed and epigastric pain (Int J Surg Case Rep 2016;25:62)
        • 45 year old woman with autoimmune pernicious anemia and Hashimoto thyroiditis monitored by upper endoscopy (Cureus 2021;13:e13556)
        • 56 year old woman with a history of cholelithiasis and irritable bowel syndrome presented with postprandial, colicky left upper quadrant pain radiating to the right shoulder lasting approximately 45 minutes (Cureus 2019;11:e4193)
        • 66 year old man with gastric NET presented as a subepithelial tumor mimicking a gastrointestinal stromal tumor (Case Rep Oncol 2021;14:1271)
        • 68 year old man with an incidental finding of a small nodule in the gastric fundus (Int J Surg Case Rep 2020;75:361)
        Treatment
        Clinical images

        Images hosted on other servers:

        Polyps and atrophic gastritis

        Gross description
        Microscopic (histologic) description
        • Architecturally, arranged in nests, cords or trabeculae
        • Bland, round to oval cells with typical salt and pepper chromatin and amphophilic cytoplasm
        • Focal endocrine atypia is acceptable
        • Prominent vasculature is often seen (Front Oncol 2013;3:2)
        • Type I NET background oxyntic mucosa is atrophic with metaplasia (intestinal type most commonly), with ECL cell hyperplasia
          • Correlates with endoscopic impression of atrophy
        • Type II NET background oxyntic mucosa is hyperplastic with ECL cell hyperplasia
          • Correlates with endoscopic impression of hypertrophic mucosal folds
        • Type III NET background oxyntic mucosa is normal without ECL cell hyperplasia (Medicine (Baltimore) 2022;101:e28550)
        Microscopic (histologic) images

        Contributed by Dana Razzano, M.D. and @RaulSGonzalezMD on Twitter

        Nests of neuroendocrine tumor cells

        Chromogranin stain

        Well differentiated neuroendocrine tumor Well differentiated neuroendocrine tumor Well differentiated neuroendocrine tumor

        Well differentiated neuroendocrine tumor


        Well differentiated neuroendocrine tumor Well differentiated neuroendocrine tumor

        Well differentiated neuroendocrine tumor

        Negative stains
        Molecular / cytogenetics description
        • Mutations in MEN1, DAXX and ATRX are entity defining for well differentiated NETs (Histopathology 2020;76:182)
        • Small cohort studies show associations with germline mutation in the ATP4A gene, with other studies demonstrating loss of heterozygosity at chromosome 11p13 (MEN1 locus) and hypermethylation at the CDKN2A locus as possibly frequent events in gastric NETs (Endocr Rev 2019;40:506)
        • Lack the common TP53 and Rb mutation found in most gastric neuroendocrine carcinomas (Histopathology 2020;76:182)
        Videos

        Neuroendocrine nuttiness in the digestive system - Dr. Raul S. Gonzalez

        GI neuroendocrine tumors classification - Dr. Vikram Deshpande

        Sample pathology report
        • Stomach, partial gastrectomy:
          • Well differentiated neuroendocrine tumor, WHO grade 1, 1.8 cm, excised (see comment and synoptic table)
          • Oxyntic mucosa with moderate chronic gastritis
          • Mild intestinal metaplasia, incomplete type
          • Severe atrophy
          • No Helicobacter organisms on immunohistochemistry
          • Comment: Histologic sections of the stomach demonstrate involvement by nests of tumor cells with round nuclei, variably prominent nucleoli and abundant clear to eosinophilic cytoplasm, consistent with a well differentiated neuroendocrine tumor. Immunostains show that the tumor cells are positive for synaptophysin, chromogranin and INSM1. In accordance with recommended criteria for grading neuroendocrine tumors, mitotic activity assessed within 50 high power fields (10 mm2) on the resection specimen is < 1 mitotic figure per 10 high power fields (2 mm2); Ki67 index, assessed in 2,000 cells in the area of highest proliferation, is < 1%. Based on these proliferation indices, the tumor is best considered WHO grade 1.
          • The background gastric mucosa shows moderate chronic gastritis with pseudopyloric and intestinal metaplasia. Linear and nodular endocrine cell hyperplasia are also present, confirmed by immunohistochemical stain for chromogranin. Gastrin immunostain is negative, indicating oxyntic mucosa with severe atrophy. The features are consistent with autoimmune gastritis in the appropriate clinical context. Correlation with clinical and laboratory findings is recommended.
        Differential diagnosis
        Board review style question #1

        A 51 year old symptomatic man was found to have multiple gastric polyps on screening esophagogastroduodenoscopy (EGD). One polyp showed an aggregate of nested bland tumor cells (seen in the figure above) that stained positive for synaptophysin and chromogranin. What is the appropriate way to grade this tumor?

        1. Ki67 and mitotic figure count, whichever is higher
        2. Ki67 and mitotic figure count, whichever is lower
        3. Ki67 index alone
        4. Mitotic figure count
        Board review style answer #1
        A. Ki67 and mitotic figure count, whichever is higher

        Comment Here

        Reference: Well differentiated neuroendocrine tumor
        Board review style question #2
        What is the most common type of gastric neuroendocrine neoplasm?

        1. Gastric well differentiated NET type I
        2. Gastric well differentiated NET type II
        3. Gastric well differentiated NET type III
        4. Gastrin producing G cell NET
        5. Serotonin producing EC cell NET
        Board review style answer #2
        A. Gastric well differentiated NET type I

        Comment Here

        Reference: Well differentiated neuroendocrine tumor

        WHO classification
        Table of Contents
        Definition / general | WHO (2019)
        Definition / general
        • WHO classification of tumors of the stomach
        • Currently on 5th edition, published in 2019
        • Based on histologic appearance, not molecular characteristics
        WHO (2019)

          Malignant epithelial tumors ICD-O codes
        • Adenocarcinoma, NOS 8140/3
          • Tubular adenocarcinoma 8211/3
          • Parietal cell carcinoma 8214/3
          • Adenocarcinoma with mixed subtypes 8255/3
          • Papillary adenocarcinoma, NOS 8260/3
          • Micropapillary carcinoma, NOS 8265/3
          • Mucoepidermoid carcinoma 8430/3
          • Mucinous adenocarcinoma 8480/3
          • Signet ring cell carcinoma 8490/3
          • Poorly cohesive carcinoma 8490/3
          • Medullary carcinoma with lymphoid stroma 8512/3
          • Hepatoid adenocarcinoma 8576/3
          • Paneth cell carcinoma
        • Squamous cell carcinoma, NOS 8070/3
        • Adenosquamous carcinoma 8560/3
        • Carcinoma, undifferentiated, NOS 8020/3
          • Large cell carcinoma with rhabdoid phenotype8014/3
          • Pleomorphic carcinoma 8022/3
          • Sarcomatoid carcinoma 8033/3
          • Carcinoma with osteoclast-like giant cells8035/3
        • Gastroblastoma 8976/1
        • Neuroendocrine tumor, NOS 8240/3
          • Neuroendocrine tumor, grade 1 8240/3
          • Neuroendocrine tumor, grade 2 8249/3
          • Neuroendocrine tumor, grade 3 8249/3
          • Gastrinoma, NOS 8153/3
          • Somatostatinoma, NOS 8156/3
          • Enterochromaffin cell carcinoid 8241/3
          • ECL cell carcinoid, malignant 8242/3
        • Neuroendocrine carcinoma, NOS 8246/3
          • Large cell neuroendocrine carcinoma 8013/3
          • Small cell neuroendocrine carcinoma 8041/3
        • Mixed neuroendocrine - nonneuroendocrine neoplasm (MiNEN) 8154/3

        Xanthoma
        Definition / general
        • Gastric xanthoma (GX) is a benign mucosal tumor-like lesion composed of foamy macrophages within the lamina propria
        Essential features
        • Benign tumor-like lesion most commonly occurring in the antrum of the stomach
        • Endoscopically seen as small, yellow-white nodules or plaque-like lesions
        • Characterized histologically by accumulation of foamy macrophages (xanthoma cells) within the lamina propria
        • Almost always associated with background gastric pathology
        Terminology
        • Xanthelasma, lipid islands
        ICD coding
        • ICD-10: K31.9 - disease of stomach and duodenum, unspecified
        Epidemiology
        • First described as lipid laden macrophages in the gastric mucosa by Orth in 1887
        • Mean age of 60 years (53% between 40 and 60 years old)
        • Rarely seen in the pediatric population
        • Male predominance (3:1)
        • Incidence is quite variable depending on the study but thought to be a relatively uncommon incidental finding at endoscopy with an estimated incidence of 0.02 - 0.8%; however, autopsy studies have reported rates of 1.9% and 58%
        • Background mucosal pathology can be seen, such as Helicobacter pylori gastritis, intestinal metaplasia, atrophic gastritis or gastric ulcers
        • May also be associated with metachronous or synchronous early gastric cancer and rapid growth of gastric cancer (J Clin Med 2021;10:5704, Can J Gastroenterol Hepatol 2020;2020:3578927)
        • May be seen after partial gastrectomy
        • Pathologists should carefully examine the background mucosa for additional pathologic findings when diagnosing gastric xanthoma since this entity does not occur in normal gastric mucosa
        • References: Can J Gastroenterol Hepatol 2020;2020:3578927, Cureus 2022;14:e25026, Oxf Med Case Reports 2018;2018:omy051, J Pediatr Gastroenterol Nutr 2021;73:e26
        Sites
        Pathophysiology
        • Pathogenesis and clinical significance remain unclear
        • Given the association with background mucosal pathology, one hypothesis suggests that gastric xanthoma is a mucosal reaction to prior injury or inflammation
        • When the mucosa is damaged, release of lipid rich material may be phagocytosed and hence, create foamy macrophages
        • Another hypothesis suggests that phagocytosis of Helicobacter pylori organisms that have penetrated the lamina propria induces the macrophages to become foamy
        • Cutaneous xanthoma and xanthelasma are both strongly associated with disorders of lipid metabolism; however, data regarding such an association in gastric xanthoma is less clear
        • References: Cureus 2022;14:e25026, Oxf Med Case Reports 2018;2018:omy051, Bosn J Basic Med Sci 2012;12:127, Can J Gastroenterol Hepatol 2020;2020:3578927, Am J Gastroenterol 2001;96:3216
        Etiology
        • The exact etiology, pathogenesis and clinical significance of gastric xanthoma is unclear but usually coexists with other gastric pathology such as Helicobacter pylori gastritis, atrophic gastritis, gastric ulcers, intestinal metaplasia and early gastric cancer (see Epidemiology and Pathophysiology)
        Clinical features
        • Most commonly an incidental finding at endoscopy for another reason
        Diagnosis
        Prognostic factors
        • Gastric xanthoma is completely benign and nonneoplastic; however, background gastric pathology may be present
        Case reports
        Treatment
        • No treatment is required for gastric xanthoma as it is a benign condition; treatment is aimed at coexistent gastric pathology
        Clinical images

        Images hosted on other servers:

        Endoscopic yellow plaque-like lesion

        Endoscopic yellow nodule and plaque-like lesion

        Gross description
        Microscopic (histologic) description
        Microscopic (histologic) images

        Contributed by Aaron R. Huber, D.O.
        Lamina propria macrophages Lamina propria macrophages Lamina propria macrophages Lamina propria macrophages

        Foamy macrophages within the lamina propria

        CD68

        CD68

        Positive stains
        Negative stains
        Sample pathology report
        • Stomach, nodule, biopsy:
          • Gastric xanthoma (see comment)
          • Comment: H&E stained sections demonstrate clusters of foamy macrophages within the lamina propria. CD68 immunohistochemical stain highlights the macrophages, while keratin AE1 / AE3 is negative within these cells. The morphologic and immunophenotypic findings are compatible with gastric xanthoma.
        Differential diagnosis
        Board review style question #1

        A 56 year old man underwent an esophagogastroduodenoscopy for reflux symptoms. A biopsy was taken of a small, sessile yellow-white nodule in the stomach. The histology is depicted in the H&E image. The unusual stroma cells were keratin negative and CD68 and CD163 positive. What is the best diagnosis?

        1. Granular cell tumor
        2. Metastatic breast carcinoma
        3. Normal gastric mucosa
        4. Signet ring cell carcinoma
        5. Xanthoma
        Board review style answer #1
        E. Xanthoma. The H&E image demonstrates an accumulation of lipid laden macrophages within the lamina propria indicative of xanthoma. Answer B is incorrect because although metastatic breast carcinoma, especially lobular carcinoma, is common in the stomach, keratin would be positive and CD68 or CD163 would be negative. In addition, more specific markers such as GATA3 would be positive. Answer D is incorrect because although signet ring cell carcinoma is one of the top differential diagnostic considerations, the cells in signet ring cell carcinoma are cytologically malignant, whereas those in gastric xanthoma are bland with abundant foamy cytoplasm. In addition, the signet ring carcinoma cells would be keratin positive and CD68 and CD163 negative. Answer A is incorrect because granular cell tumor is more commonly seen in the esophagus and has granular cytoplasm with pyknotic nuclei. These cells are CD68 positive but also S100 positive, unlike xanthoma cells (S100 negative). Answer C is incorrect because normal gastric mucosa would not demonstrate these findings.

        Comment Here

        Reference: Xanthoma
        Board review style question #2
        A 45 year old woman underwent an esophagogastroduodenoscopy for reflux symptoms. A biopsy was taken of a yellow-white plaque-like lesion in the gastric antrum. The histology demonstrated antral mucosa with clusters of foamy histiocytes within the lamina propria. Which of the following is true about the pathologic entity described?

        1. May be associated with Helicobacter pylori gastritis
        2. Most commonly occurs in teenagers and is rare in adults
        3. Occurs commonly in patients with disorders of lipid metabolism, similar to cutaneous examples
        4. Occurs in normal gastric mucosa
        Board review style answer #2
        A. May be associated with Helicobacter pylori gastritis. Answer D is incorrect because gastric xanthoma (GX) essentially never occurs in normal gastric mucosa and may be associated with Helicobacter gastritis, atrophic gastritis, gastric ulcers, intestinal metaplasia and gastric cancer. Answer B is incorrect because gastric xanthoma has been reported in children; however, it is much more common in adults. Answer C is incorrect because unlike cutaneous examples of xanthelasma or xanthoma, evidence linking gastric xanthoma with disorders of lipid metabolism is inconclusive.

        Comment Here

        Reference: Xanthoma

        Zollinger-Ellison
        Definition / general
        • Duodenal ulceration, diarrhea and gastric gland hyperplasia caused by serum hypergastrinemia from pancreatic or duodenal neoplasm, or rarely by primary G cell hyperplasia of antral mucosa
        • Grossly and radiologically resembles Menetrier's disease
        • Associated with MEN1 syndrome and multicentric carcinoid tumors (Am J Surg Pathol 1990;14:503)
        • Intramucosal cysts in body in 70%, may give rise to fundic gland polyps, severity associated with serum gastrin levels (Hum Pathol 2000;31:140)
        Gross description
        • Enlarged fundic mucosal folds with cerebriform pattern
        Microscopic (histologic) description
        • Hyperplasia primarily of parietal cells in fundic glands
        • Normal antral glands and gastric pits, often increase in enterochromaffin-like cells
        Back to top
        Recent Stomach Pathology books

        Arnold: 2019

        IARC: 2019

        Lamps: 2015

        Lauwers: 2021

        Montgomery: 2017

        Montgomery: 2018

        Montgomery: 2017

        Noffsinger: 2017

        Odze: 2022

        Srivastava: 2023

        Yantiss: 2021



        Find related Pathology books: GI, liver
        Image 01 Image 02